3CK PART II Flashcards

1
Q

Explain relationship between hyponatremia and thyroid

A

Hypothyroidism can cause hyponatremia due to decreased clearance of free water and increased release of antidiuretic hormone.

Euvolemic hyponatremia.

How well did you know this?
1
Not at all
2
3
4
5
Perfectly
2
Q

Algorithm hyponatremia

A

DO

How well did you know this?
1
Not at all
2
3
4
5
Perfectly
3
Q

Postpartum thyroiditis presentation

A

Is a variant of hashimoto

brief thyrotoxic phase, followed by a self-limited hypothyroid phase and eventual return to a euthyroid state.

If mild, no need of supplementation
If sever needs levothyroxine

How well did you know this?
1
Not at all
2
3
4
5
Perfectly
4
Q

patient hypotense, tachycardi and blunt thoracic trauma , next step in management?

A

FAST! lead to detection of other causes that can be life threatening-pneumothorax, aortic dissection, hemoperitoneum, pericardial effusion leading to tamponade

  • FAST is not only in abdomen
  • Chest X rays are often done, but FIRST FAST
How well did you know this?
1
Not at all
2
3
4
5
Perfectly
5
Q

What are the preferred Chest X rays in trauma?

A

bedside (ie, anteroposterior) chest x-rays are typically performed as obtaining posteroanterior and lateral chest x-rays requires patients to stand

How well did you know this?
1
Not at all
2
3
4
5
Perfectly
6
Q

type I error, definition and ways that it is increased

A

rejecting a true null hypothesis [false positive]

increases as sample size increases

How well did you know this?
1
Not at all
2
3
4
5
Perfectly
7
Q

type II error

A

probability of failing to reject a false null hypothesis [false negative

How well did you know this?
1
Not at all
2
3
4
5
Perfectly
8
Q

2 ways in which Nonresponse increases the potential for bias

A

when nonrespondents differ from respondents in the outcome of interest; and it contributes to a decrease in statistical power given that the sample size has been reduced. However, a significant nonresponse rate in a study does not necessarily translate into bias.

How well did you know this?
1
Not at all
2
3
4
5
Perfectly
9
Q

Patient with alcoholism that after treatment with intravenous glucose, thiamine and fluids develops weakness of arms

A

Its due to hypophosphatemia ( refeeding syndrome) which can lead to development of rhabdomyolisis due to myopathy.

  • check CPK in this setting.
How well did you know this?
1
Not at all
2
3
4
5
Perfectly
10
Q

Absolute iron deficiency is defined as

A

transferrin saturation <20% or ferritin <100 ng/mL.

How well did you know this?
1
Not at all
2
3
4
5
Perfectly
11
Q

Causes of anemia in ESDR

A
MC decreased EPO production
BUT ALSO: 
iron deficiency ( blood loss, frequent blood testing, dyalisis, functional
severe hyperPTH( resistance to EPO)
Folate deficiency
systemic inflammation 
aluminium toxicity
How well did you know this?
1
Not at all
2
3
4
5
Perfectly
12
Q

functional iron deficiency

A

Can occur in ESRD but these can also have the normal iron deficiency.

In functional-> unable to mobilize iron from stores.

How well did you know this?
1
Not at all
2
3
4
5
Perfectly
13
Q

When do you start EPO in ESDR?What considerations are needed before starting it?

A

Hemoglobin < 10

The goal is to increase hemoglobin by 1.5-2 g/dL over 4-6 weeks to target hemoglobin to 10-11.5 g/dL.

How well did you know this?
1
Not at all
2
3
4
5
Perfectly
14
Q

When do you supplement with iron in ESDR?

A

Iron supplementation is recommended for ESRD patients with transferrin saturation <30% and ferritin <500 ng/mL

** Think on the patient who despite receiving EPO keeps going with anemia.

How well did you know this?
1
Not at all
2
3
4
5
Perfectly
15
Q

Stress urinary incontinence tto

A

Intrinsic sphincter deficiency and urethral hypermobility

Lifestyle modification
Pelvic floor exercises
Pessary
Urethral sling surgery

How well did you know this?
1
Not at all
2
3
4
5
Perfectly
16
Q

Urgency urinary incontinence tto

A

Lifestyle modification
Bladder training
Antimuscarinic medications

How well did you know this?
1
Not at all
2
3
4
5
Perfectly
17
Q

Overflow urinary incontinence tto

A

Intermittent catheterization

Correct underlying etiology

How well did you know this?
1
Not at all
2
3
4
5
Perfectly
18
Q

Patients with an underlying neuropathy can develop overflow incontinence when additional risk factors (eg, antihistamines) result in exacerbation of symptoms

A

t

How well did you know this?
1
Not at all
2
3
4
5
Perfectly
19
Q

Overflow incontinence- post void residual volume

A

> 150

How well did you know this?
1
Not at all
2
3
4
5
Perfectly
20
Q

Presentation Genitourinary syndrome of menopause

A
urinary incontinence PLUS
vaginal atrophy
dyspareunia, 
vulvar irritation
pelvic organ prolapse
How well did you know this?
1
Not at all
2
3
4
5
Perfectly
21
Q

Normal JVD

A

6-8

How well did you know this?
1
Not at all
2
3
4
5
Perfectly
22
Q

notched (bifid) P waves in lead II

A

left atrial enlargement due to mitral stenosis

How well did you know this?
1
Not at all
2
3
4
5
Perfectly
23
Q

Mitral stenosis in women think of

A

rheumatic fever

How well did you know this?
1
Not at all
2
3
4
5
Perfectly
24
Q

hear murmur + dyspnea, orthopnea, hemoptysis , hoarseness

A

mitral stenosis

How well did you know this?
1
Not at all
2
3
4
5
Perfectly
25
Q

Auscultation and ECG findings of MS

A

Auscultation: loud S1, loud P2if pulmonary hypertension
Mid-diastolic murmur

ECG: notched p wave
atrial tachyarryhtmias, high R’S IN v1,v2

How well did you know this?
1
Not at all
2
3
4
5
Perfectly
26
Q

Cardiac manifestations Noonan syndrome

A

pulmonic stenosis, atrial septal defects, and hypertrophic cardiomyopathy.

How well did you know this?
1
Not at all
2
3
4
5
Perfectly
27
Q

Indications of IVC filters

A

In whom anti coagulation is contraindicated :recent surgery, hemorrhagic stroke, bleeding diathesis, active bleeding

How well did you know this?
1
Not at all
2
3
4
5
Perfectly
28
Q

goal of IVC filters

A

inhibit progression of lower extremity clots through the IVC toward the lungs.

How well did you know this?
1
Not at all
2
3
4
5
Perfectly
29
Q

Acute and long term complications of IVC filter placement

A

acute: acute insertion site thrombosis, hematoma, arteriovenous fistula

long term recurrent DVTs andIVC thrombosis.

IVC filters do not appear to affect overall mortality significantly.

The filter can prevent clot progression to a pulmonary embolism, but it does not prevent future DVTs or treat the underlying thrombotic predisposition.

How well did you know this?
1
Not at all
2
3
4
5
Perfectly
30
Q

heteroplasmy - mitochondrial disorders

A

mom passess diasease to both male and women, but then her female child is the only one able to pass to the offspring.

How well did you know this?
1
Not at all
2
3
4
5
Perfectly
31
Q

Stroke + Seizures

A

MELAS

How well did you know this?
1
Not at all
2
3
4
5
Perfectly
32
Q

MELAS presentation

A

Mitochondrial encephalomyopathy with lactic acidosis and stroke like episodes

  • stroke
  • seizures
  • weakness
  • hearing loss
  • lactic acidosis
How well did you know this?
1
Not at all
2
3
4
5
Perfectly
33
Q

Corrected calcium formula

A

Corrected calcium in mg/dL = (measured total calcium) + 0.8 (4.0 g/dL - measured serum albumin in g/dL).

calcium concentration decreases by 0.8 mg/dL for every 1 g/dL decrease in serum albumin concentration.

How well did you know this?
1
Not at all
2
3
4
5
Perfectly
34
Q

ss hypocalcemia

A

oral paresthesias, carpopedal spasm, tetany, seizures

How well did you know this?
1
Not at all
2
3
4
5
Perfectly
35
Q

tto hypocalcemia

A

Oral calcium citrate or carbonate mild acute hypocalcemia (corrected calcium of 7.5-8.5 mg/dL) or chronic hypocalcemia.

for symptomatic : Intravenous calcium gluconate

How well did you know this?
1
Not at all
2
3
4
5
Perfectly
36
Q

distribution of calcium in the body

A

albumin-bound calcium (45%), ionized calcium (40%), or calcium bound to inorganic and organic anions (15%).

How well did you know this?
1
Not at all
2
3
4
5
Perfectly
37
Q

organism in chagas

A

PROTOZOAN-Tripanosoma cruzi

How well did you know this?
1
Not at all
2
3
4
5
Perfectly
38
Q

Cardiac manifestations of chagas

A
  • Ventricular apical aneurysm in the absence of coronary disease
  • Biventricular heart failure (R>L) with cardiomegaly
  • Mural thrombosis with embolic complications
  • Fibrosis leading to conduction abnormality ( heart block , ventricular tachycardia)
How well did you know this?
1
Not at all
2
3
4
5
Perfectly
39
Q

Cardiac manifestations of lyme

A

atrioventricular conduction block. In rare cases, patients can also develop myopericarditis with dilated cardiomyopathy.

How well did you know this?
1
Not at all
2
3
4
5
Perfectly
40
Q

Infection with human immunodeficiency virus has been associated with the development of dilated cardiomyopathy.

A

T

How well did you know this?
1
Not at all
2
3
4
5
Perfectly
41
Q

Diagnostic criteria for C difficile

A

Characteristic symptoms – Watery diarrhea (≥3 loose stools in 24 hours) with or without lower abdominal pain, low-grade fever, and leukocytosis
AND
Positive stool testing

Positive laboratory testing in the absence of symptoms is not sufficient for diagnosis as asymptomatic carriage is common.

How well did you know this?
1
Not at all
2
3
4
5
Perfectly
42
Q

4 RF for C.difficile

A

PPIs – C difficile spores are acid-resistant, but proton pump inhibitors are thought to alter the colonic microbiome, which increases the risk of C difficile proliferation.

Recent antibiotic use – Antibiotics disrupt the barrier function of normal colonic flora. Fluoroquinolones, clindamycin, and broad-spectrum penicillins/cephalosporins are most likely to cause CDAD.

Hospitalization

Advanced age

How well did you know this?
1
Not at all
2
3
4
5
Perfectly
43
Q

Medications related to C. difficile

A

Fluoroquinolones, clindamycin, and broad-spectrum penicillins/cephalosporins

How well did you know this?
1
Not at all
2
3
4
5
Perfectly
44
Q

Newborn with red sac in lumbar spine, dx?

A

Spina bifida = myelomeningocele

How well did you know this?
1
Not at all
2
3
4
5
Perfectly
45
Q

Complications of spina bifida (4)

A

Motor/sensory dysfunction
Neurogenic bladder/bowel ( urinary retention/constipation)
Hydrocephalus
Scoliosis

How well did you know this?
1
Not at all
2
3
4
5
Perfectly
46
Q

tto spina bifida and its complciations

A

Surgical closure
Clean intermittent catheterization
Scheduled laxatives/enemas
Bracing; correction of deformities

Patients undergo urgent surgical repair, which results in hydrocephalus requiring a ventriculoperitoneal shunt in over half of infants.

How well did you know this?
1
Not at all
2
3
4
5
Perfectly
47
Q

Presentation of transverse myelitis

A

Rapidly progressive weakness of the lower extremities following an upper respiratory infection, accompanied by sensory loss and urinary retention

muscle flaccidity and hyporeflexia, but spasticity and hyperreflexia develop subsequently.

How well did you know this?
1
Not at all
2
3
4
5
Perfectly
48
Q

type of patients that can present with epidural abscess

A

IV drug users

How well did you know this?
1
Not at all
2
3
4
5
Perfectly
49
Q

crescent, hypoechoic lesions adjacent to the gestational sac

A

subchorionic hematoma

abnormal collection of blood between the uterus and the gestational sac

How well did you know this?
1
Not at all
2
3
4
5
Perfectly
50
Q

management of subchorionic hematoma

A

management is expectant; patients can be followed with serial ultrasounds to help provide reassurance.

**DO NOT RECOMMEND HOSPITALIZATION OR BED REST- DOESNT CHANGE ANYTHING, AND INCREASES RISK OF DVT

How well did you know this?
1
Not at all
2
3
4
5
Perfectly
51
Q

Kleihauer-Betke test

A

determines the amount of fetomaternal hemorrhage that occurred due to delivery, maternal trauma, or first-trimester bleeding in an Rh-negative mother.

determines the amount of Rho(D) immune globulin to be administered.

How well did you know this?
1
Not at all
2
3
4
5
Perfectly
52
Q

Pts with subchorionic hematoma are at higher risk of

A
Spontaneous abortion
Abruptio placentae
Preterm premature rupture of membranes
Preterm delivery
preeclampsia, fetal growth restriction, and intrauterine fetal demise
How well did you know this?
1
Not at all
2
3
4
5
Perfectly
53
Q

RF for subchorionic hematoma

A

Infertility treatment
Anticoagulation
Uterine anomalies
Recurrent pregnancy loss

How well did you know this?
1
Not at all
2
3
4
5
Perfectly
54
Q

RF Placenta previa

A

prior placenta previa.,history of prior cesarean delivery, multiple gestations, multiparity

How well did you know this?
1
Not at all
2
3
4
5
Perfectly
55
Q

RF Placenta accreta

A

previous cesarea

How well did you know this?
1
Not at all
2
3
4
5
Perfectly
56
Q

Indication for Fetal fibronectin

A

cervicovaginal discharge of patients at <34 weeks gestation with preterm contractions

not older than 34

How well did you know this?
1
Not at all
2
3
4
5
Perfectly
57
Q

patient third trimester bleeding, fetus ok, next step?

A

transvaginal US- likely caused by placenta previa

Transvaginal ultrasound and speculum examination are safe in placenta previa as neither the transvaginal probe nor the speculum enters the endocervical canal. DIGITAL EXAM IS CONTRAINDICATED

How well did you know this?
1
Not at all
2
3
4
5
Perfectly
58
Q

Woman that after birth- shortness of breath, unresponsive, hypotension

A

Amniotic fluid embolism:

Cardiogenic shock
Hypoxemic respiratory failure
Disseminated intravascular coagulopathy
Coma or seizures

How well did you know this?
1
Not at all
2
3
4
5
Perfectly
59
Q

Management of amniotic fluid embolism

A

Respiratory & hemodynamic support

± Transfusion

How well did you know this?
1
Not at all
2
3
4
5
Perfectly
60
Q

RF for amniotic fluid embolism

A

cesarean/operative delivery, placenta previa, and abruptio placentae.

How well did you know this?
1
Not at all
2
3
4
5
Perfectly
61
Q

Peripartum cardiomyopathy

A

in the late third trimester or early postpartum period with symptoms of heart failure (eg, dyspnea, orthopnea, hemoptysis, pedal edema)

How well did you know this?
1
Not at all
2
3
4
5
Perfectly
62
Q

Initial management of aspiration pneumonia:

A

blood and sputum cultures with initiation of broad-spectrum antibiotics with anaerobic coverage (eg, clindamycin).

How well did you know this?
1
Not at all
2
3
4
5
Perfectly
63
Q

Patient diabetic with sinusitis, next step

A

depends on glycemic control:

If adequate glucose: amoxi-clavulanate for 5-7 days

If not control ( i.e DKA)- think of mucor - and admit to hospital for antifungals

How well did you know this?
1
Not at all
2
3
4
5
Perfectly
64
Q

Acute bacterial rhinosinusitis tto

A

1st-line therapy: Amoxicillin-clavulanate (5-7d)
Alternate agent: Doxycycline or fluoroquinolones
Supportive care: Analgesics, decongestants, saline irrigation, topical glucocorticoids

How well did you know this?
1
Not at all
2
3
4
5
Perfectly
65
Q

Dx Acute bacterial rhinosinusitis

A

> 1 of the following is present:

Persistent symptoms/signs of rhinosinusitis for >10 days

Severe symptoms, high fever (>39 C [102.2 F]), purulent nasal discharge, and/or facial pain for >3 consecutive days

“Double sickening” - initial improvement of viral upper respiratory symptoms for 5-6 days, followed by clinical deterioration (eg, worsened fever, headache, nasal discharge)

How well did you know this?
1
Not at all
2
3
4
5
Perfectly
66
Q

MC organisms in bacterial rhinosinusitis

A

Streptococcus pneumoniae, Haemophilus influenzae, or Moraxella catarrhalis.

How well did you know this?
1
Not at all
2
3
4
5
Perfectly
67
Q

ST elevation

A

ST-elevation myocardial infarction (STEMI) is diagnosed by ECG showing >1 mm (0.1 mV) ST elevation (>2 mm in leads V2 and V3) in ≥2 anatomically contiguous leads.

How well did you know this?
1
Not at all
2
3
4
5
Perfectly
68
Q

If suspicion of perforated peptic ulcer, next step

A

Chest and Abdomen X ray

How well did you know this?
1
Not at all
2
3
4
5
Perfectly
69
Q

sudden-onset abdominal pain with significant tenderness and guarding

A

peritonitis

How well did you know this?
1
Not at all
2
3
4
5
Perfectly
70
Q

management of peptic ulcer perforation

A

Fluids, antibiotics with coverage for gram negative, PPIs IN preparation for surgery

How well did you know this?
1
Not at all
2
3
4
5
Perfectly
71
Q

cause of small intestinal obstruction in patients with a history of abdominal surgery.

A

adhesions

How well did you know this?
1
Not at all
2
3
4
5
Perfectly
72
Q

Conditions associated with Down Syndrome

A
VSD/ASD
duodenal atresia,
 Hirschsprung's disease, 
atlanto-axial instability,
 and hypothyroidism.
How well did you know this?
1
Not at all
2
3
4
5
Perfectly
73
Q

Pts with Down syndrome on the long run are at increased risk of ..

A
Acute Leukemia 
Alzheimer
ADHD 
Autism 
seizure disorder 
depression
How well did you know this?
1
Not at all
2
3
4
5
Perfectly
74
Q

Management of dyspepsia

A

Age ≥60: Upper endoscopy
Age <60:
Testing and treatment for H pylori
Upper endoscopy in high-risk patients (eg, overt GI bleeding, significant weight loss, >1 alarm symptom)

How well did you know this?
1
Not at all
2
3
4
5
Perfectly
75
Q

presentation of dyspepsia

A

Epigastric pain often described as “burning”

± Nausea, vomiting, epigastric fullness, heartburn

How well did you know this?
1
Not at all
2
3
4
5
Perfectly
76
Q

pelvic organ prolapse

A

Weight loss
Pelvic floor exercises
Vaginal pessary
Surgical repair

How well did you know this?
1
Not at all
2
3
4
5
Perfectly
77
Q

rectocele presentation

A

posterior vaginal mass that increases with the Valsalva maneuver.
pelvic pressure, lower back pain, constipation, and fecal incontinence.

How well did you know this?
1
Not at all
2
3
4
5
Perfectly
78
Q

rectocele treatment

A

Pelvic floor exercises
Vaginal pessary
Surgical repair

How well did you know this?
1
Not at all
2
3
4
5
Perfectly
79
Q

What findings are associated with poor prognosis in CLL

A

lymphadenopathy, organomegaly, and anemia/thrombocytopenia,

How well did you know this?
1
Not at all
2
3
4
5
Perfectly
80
Q

erythrocytosis and hematuria

A

Renal cell carcinoma

lank pain, hematuria, and palpable abdominal mass; h

How well did you know this?
1
Not at all
2
3
4
5
Perfectly
81
Q

suspect renal ca, next step

A

CT scan of the abdomen

How well did you know this?
1
Not at all
2
3
4
5
Perfectly
82
Q

JAK2 mutation is seen in which condition

A

Polycytemia vera

How well did you know this?
1
Not at all
2
3
4
5
Perfectly
83
Q

polycythemia vera presentation

A

Erythrocytosis is the defining feature; patients also often

aquagenic pruritus, hypertension, and arterial or venous thrombus.

How well did you know this?
1
Not at all
2
3
4
5
Perfectly
84
Q

CT scan of the abdomen reveals an enhancing mass of the kidney with thickened, irregular septa, raising strong suspicion for renal cell carcinoma, next step

A

referral to nephrology for nephrectomy!

How well did you know this?
1
Not at all
2
3
4
5
Perfectly
85
Q

Polycystic kidney disease tto

A

ACE inhibitors- reduce risk of chronic renal insufficiency.

How well did you know this?
1
Not at all
2
3
4
5
Perfectly
86
Q

management of kidney cysts.

A

Complex kidney cysts often require surveillance with repeat imaging

How well did you know this?
1
Not at all
2
3
4
5
Perfectly
87
Q

viral conjunctivitis, when does it stop to be contagious?

A

when watery discharge resolves

Other symptoms, such as morning crusting or eye redness, may persist longer than eye discharge but do not contribute to infectivity

How well did you know this?
1
Not at all
2
3
4
5
Perfectly
88
Q

Difference between allergic vs nonallergic rhinitis

A

Allergic:
earlier age (<20) with predominant eye symptoms (eg, watery eyes), sneezing, nasal congestion, and watery rhinorrhea. Patients usually can identify a trigger
PE:normal or pale blue nasal mucosa with pallor and occasionally shows polyps
TTO: allergen avoidance and intranasal glucocorticoids

Non allergic after age 20 with nasal blockage, rhinorrhea, and postnasal drip.. throughout the year.
PE:nasal mucosa can appear normal or boggy and erythematous on examination. topical intranasal glucocorticoids (eg, fluticasone) or intranasal antihistamine (eg, azelastine).

How well did you know this?
1
Not at all
2
3
4
5
Perfectly
89
Q

Tto of allergic and nonallergic rhinitis

A

aLLERGIC: Avoid trigger, and intranasal glucocorticoids

NON ALLERGIC : topical intranasal glucocorticoids (eg, fluticasone) or intranasal antihistamine (eg, azelastine).

How well did you know this?
1
Not at all
2
3
4
5
Perfectly
90
Q

When to consider allergy testing/IgE in rhinitis

A

reserved for differentiating between NAR and AR in patients who do not respond to initial treatment or in those being considered for immunotherapy.

How well did you know this?
1
Not at all
2
3
4
5
Perfectly
91
Q

Elderly woman saying that sexual activity is no longer “pleasurable.” cause?

A

vaginal atrophy- vaginal dryness and dyspareunia.

*** if they describe interest then is not age-related libido

How well did you know this?
1
Not at all
2
3
4
5
Perfectly
92
Q

treatment of postmenopausa

A

SSRIs, NSRIs ( Venlafaxine)

How well did you know this?
1
Not at all
2
3
4
5
Perfectly
93
Q

Considerations before prescribing opioids

A

evaluate prescription history and use
- Also check the prescription drug monitoring program ( identify undisclosed prescriptiosn, clarify prescription patters and see if patient has obtained prescriptions from multiple providers)

How well did you know this?
1
Not at all
2
3
4
5
Perfectly
94
Q

Tto of seborrheic dermatitis

A

TOPICAL antifungals (eg, selenium sulfide SHAMPOO, ketoconazole)
Topical glucocorticoids
Topical calcineurin inhibitors (eg, pimecrolimus, tacrolimus)
keratolytic agents (eg, salicylic acid)

**Oral could be used but are NOT the first line

How well did you know this?
1
Not at all
2
3
4
5
Perfectly
95
Q

organism involved in seborrheic dermatitis

A

Malassezia

How well did you know this?
1
Not at all
2
3
4
5
Perfectly
96
Q

Griseofulvin is used in

A

tinea capitis

How well did you know this?
1
Not at all
2
3
4
5
Perfectly
97
Q

Permethrin is used to

A

scabies

How well did you know this?
1
Not at all
2
3
4
5
Perfectly
98
Q

Crusted scabies can cause large, scaly patches but is not pruritic and occurs most commonly in immunocompromised patients (eg, lymphoma, AIDS).

A

t

How well did you know this?
1
Not at all
2
3
4
5
Perfectly
99
Q

long term management of seborrheic dermatitis/ prognosis

A

chronic, relapsing condition.
Initial treatment -> significant improvement but no remission.

SO: intermittent re-treatment. Typical regimens include topical ketoconazole or ciclopirox every 1-2 weeks.

How well did you know this?
1
Not at all
2
3
4
5
Perfectly
100
Q

factorial design

A

STUDY arms in RCT

How well did you know this?
1
Not at all
2
3
4
5
Perfectly
101
Q

Does psoriasis cause alopecia?

A

NO, usually no!

How well did you know this?
1
Not at all
2
3
4
5
Perfectly
102
Q

well-demarcated, often round, non-scarred patch of complete hair loss

A

Alopecia areata: can be multiple patches! not only one.

“exclamation point hairs” (hairs which are tapered near the insertion into the scalp), especially at the periphery of an alopecic plaque

How well did you know this?
1
Not at all
2
3
4
5
Perfectly
103
Q

Nail pitting associated to alopecia areata

A

yes

How well did you know this?
1
Not at all
2
3
4
5
Perfectly
104
Q

Nail pitting is seen in

A

Psoriasis

Alopecia areata

How well did you know this?
1
Not at all
2
3
4
5
Perfectly
105
Q

prognosis of alopecia areata

A

self-limited but may be relapsing and remitting or chronic and progressive.

increased risk -> autoimmune thyroid disease, vitiligo, and pernicious anemia.

How well did you know this?
1
Not at all
2
3
4
5
Perfectly
106
Q

Tinea capitis vs alopecia areata

A

tinea capitis: erythema, scaling, “black dot” alopecia (secondary to breakage of hairs near the scalp), pustules, or boggy plaques with posterior

alopecia areata: well emarcated, non-scarred, exclamation points around it.

How well did you know this?
1
Not at all
2
3
4
5
Perfectly
107
Q

Tto of normal pressure hydrocephalus

A

Ventriculoperitoneal shunting

***no eviudence for acetazolamide

How well did you know this?
1
Not at all
2
3
4
5
Perfectly
108
Q

Diagnosis of normal pressure hydrocephalus

A

Marked improvement in gait with spinal fluid removal: Miller Fisher (lumbar tap) test
Enlarged ventricles out of proportion to the underlying brain atrophy on MRI

How well did you know this?
1
Not at all
2
3
4
5
Perfectly
109
Q

ppt normal pressure hydrocephalus

A
Gait instability (wide-based) with frequent falls
Cognitive dysfunction
Urinary urgency/incontinence
Depressed affect (frontal lobe compression)
Upper motor neuron signs in lower extremities ( they can have tremor, rigidity-parecido a parkinson)
How well did you know this?
1
Not at all
2
3
4
5
Perfectly
110
Q

lumbar tap test (Miller Fisher test)

A

Is for normal pressure hydrocephalus

evaluates parameters such as gait speed, stride length, verbal memory, and visual attention before and after removal of 30-50 mL of CSF.

How well did you know this?
1
Not at all
2
3
4
5
Perfectly
111
Q

red flags for a pathologic etiology in scoliosis

A

Back pain ( nocturnal awakening- concerning for tumor)

Neurologic symptoms

Rapidly progressing curvature (>10 degrees each year)

Vertebral anomalies on x-ray

** degree of curvature is not associated with etiology

How well did you know this?
1
Not at all
2
3
4
5
Perfectly
112
Q

suspect scoliosis, next step

A

X ray, although MRI is gold standard

How well did you know this?
1
Not at all
2
3
4
5
Perfectly
113
Q

Anterior horn cells disorders

A

SMA
ALS
Paraneoplastic syndromes
Polio

How well did you know this?
1
Not at all
2
3
4
5
Perfectly
114
Q

UMN disorders

A

leukodystrophies
vasculitis
brain mass
vit B12

How well did you know this?
1
Not at all
2
3
4
5
Perfectly
115
Q

Pathogenesis of botulism

A

peripheral nerve endings to inhibit release of acetylcholine into the synaptic cleft.

How well did you know this?
1
Not at all
2
3
4
5
Perfectly
116
Q

patient with shock that after resuscitation develops hyperactive reflexes

A

multiple transfusions can lead to hypocalcemia as Ca joins with citrate .
—-citrate chelates with calcium
hypocalcemia : hyperactive reflexes

How well did you know this?
1
Not at all
2
3
4
5
Perfectly
117
Q

relation between hypomagnesemia and reflexes

A

hyporeflexia

How well did you know this?
1
Not at all
2
3
4
5
Perfectly
118
Q

Patient in whom you suspect varicella zoster, next step

A

Treat with oral acyclovir/valacyclovir

atients with localized herpes zoster who have had lesions <72 hours typically receive antiviral treatment with oral valacyclovir to reduce transmission risk, new lesion formation, and (possibly) risk of postherpetic neuralgia. Analgesics for acute neuritis are also typically required.

No need for testing. testing is reserved for atypical lesions

How well did you know this?
1
Not at all
2
3
4
5
Perfectly
119
Q

Healthcare precautions for patients with varicella zoster

A

Localized infection - standard precautions and lesion covering
Disseminated infection - standard precautions plus contact and airborne precautions

How well did you know this?
1
Not at all
2
3
4
5
Perfectly
120
Q

Definition of hypoglycemia

A

< 60

normal fasting blood sugar range is 70 to 100 mg/dL

The target pre-meal blood sugar level in patients with diabetes is between 80 and 120 mg/dL.

How well did you know this?
1
Not at all
2
3
4
5
Perfectly
121
Q

Whipple’s triad

A

low blood glucose level
symptoms of hypoglycemia
symptomatic relief with the administration of glucose.

Whipple’s triad is strongly suggestive of true hypoglycemia.

How well did you know this?
1
Not at all
2
3
4
5
Perfectly
122
Q

Hypoglycemic episodes during exercise

A

decreasing the insulin dose (NPH, glargine - not the regular that ggoes with each meal ), eating before exercising, and avoiding injections of insulin in the exercising limbs.

How well did you know this?
1
Not at all
2
3
4
5
Perfectly
123
Q

unilateral facial nerve palsy, hepatomegaly (>2 cm below costal margin), and lymphadenopathy AND significant fatigue

A

extrapulmonary sarcoidosis

How well did you know this?
1
Not at all
2
3
4
5
Perfectly
124
Q

suspect sarcoidosis, next step

A

a chest x-ray as >90% of patients have bilateral hilar or mediastinal adenopathy

How well did you know this?
1
Not at all
2
3
4
5
Perfectly
125
Q

Löfgren syndrome

A

SARCOIDOSIS

Erythema nodosum
Hilar adenopathy
Migratory polyarthralgia
Fever

How well did you know this?
1
Not at all
2
3
4
5
Perfectly
126
Q

Eye manifestations in sarcoidosis

A

Anterior uveitis (iridocyclitis or iritis)
Posterior uveitis
Keratoconjunctivitis sicca

How well did you know this?
1
Not at all
2
3
4
5
Perfectly
127
Q

CNS/endocrine manifestations

A

Facial nerve palsy
Central diabetes insipidus
Hypercalcemia

How well did you know this?
1
Not at all
2
3
4
5
Perfectly
128
Q

Herpes simplex meningitis presentation

A

can have focal signs, including nerve palsy but in 90% cases ALTERED MENTAL STATUS

How well did you know this?
1
Not at all
2
3
4
5
Perfectly
129
Q

Patients with facial nerve palsy should undergo brain imaging if symptoms do not improve by 3 weeks or resolve by 4 months.

A

T

How well did you know this?
1
Not at all
2
3
4
5
Perfectly
130
Q

Chest x-ray reveals bilateral hilar adenopathy and interstitial infiltrates, next step to confirm diagnosis

A

excisional lymph node biopsy- noncaseous granulomas

usually a palpable lymph node or skin lesion. When there is no easily accessible lesion, fiberoptic bronchoscopy with transbronchial lung biopsy is often performed.

*** Although ACE levels will be high in 75%, not recommended.

How well did you know this?
1
Not at all
2
3
4
5
Perfectly
131
Q

All patients with pulmonary sarcoidosis should undergo pulmonary function testing to evaluate for respiratory impairment

A

true, but this is not to confirm diagnosis.

Diagnosis after chest X ray: excisional lymph node

How well did you know this?
1
Not at all
2
3
4
5
Perfectly
132
Q

Management of bacterial conjuncitvitis

A

S.aureus, S. pneumonia, moraxella, H. influenza

Erythromycin ointment
Polymyxin-trimethoprim drops
Azithromycin drops
Preferred agent in contact lens wearers: fluoroquinolone drops (pseudomona)

How well did you know this?
1
Not at all
2
3
4
5
Perfectly
133
Q

Management of viral conjunctivitis

A

Warm or cold compresses

± Antihistamine/decongestant drops

How well did you know this?
1
Not at all
2
3
4
5
Perfectly
134
Q

When keratitis is suspected, next step

A

ophthalmology referral

diagnosis of keratitis is made by slit-lamp examination showing corneal ulceration.

cultures are needed

Keratitis can cause scarring or ulceration of the cornea and subsequent blindness if not aggressively treated.

How well did you know this?
1
Not at all
2
3
4
5
Perfectly
135
Q

well-defined plaques covered by thick silvery scales

A

psoriasis

How well did you know this?
1
Not at all
2
3
4
5
Perfectly
136
Q

Treatment of psoriasis

A

Mild-to moderate plaques: TOPICAL high potency steroids ( fluocinonide, augmented betamethasone dipropionate 0.05%)
-topical vit D derivatives

Severe plaques: phtototherapy or systemic agents(methotrexate)

How well did you know this?
1
Not at all
2
3
4
5
Perfectly
137
Q

Pt who had runny nose, sore throat, tactile, then develops headache, vomiting confusion and focal neuro signs?

A

Viral meningoencephalitis

How well did you know this?
1
Not at all
2
3
4
5
Perfectly
138
Q

Suspect viral meningoencephalitis

A

LP followed by vancomycin, ceftriaxone, and ACYCLOVIR

How well did you know this?
1
Not at all
2
3
4
5
Perfectly
139
Q

MC causes of meningoencephalitis

A

enterovirus( coxsackie)
herpes
arbovirus(West Nile)

How well did you know this?
1
Not at all
2
3
4
5
Perfectly
140
Q

role of dexamethasone in pneumococcal meningitis.

A

Prevents neurological complications

How well did you know this?
1
Not at all
2
3
4
5
Perfectly
141
Q

loss of consciousness. He was standing in a crowded subway station when he felt lightheaded, had a pounding sensation in his chest, and passed out. On awakening, he felt short of breath for a little while and then was “completely fine.” Causes?

A

Vasovagal syncope could be
BUT CONSIDER
-Tachyarrhythmia- WPW so look for it in the ECG

How well did you know this?
1
Not at all
2
3
4
5
Perfectly
142
Q

Managemetn of WPW

A

Catheter ablation

How well did you know this?
1
Not at all
2
3
4
5
Perfectly
143
Q

Altered mental status in patients with lithium, what to consider

A

Any new meds? volume depletion? CKD?

Lithium has a very narrow therapeutic index and can interact with many drugs

THIAZIDES ( chlortalidone) , ACEIs, NSAIDs increase levels of lithium.

How well did you know this?
1
Not at all
2
3
4
5
Perfectly
144
Q

therapeutic and toxic levels of lithium

A

Therapeutic lithium levels are 0.8-1.2 mEq/L.

Serum lithium levels >1.5 mEq/L confirm toxicity, and levels >2.5 mEq/L require emergency management.

How well did you know this?
1
Not at all
2
3
4
5
Perfectly
145
Q

Drugs that increase lithium levels

A

THIAZIDES ( chlortalidone) , ACEIs, NSAIDs

How well did you know this?
1
Not at all
2
3
4
5
Perfectly
146
Q

Management of lithium toxicity

A

Lithium levels every 2-4 hours
Intravenous hydration
Bowel irrigation (asymptomatic acute overdose)
Hemodialysis:
>4 mEq/L
>2.5 mEq/L with symptoms or renal failure
Increasing level despite intravenous fluids

How well did you know this?
1
Not at all
2
3
4
5
Perfectly
147
Q

presentation of lithium toxicity

A

Acute

GI: Nausea, vomiting, diarrhea
Neurologic findings occur much later
Chronic

Neurologic: Ataxia, confusion, agitation, neuromuscular excitability (tremor)

How well did you know this?
1
Not at all
2
3
4
5
Perfectly
148
Q

Mammogram screening

A

Mammogram every 2 years for women age 50-74

Screening for patients age 40-49 is based on individual risk factors

Breast cancer screening is not indicated in low-risk patients age <40.

How well did you know this?
1
Not at all
2
3
4
5
Perfectly
149
Q

self manual exam is recommended annuallu

A

FALSE, Breast self-examination does not reduce breast cancer mortality. It has a high false-positive rate and leads to unnecessary biopsies; therefore, it is not recommended at any age.

How well did you know this?
1
Not at all
2
3
4
5
Perfectly
150
Q

Management of acute diverticulitis

A

bowel rest

ciprofloxacin and metronidazole,

How well did you know this?
1
Not at all
2
3
4
5
Perfectly
151
Q

Patient with diverticulitis on antibiotics who after 2-3 days persists with fever, abodminal pain, next step

A

if no improvement after 2-3 days repeat CT to evaluate for complications: colonic abscess, obstruction, and perforation.

How well did you know this?
1
Not at all
2
3
4
5
Perfectly
152
Q

MC complication of diverticulitis

A

Colonic abscess (5-15%)

require percutaneous drainage and intravenous antibiotics followed by elective partial colectomy several weeks later.

How well did you know this?
1
Not at all
2
3
4
5
Perfectly
153
Q

When do you recommend colonoscopy in diverticulitis

A

patients with diverticulitis 6-8 weeks after the complete resolution of symptoms to rule out colon cancer.

How well did you know this?
1
Not at all
2
3
4
5
Perfectly
154
Q

pithelial cells that have a stippled appearance but no ferning

A

Vacterial vaginosis

How well did you know this?
1
Not at all
2
3
4
5
Perfectly
155
Q

Bacterial vaginosis

A

Clindamycin or MTZ

How well did you know this?
1
Not at all
2
3
4
5
Perfectly
156
Q

Candida vaginosis presentation

A

thick, white, clumpy discharge and vulvar pruritus

How well did you know this?
1
Not at all
2
3
4
5
Perfectly
157
Q

Why should be treat bacterial vaginosis in pregnancy

A

To relieve symptoms.

↑ Risk of preterm birth, BUT NOT IUGR

How well did you know this?
1
Not at all
2
3
4
5
Perfectly
158
Q

lung cancer screening

A

55-80

Patient has ≥30-pack-year smoking history
AND

Patient is a current smoker or quit smoking within the last 15 years

How well did you know this?
1
Not at all
2
3
4
5
Perfectly
159
Q

lung Ca screening termination

A

Age >80
OR

Patient successfully quit smoking for ≥15 years
OR

Patient has other medical conditions that significantly limit life expectancy or ability/willingness to undergo lung cancer surgery

How well did you know this?
1
Not at all
2
3
4
5
Perfectly
160
Q

Smoking cessation reduces the risk of developing lung cancer and COPD exacerbations, even in long-term heavy smokers.

A

true

How well did you know this?
1
Not at all
2
3
4
5
Perfectly
161
Q

What organism is most commonly cultured from corneal foreign bodies in the eyes?

A

Staph aureus

Other: Strep, hemophilus, pseudomona

How well did you know this?
1
Not at all
2
3
4
5
Perfectly
162
Q

treatment for all foreign-body associated corneal abrasions

A

empiric broad-spectrum antibiotic eye drops or ointments (e.g., erythromycin, sulfacetamide, ciprofloxacin, ofloxacin).

How well did you know this?
1
Not at all
2
3
4
5
Perfectly
163
Q

Pt comes for blood in urine, next step

A

hx of trauma or suspected stone: CT/US

IF no: urianalysis

even if you think is bladder cancer or you know the etiology need to confirm first

How well did you know this?
1
Not at all
2
3
4
5
Perfectly
164
Q

The first step in evaluating hematuria is

A

urinalysis and culture

How well did you know this?
1
Not at all
2
3
4
5
Perfectly
165
Q

acute onset of flushing, a throbbing headache, palpitations and abdominal cramps, after 30 min eating fish. Dx?

A

Scombroid poisoning

can also have bitter taste
skin erythema, wheezing, tachycardia and hypotension.

How well did you know this?
1
Not at all
2
3
4
5
Perfectly
166
Q

Pufferfish poisoning

A

prominence of neurological symptoms (perioral tingling, incoordination, weakness, etc).

How well did you know this?
1
Not at all
2
3
4
5
Perfectly
167
Q

Scombroid poisoning

A

acute onset of flushing, a throbbing headache, palpitations and abdominal cramps after 30 minutes of eating fish

can also have bitter taste
skin erythema, wheezing, tachycardia and hypotension.

How well did you know this?
1
Not at all
2
3
4
5
Perfectly
168
Q

Carcinoid syndrome

A

characterized by diarrhea, flushing, dyspnea, and wheezing – may occur if a serotonin-producing tumor has metastasized to the liver, bypassing first-pass metabolism. Elevated 5-HIAA in the urine helps to establish the diagnosis

How well did you know this?
1
Not at all
2
3
4
5
Perfectly
169
Q

Recommendations on exercise for pt with cerclage

A

Contraindicated

How well did you know this?
1
Not at all
2
3
4
5
Perfectly
170
Q

Contraindications of exercise during pregnancy

A

Patients at risk for preterm delivery
Cervical insufficiency
Preterm labor during current pregnancy
Preterm premature rupture of membranes

Patients at risk for antepartum bleeding
Placenta previa
Persistent second- or third-trimester bleeding

Patients with an underlying condition that could be exacerbated by exercise
Severe anemia
Hypertensive disorders of pregnancy (eg,
preeclampsia)
Restrictive lung disease
Severe heart disease

How well did you know this?
1
Not at all
2
3
4
5
Perfectly
171
Q

prognosis of night terrors

A

typically occurs between ages 4-12. The condition is usually benign and self-limiting, resolving spontaneously over 1-2 years.

low-dose BZD at bedtime if episodes are frequent, persistent & distressing

How well did you know this?
1
Not at all
2
3
4
5
Perfectly
172
Q

non-rapid eye movement (NREM) sleep disorders

A

night terrors, sleep walking ( do not remember)

How well did you know this?
1
Not at all
2
3
4
5
Perfectly
173
Q

REM sleep-related behavior disorder

A

acts out the dreams

How well did you know this?
1
Not at all
2
3
4
5
Perfectly
174
Q

MOA of SGLT2 inhibitors for DM

A

proximal tube, exchange sodium for glucose. So inhibit re absorption glucose

canagliflozin, dapagliflozin, and empagliflozin

How well did you know this?
1
Not at all
2
3
4
5
Perfectly
175
Q

SE SGLT2 inhibitors for DM

A

Vulvovaginal candidiasis
Polyuria
UTI
* In the setting of glucosuria

canagliflozin, dapagliflozin, and empagliflozin

How well did you know this?
1
Not at all
2
3
4
5
Perfectly
176
Q

canagliflozin, dapagliflozin, and empagliflozin are what type of drugs

A

SGLT2 inhibitors for DM

How well did you know this?
1
Not at all
2
3
4
5
Perfectly
177
Q

Dipeptidyl-peptidase 4 (DPP4) inhibitors

A

sitagliptin, saxagliptin, linagliptin, and alogliptin.

increase incretin levels (GLP-1 and GIP)–> inhibit glucagon release, which in turn increases insulin secretion, decreases gastric emptying, and decreases blood glucose levels.

How well did you know this?
1
Not at all
2
3
4
5
Perfectly
178
Q

DDP4 inhibitors SE

A

Pancretatitis, may cause hypersensitivity ( ie. angioedema)

How well did you know this?
1
Not at all
2
3
4
5
Perfectly
179
Q

Schizoaffective disorder

A

Major depressive or manic episode concurrent with symptoms of schizophrenia

Lifetime history of delusions or hallucinations for ≥2 weeks in the absence of major depressive or manic episode

Mood episodes are prominent & recur throughout illness

How well did you know this?
1
Not at all
2
3
4
5
Perfectly
180
Q

Schizoaffective disorder vs Major depressive or bipolar disorder with psychotic features:

A

Schizoaffective disorder: has concurrent mood symptoms with psychotic symptoms, PLUS ≥2 weeks of psychotic symptoms in the absence of a major depressive or manic episode.

In the other disorder: Psychotic symptoms occur exclusively during mood episodes

How well did you know this?
1
Not at all
2
3
4
5
Perfectly
181
Q

Schizoaffective disorder is distinguished from schizophrenia by

A

the presence of mood symptoms lasting a significant portion of the illness.

How well did you know this?
1
Not at all
2
3
4
5
Perfectly
182
Q

treating friends and family is not always unethical

A

true, there may be emergency cases in which is needed

How well did you know this?
1
Not at all
2
3
4
5
Perfectly
183
Q

Friend asking for a prescription of opioid, what to say

A

” I would like to help you but I feel uncomfortable prescribing to someone I do not treat

How well did you know this?
1
Not at all
2
3
4
5
Perfectly
184
Q

Women in 50s with symptoms of amenorrhea, and increased vaginal bleeding, next step?

A

endometrial biopsy

her bleeding is anovulatory, and there is risk of endometrial cancer due to unopposed estrogen production.

ALL WOMEN>=45 with anovulatory bleeding need to have endometrial biopsy

How well did you know this?
1
Not at all
2
3
4
5
Perfectly
185
Q

Indications for endometrial biopsy

A
  • ALL WOMEN>=45 with anovulatory bleeding

- Women < 45 with RF ( POS, Obesity), failed management, persist with abnormal bleeding.

How well did you know this?
1
Not at all
2
3
4
5
Perfectly
186
Q

Why does menopause occur? At what age? How is diagnosis made?

A
  • loss of ovarian function (oocyte depletion, and abnormal follicular development)
  • oocytes fail to produce progesterone

Usually by 51

Dx with >=1 year of amenorrhea

How well did you know this?
1
Not at all
2
3
4
5
Perfectly
187
Q

Treatment of anovulatory bleeding in transition to menopause

A
First - do an endometrial biopsy!
If normal: Other options: 
Cyclic progestin
Low dose OCP 
DIU
How well did you know this?
1
Not at all
2
3
4
5
Perfectly
188
Q

Is measure of FSH needed for diagnosis of menopause?

A

NO! Menopause is a clinical diagnosis and FSH is not required to confirm.

How well did you know this?
1
Not at all
2
3
4
5
Perfectly
189
Q

MCC of viral meningoencephalitis

A

Enterovirus (Coxsackie)
Herpes (HSV)
Arbovirus( West Nile Virus)

How well did you know this?
1
Not at all
2
3
4
5
Perfectly
190
Q

presentation of viral meningoencephalitis

A

fever, headache, altered mental status, nuchal rigidity.

How well did you know this?
1
Not at all
2
3
4
5
Perfectly
191
Q

West Nile infection - time of the year and presentation

A

Summer and early fall

meningitis, including fever, headache, photophobia, nuchal rigidity, and/or signs of encephalitis, such as confusion and focal neurologic findings (eg, hyperreflexia).

  • Rash can be present but not always
    patients may have excoriated mosquito bites,
How well did you know this?
1
Not at all
2
3
4
5
Perfectly
192
Q

Dx of West Nile virus

A

West Nile IgM antibody in the cerebrospinal fluid

How well did you know this?
1
Not at all
2
3
4
5
Perfectly
193
Q

Does Lyme cause encephalitis?

A

NO, most commonly meningitis, CN palsy, or peripheral neuropathy.

How well did you know this?
1
Not at all
2
3
4
5
Perfectly
194
Q

Presentation of Duchenne

A

2-3 years old, X-linked myopathy
developmental delay ( can have speech delays, late onset walking)
proximal lower extremities weakness (unable to run, walk, jump) for a long time
Dilated Cardiomyopathy
Scoliosis
Waddling gait
pseudohypertrophy of calves – as fat replaces degrading muscle.
THEY CAN HAVE HYPOREFLEXIA of achilles tendon

How well did you know this?
1
Not at all
2
3
4
5
Perfectly
195
Q

First step when suspecting Duchenne’s myopathy

A

CK levels

How well did you know this?
1
Not at all
2
3
4
5
Perfectly
196
Q

GOLD standard or CONFIRMATORY study for Duchenne?

A

Genetic testing showing deletion of one or more exons of the dystrophin gene

Not muscle biopsy

How well did you know this?
1
Not at all
2
3
4
5
Perfectly
197
Q

Most accurate diagnostic tool for Parkinson disease?

A

Clinical- physical examination

Striatal dopamine transporter scan ( DATscan) can be considered when the diagnosis of Parkinson’s disease is equivocal.

How well did you know this?
1
Not at all
2
3
4
5
Perfectly
198
Q

Management of Parkinson

A

Initial treatment:
< 65 : pramipexole, bromocriptine ( dopamine agonists)
>65 : levodopa

Levodopa concerns with prolonged use. It hastens destruction of substantia nigra cells and worsen symptoms in the long-term.

How well did you know this?
1
Not at all
2
3
4
5
Perfectly
199
Q

Role of entacapone in parkinson

A

Entacapone by itself not helpful for PD. It helps prolong the effect of levodopa.

How well did you know this?
1
Not at all
2
3
4
5
Perfectly
200
Q

Patients with untreated celiac disease are at high risk of

A

enteropathy-associated T-cell lymphoma (EATL) in the jejunum – poor prognosis - 10 months.

How well did you know this?
1
Not at all
2
3
4
5
Perfectly
201
Q

Foodborne illness- primarily emesis

A

S. aureus, B cereus

Norovirus (Norwalk)

How well did you know this?
1
Not at all
2
3
4
5
Perfectly
202
Q

Foodborne illness-predominant watery diarrhea

A
C. perfringes
Enterotoxin E.coli
Enteric viruses
Cryptosporidium
Cyclospora
Intestinal  tapeworms
How well did you know this?
1
Not at all
2
3
4
5
Perfectly
203
Q

Foodborne illness-predominant inflammatory diarrhea ( bloody)

A
  • Shigella
  • Salmonella
  • Shiga toxin E.coli
  • Campylobacter
  • Enterobacter
  • Vibrio parahemolyticus
  • Yersinia
How well did you know this?
1
Not at all
2
3
4
5
Perfectly
204
Q

Foodborne illness- Ciguatera toxin

A

paresthesia

How well did you know this?
1
Not at all
2
3
4
5
Perfectly
205
Q

Foodborne illness- Scombroid

A

Flushing, urticaria

How well did you know this?
1
Not at all
2
3
4
5
Perfectly
206
Q

Foodborne illness- Listeria

A

Meningitis

How well did you know this?
1
Not at all
2
3
4
5
Perfectly
207
Q

Foodborne illness- Vibrio Vulnificus

A

Cellulitis, sepsis

How well did you know this?
1
Not at all
2
3
4
5
Perfectly
208
Q

Foodborne illness- Brucellosis

A

Arthtralgias, fever

How well did you know this?
1
Not at all
2
3
4
5
Perfectly
209
Q

Norovirus ( 3 types of presentation)

A
  • Asymptomatic infection
  • Fever with watery diarrhea - non-inflammatory small-bowel process
  • Severe illness - fever, vomiting, headache, and other systemic symptoms

Symptoms usually start suddenly and last 48-72 hours. Diagnosis is generally clinical, and most patients improve with supportive care. If needed, diagnosis can be confirmed by polymerase chain reaction or nucleic acid-based testing.

How well did you know this?
1
Not at all
2
3
4
5
Perfectly
210
Q

Dx of campylobacter

A

Stool culture

How well did you know this?
1
Not at all
2
3
4
5
Perfectly
211
Q

Pathogen of Traveler’s diarrhea

A

Enterotoxigenic E coli

How well did you know this?
1
Not at all
2
3
4
5
Perfectly
212
Q

The most common cause of gastroenteritis in children and adults

A

Norovirus

How well did you know this?
1
Not at all
2
3
4
5
Perfectly
213
Q

Tto of asymptomatic bacteriuria or cystitis in pregnancy

A

Nitrofurantoin, Cephalexin, amoxi-clavulanate, or fosfomycin

3-7 days

How well did you know this?
1
Not at all
2
3
4
5
Perfectly
214
Q

Are fluoroquinolones CI in pregnancy

A

Yes, risk of cartilage damage in the fetus

How well did you know this?
1
Not at all
2
3
4
5
Perfectly
215
Q

First line of treatment for cystitis in non-pregnat

A

TMP-SMX

In pregnant: Cephalexin, amoxi-clavulanate, or fosfomycin

How well did you know this?
1
Not at all
2
3
4
5
Perfectly
216
Q

Why is Trimethoprim-sulfamethoxazole CI in pregnancy

A

irst trimester has been associated with neural tube defects due to the folate antagonist

Late third trimester associated with kernickterus

How well did you know this?
1
Not at all
2
3
4
5
Perfectly
217
Q

Management of pyelonephritis during pregnancy

A

Ceftriaxone, Cefepime IV
Once afebrile can be transitioned to 10-14 days of oral antibiotics.

After treatment completion- daily suppressive therapy (eg, low-dose nitrofurantoin or cephalexin) is initiated and is maintained until 6 weeks postpartum to prevent recurrence.

How well did you know this?
1
Not at all
2
3
4
5
Perfectly
218
Q

Outpatient management of pyelonephritis is not recommended in pregnancy due to risk of complications ie. pulmonary edema

A

true

How well did you know this?
1
Not at all
2
3
4
5
Perfectly
219
Q

Cholestatic pattern in labs

A

high conjugated bilirubin, high cholesterol, high ALP.

How well did you know this?
1
Not at all
2
3
4
5
Perfectly
220
Q

Algorithm for high ALP

A

Check GGT,
If GGT Normal- then ALP is likely bone origin
If GGT elevated- liver
If liver– RUQ US and AMA!

How well did you know this?
1
Not at all
2
3
4
5
Perfectly
221
Q

Suspect of primary sclerosing cholangitis, next step

A

ERCP intra and extrahepatic sclerosis- strictures alternating with dilation and beading.

How well did you know this?
1
Not at all
2
3
4
5
Perfectly
222
Q

Primary biliary cholangitis/cirrhosis with AMA + or US abnormal , next step

A

Liver biopsy

How well did you know this?
1
Not at all
2
3
4
5
Perfectly
223
Q

Complications of Primary biliary cirrhosis/cholangitis

A

Osteomalacia ( despite vit D levels normal), malabsorption, ADEK deficiencies

How well did you know this?
1
Not at all
2
3
4
5
Perfectly
224
Q

PPT of Primary biliary cirrhosis/cholangitis

A
Fatigue &amp; pruritus (most common)
Inflammatory arthritis (40%-70%)
Hyperpigmented skin (25%-50%)
Right upper quadrant discomfort (10%)
Xanthelasmata (10%) &amp; xanthomata (5%)
How well did you know this?
1
Not at all
2
3
4
5
Perfectly
225
Q

Alpha-1 antitrypsin deficiency

A

cirrhosis (with significant elevations in hepatic transaminases) and emphysema.

How well did you know this?
1
Not at all
2
3
4
5
Perfectly
226
Q

Anti–smooth-muscle antibodies

A

type 1 autoimmune hepatitis, which typically causes elevation of hepatic transaminases. Alkaline phosphatase is usually normal or only mildly elevated.

How well did you know this?
1
Not at all
2
3
4
5
Perfectly
227
Q

ppt hemochromatosis

A

elevations in transaminases, and often has additional findings including hyperpigmentation, diabetes mellitus, arthropathy, and cardiomyopathy.

How well did you know this?
1
Not at all
2
3
4
5
Perfectly
228
Q

Treatment of primary biliary cholangitis

A

ursodeoxycholic acid , if not liver transplant

Ca and Vit D supplementation

How well did you know this?
1
Not at all
2
3
4
5
Perfectly
229
Q

Management of asthma exacerbation

A

Mild to moderate ( PEF or FEV >=40): SABA ( up to 3 doses in 1 hour), PO corticosteroids if no reposnse to SABA or prior response to Corticosteroids. O2 if Sat <90

Moderate to Severe ( PEF or FEV<40): SABA +ipratropium in 1 hour , PIV or pO corticosteroids, O2 for Sat<90

Impending or actual respiratory distress
- SABA +Ipratropium
-IV corticosteroids + Mg
- +/- SQ terbutaline or epi
\+/- intubation 
and admission to PICU
How well did you know this?
1
Not at all
2
3
4
5
Perfectly
230
Q

Inhaled short-acting beta agonists are the first-line treatment IN ASTHMA regardless of severity.

A

TRUE

How well did you know this?
1
Not at all
2
3
4
5
Perfectly
231
Q

Treatment of CML

A

tYROSINE KINASE INHIBITOR– IMATINIB

How well did you know this?
1
Not at all
2
3
4
5
Perfectly
232
Q

Management of chronic urticaria

A

nothing! unless systemic ss

80-90% of urticaria is idiopathic

How well did you know this?
1
Not at all
2
3
4
5
Perfectly
233
Q

Presentation of chronic urticaria

A

Episodes of urticaria > 6 weeks
episodes of rash that can worsen over min to hours, and resolve really fast
INTENSE PRURITUS that can extend throughout the night.
40% may associate with angioedema ( mast cell activation extends to subcutuaneous tissue and deeper layers)

How well did you know this?
1
Not at all
2
3
4
5
Perfectly
234
Q

Causes of chronic urticaria

A
80-90% idiopathic
physical stimuli - cold, skin pressure
NSAIDs
stress
systemic disorders ( vasculitis, autoimmunde diseases)
How well did you know this?
1
Not at all
2
3
4
5
Perfectly
235
Q

C1 esterase inhibitor levels

A

hereditary angioedema
angioedema of the throat, tongue, or lips without urticaria. Patients can also have abdominal pain due to angioedema of the intestinal mucosa.

How well did you know this?
1
Not at all
2
3
4
5
Perfectly
236
Q

When do you do biopsy in urticaria

A

NOT RECOMMENDED,

ONLY with lesions that last >24 hours, are painful, have associated petechiae or purpura, associated systemic symptoms (eg, fever), elevated erythrocyte sedimentation rate/C-reactive protein, or do not respond to treatment.

How well did you know this?
1
Not at all
2
3
4
5
Perfectly
237
Q

1st and 2 nd generation H1 blockers

A

diphenydramine , chlorphenidramine
and others for motion sickeness: mezicline hydroxycine, promethazine.
2nd generation :cetirizine, loratadine, desloratadine

How well did you know this?
1
Not at all
2
3
4
5
Perfectly
238
Q

Management of acute urticaria < 6 weeks

A

Mild: 1st or 2nd generation H1
Moderate h1 +h2 blockers
Sever: oral steroid

How well did you know this?
1
Not at all
2
3
4
5
Perfectly
239
Q

Management of chronic urticaria > 6 weeks

A

daily 2nd generation H1 blocker

If no improvement in 2 weeks consider increasing dosing, add a first generation, add a H2 blocker, anti-leukotriene, brief course of oral steroids.

If no improvement consider hydroxycloroquine, omalizumab or tacrolimus.

How well did you know this?
1
Not at all
2
3
4
5
Perfectly
240
Q

H2 BLOCKERS EXAMPLE

A

Ranitidine , cimetidine

How well did you know this?
1
Not at all
2
3
4
5
Perfectly
241
Q

prognosis of chronic urticaria

A

most patients have spontanoeus resolution within 1-5 years . 1 (50%), at 5 years 70%

How well did you know this?
1
Not at all
2
3
4
5
Perfectly
242
Q

Causes of acute pericarditis

A
Viral  or idiopathic 
TB
Uremia 
Autoimmune ( SLE)
Post MI ( early ( Peri infarct)< 4 days, late ( Dressler))
How well did you know this?
1
Not at all
2
3
4
5
Perfectly
243
Q

Management of peri infarct pericarditis vs. viral/idiopathic pericarditis

A

Peri-infarct pericarditis: high dose aspirin 650 mgTID
- small effect on myocardial healing

Pericarditis idiopathic or viral: NSAIDs and colchicine

NSAIDS to be avoided in peri-infarct as impair myocardial healing and risk of wall rupture .

How well did you know this?
1
Not at all
2
3
4
5
Perfectly
244
Q

Long term prognosis of pulmonary sarcoidosis

A

Asymptomatic patients often require no treatment

those with symptoms or pulmonary function impairment usually receive 12-24 months of oral glucocorticoids

Most cases (~75%) resolve over time and do not recur.

How well did you know this?
1
Not at all
2
3
4
5
Perfectly
245
Q

TB screening

A

Can be done with the tuberculin skin test or Interferon Gamma

Interferon Gamma preferred in patients who had BCG.

How well did you know this?
1
Not at all
2
3
4
5
Perfectly
246
Q

Patient tested positive for TB, asymptomatic and nothing on X ray

A

latent TB
6-9 months of isoniazid ( daily) OR
Rifampin 4 months

How well did you know this?
1
Not at all
2
3
4
5
Perfectly
247
Q

PFTs in asthma

A
In patients with active ss: 
obstructive pattern (reduced FEV1 and a reduced FEV1/FVC ratio (total lung capacity and diffusing capacity of the lungs for carbon monoxide [DLCO] are typically normal or sometimes elevated).  In these patients, a bronchodilator (eg, albuterol) can be administered and should result in significant improvement in FEV1 (eg, >15% from baseline).

In pts without ss- normal PFT findings. The administration of methacholine in these patients is likely to cause >20% reduction in FEV1,

( so normal test but an obstructive pattern following methacholine challenge)!

How well did you know this?
1
Not at all
2
3
4
5
Perfectly
248
Q

Mild, moderate, and severe lead toxicity . values

A

5-44

45-69

> =70

How well did you know this?
1
Not at all
2
3
4
5
Perfectly
249
Q

TTO lead poisoning

A

5-44- no meds

45-69 - Meso-2,3-dimercaptosuccinic acid (DMSA, succimer)

> =70Dimercaprol (British Anti-Lewisite)

plus

calcium disodium edetate (EDTA)

How well did you know this?
1
Not at all
2
3
4
5
Perfectly
250
Q

Indications of statins

A

Atherosclerotic cardiovascular disease ( CABG, TIA, etc)
LDL>=190
Age>-45 with DM
10 YEAR RISK OF CABG is >7.5-10%%

How well did you know this?
1
Not at all
2
3
4
5
Perfectly
251
Q

cholestyramine, colestipol, colesevelam - what class

A

bile acid sequestrants

How well did you know this?
1
Not at all
2
3
4
5
Perfectly
252
Q

5 Types of lipid lowering agents

A
  1. Statins
  2. Bile acid sequestrans ( cholestyramine, colestipol, colesevelam)
  3. Ezetimibe
  4. Fibrates( Gemfibrozil, fenofibrate)
  5. Niacin (B3)
How well did you know this?
1
Not at all
2
3
4
5
Perfectly
253
Q

MOA statins

A

Inhibit conversionof HMG-CoA to mevalonate, a precursor of cholesterol.
Decreases mortality in CAD patients

How well did you know this?
1
Not at all
2
3
4
5
Perfectly
254
Q

EA statins

A

Hepatoxicity (increases LFTs)

myopathy ( particularly when used with fibrates, or niacin)

How well did you know this?
1
Not at all
2
3
4
5
Perfectly
255
Q

MOA Bile acid sequestrants

A

Prevent intestinal reabsorption of bile acids, liver must use cholesterol to make more

How well did you know this?
1
Not at all
2
3
4
5
Perfectly
256
Q

EA Bile acid sequestrants

A

GI upset, decreased absorption of other drugs and fat soluble vitamins

How well did you know this?
1
Not at all
2
3
4
5
Perfectly
257
Q

MOA Ezetimibe

A

Prevent cholesterol absorption at small intestine brush border

How well did you know this?
1
Not at all
2
3
4
5
Perfectly
258
Q

EA Ezetimibe

A

Rarely increased LFTs, diarrhea

How well did you know this?
1
Not at all
2
3
4
5
Perfectly
259
Q

Fibrates MOA

A

Upregulate LPL–> TG clearance.

Active PPAR alpha to induce HDL synthesis

How well did you know this?
1
Not at all
2
3
4
5
Perfectly
260
Q

EA Fibrates

A

Mypathy ( if given with statins), gallbladder stones

How well did you know this?
1
Not at all
2
3
4
5
Perfectly
261
Q

Niacin MOA

A

Inhibits lipolysis in adipose tissue , reduces hepatic VLDL syndthesis

How well did you know this?
1
Not at all
2
3
4
5
Perfectly
262
Q

SE Niacin

A

Red, flushed face, which is decreased by NSAIDs or long term use

Hyperglycemia, Hyperuricemia

How well did you know this?
1
Not at all
2
3
4
5
Perfectly
263
Q

Which of the lipid lower agents decrease the most the TRG

A

Fibrates

How well did you know this?
1
Not at all
2
3
4
5
Perfectly
264
Q

Which of the lipid lower agents increased HDL

A

Niacin

How well did you know this?
1
Not at all
2
3
4
5
Perfectly
265
Q

1st degree block

A

constant PR prolonged.

How well did you know this?
1
Not at all
2
3
4
5
Perfectly
266
Q

2nd degree block MObitz I

A

PR prolonges before dropped beat

How well did you know this?
1
Not at all
2
3
4
5
Perfectly
267
Q

2nd degree block MObitz Ii

A

Constant PR dropped beat

How well did you know this?
1
Not at all
2
3
4
5
Perfectly
268
Q

Treatment of mobitz II and 3rd degree AV block

A

pacemaker

How well did you know this?
1
Not at all
2
3
4
5
Perfectly
269
Q

Suspect appendicitis in pregnancy next step

A

graded compression abdominal ultrasound

NO CT

How well did you know this?
1
Not at all
2
3
4
5
Perfectly
270
Q

Appendicitis in pregnancy

A

atypical presentation, including right-sided abdominal pain with no peritoneal signs or McBurney point tenderness.

abdominal pain, fever. high clinical suspicion

How well did you know this?
1
Not at all
2
3
4
5
Perfectly
271
Q

Pylephlebitis, what is

A

infective suppurative portal vein thrombosis (THROMBOSIS, INFECTED)

rare but devastating complication of untreated appendicitis or other intraabdominal or pelvic infections (eg, diverticulitis)

How well did you know this?
1
Not at all
2
3
4
5
Perfectly
272
Q

pyelephlebitis is a rare complication of appendicitis or intraabdominal pathology.

A

TRUE

How well did you know this?
1
Not at all
2
3
4
5
Perfectly
273
Q

negative predictive value

A

probability of not having a disease given a negative result on that test.

How well did you know this?
1
Not at all
2
3
4
5
Perfectly
274
Q

likelihood ratio (LR)

A

expression of sensitivity and specificity that can be used to assess the value of a diagnostic test indepedently of prevalence.

How well did you know this?
1
Not at all
2
3
4
5
Perfectly
275
Q

positive likelihood ratio (LR+)

A

represents the value of a positive test result. It is the probability of a patient with the disease testing positive divided by the probability of a patient without the disease testing positive.

How well did you know this?
1
Not at all
2
3
4
5
Perfectly
276
Q

lR value

A

The smaller the LR, the less likely it is that the disease is actually presen

How well did you know this?
1
Not at all
2
3
4
5
Perfectly
277
Q

negative likelihood ratio (LR-

A

he value of a negative test result.

How well did you know this?
1
Not at all
2
3
4
5
Perfectly
278
Q

Psychosis can be the first presentation of SLE

A

Psychosis due to SLE may be treated symptomatically with antipsychotics, but symptoms usually respond well to oral steroids within 2-4 weeks.

How well did you know this?
1
Not at all
2
3
4
5
Perfectly
279
Q

Metabolic causes of psychosis

A
Urea cycle disorders
Acute intermittent porphyria
Wilson disease
Renal/liver failure
Hypoglycemia
Sodium/calcium/magnesium disturbances
How well did you know this?
1
Not at all
2
3
4
5
Perfectly
280
Q

Systemic disorder causing psychosis

A

Systemic lupus erythematosus

Thyroiditis

How well did you know this?
1
Not at all
2
3
4
5
Perfectly
281
Q

Labs in SLE

A

leukopenia!!, anemia, and/or thrombocytopenia

How well did you know this?
1
Not at all
2
3
4
5
Perfectly
282
Q

Management of allergic rhinitis

A
Allergen avoidance is the first step!
Intranasal corticosteroids ( fluticasone spray)
How well did you know this?
1
Not at all
2
3
4
5
Perfectly
283
Q

Cat scratch disease

A

papule, nodule
fever < 50%
1-2 weeks tender and erythematous LAD

How well did you know this?
1
Not at all
2
3
4
5
Perfectly
284
Q

Treatment Cat scratch disease

A

Azithromycin

Mild cases in healthy patients often self-resolve within 1-4 months; but azithro reduces severity and lenght.

But if not very sure add clindamycin as empiric tto for s. aureus and strep

How well did you know this?
1
Not at all
2
3
4
5
Perfectly
285
Q

standard caloric intake recommended for enteral feeding

A

30 kcal/kg/day, with a lower amount of calories and higher protein can be used for patients with severe preexisting malnutrition in order to prevent refeeding syndrome.

A 1g/kg level of protein is appropriate for most patients.

How well did you know this?
1
Not at all
2
3
4
5
Perfectly
286
Q

Leriche syndrome

A

lower-extremity claudication, absent or diminished femoral pulses, and ED.

Erectile dysfunction- no erections in the morning

How well did you know this?
1
Not at all
2
3
4
5
Perfectly
287
Q

Ankle-brachial index

A

highest ankle SB pressure/highest arm SBP

=<0.9 abnormal
0.9-1.2- normal
>1.2-suggestive of calcified or uncompressible vessels.additional vascular studies needed

How well did you know this?
1
Not at all
2
3
4
5
Perfectly
288
Q

patients with testosterone deficiency can usually still achieve nocturnal penile erections.

A

true, and Testosterone deficiency can lead to a decrease in libido and erectile dysfunction

How well did you know this?
1
Not at all
2
3
4
5
Perfectly
289
Q

Patients with Erectile Dysfunction and atherosclerotic risk factors should receive appropriate diagnostic testing (eg, ankle-brachial index, cardiac stress testing) prior to initiating specific therapy for sexual dysfunction.

A

T

How well did you know this?
1
Not at all
2
3
4
5
Perfectly
290
Q

patient on chronic prednisone, advice for bone loss

A

Calcium and vit D

If there is no osteoporosis, no need for alendronate

How well did you know this?
1
Not at all
2
3
4
5
Perfectly
291
Q

How do corticoid cause bone loss

A
  1. Decrease Ca absorption in the gut
  2. Increased Renal Ca wasting
  3. Anti-anabolic effect on bone
  4. Decrease GnRH release– central hypogonadism.
How well did you know this?
1
Not at all
2
3
4
5
Perfectly
292
Q

stones, bones, groans, psychiatric overtones

A

“Stones” refers to kidney stones, nephrocalcinosis, and diabetes insipidus (polyuria and polydipsia). These can ultimately lead to renal failure.
“Bones” refers to bone-related complications. The classic bone disease in hyperparathyroidism is osteitis fibrosa cystica, which results in pain and sometimes pathological fractures. Other bone diseases associated with hyperparathyroidism are osteoporosis, osteomalacia, and arthritis.
“Abdominal groans” refers to gastrointestinal symptoms of constipation, indigestion, nausea and vomiting. Hypercalcemia can lead to peptic ulcers and acute pancreatitis. The peptic ulcers can be an effect of increased gastric acid secretion by hypercalcemia.[1]
“Thrones” refers to polyuria and constipation
“Psychiatric overtones” refers to effects on the central nervous system. Symptoms include lethargy, fatigue, depression, memory loss, psychosis, ataxia, delirium, and coma.
Left ventricular hypertrophy may also be seen.[2]

How well did you know this?
1
Not at all
2
3
4
5
Perfectly
293
Q

Causes of PTH independent hypercalcemia

A
Malignancy
High vit A, VIT D
Granulomatosis
Milk-alkali syndrome 
Excess intake Ca
immobilization
thiazides
thyrotoxicosis
How well did you know this?
1
Not at all
2
3
4
5
Perfectly
294
Q

Causes of priamry hyperPTH

A

high urine Ca

  • Adenoma, hyperplasia, cancer ( high ALP,cAMP)
  • Osteitis fibrosa cystica
  • familial hypocalciuric hyperCa
  • Lithium
How well did you know this?
1
Not at all
2
3
4
5
Perfectly
295
Q

Causes of secondary HYpert PTH

A
Renal failure ( high PO4)
Vit D deficiency (low  PO4)
pseudohypoPTH
Tumor lysis syndrome
pancreatitis
sepsis
How well did you know this?
1
Not at all
2
3
4
5
Perfectly
296
Q

Causes of 1ary hypoPTH

A

Surgical resection
Polyglandular autoimmune syndrome
Infiltrative disease ( Wilson, hemochromatosis, metastasis)
DiGeorge.

How well did you know this?
1
Not at all
2
3
4
5
Perfectly
297
Q

Corrected Ca

A

measured calcium + 0.8 x [4 – albumin]

How well did you know this?
1
Not at all
2
3
4
5
Perfectly
298
Q

Bisphosphonates are useful in decreasing bone resorption and may preserve bone mass in patients who are immobilized for extended periods.

A

t, can decrease bone turnover and preserve bone mass

How well did you know this?
1
Not at all
2
3
4
5
Perfectly
299
Q

Saw palmetto, what is it, and what is it used for

A

herbal preparation, benign prostatic hyperplasia. Its use has not been shown to significantly improve urinary symptoms or flow measures. In addition, saw palmetto does not appear to affect prostate-specific antigen levels or prostate size.

How well did you know this?
1
Not at all
2
3
4
5
Perfectly
300
Q

Kava kava, what is it, and what is it used for

A

anxiety and insomnia; however, its use is not recommended due to the potential risk of severe liver toxicity.

How well did you know this?
1
Not at all
2
3
4
5
Perfectly
301
Q

St. John’s wort ,what is it, and what is it used for

A

treatment of depression,insomnia,

SE:Drug interactions: Antidepressants (serotonin syndrome), OCs, anticoagulants (↓ INR), digoxin
Hypertensive crisis

How well did you know this?
1
Not at all
2
3
4
5
Perfectly
302
Q

SE of saw palmetto

A

bleeding

mild abdominal discomfort

How well did you know this?
1
Not at all
2
3
4
5
Perfectly
303
Q

Ginkgo biloba, what is it, and what is it used for , SE

A

memory enhancement, SE is bleeding

How well did you know this?
1
Not at all
2
3
4
5
Perfectly
304
Q

Ginseng,what is it, and what is it used for , SE

A

improved mental performance, SE is bleeding

How well did you know this?
1
Not at all
2
3
4
5
Perfectly
305
Q

Black cohosh,what is it, and what is it used for , SE

A

Post menopausal ss(hot flashes & vaginal dryness)

SE: Hepatic injury

How well did you know this?
1
Not at all
2
3
4
5
Perfectly
306
Q

Licorice,what is it, and what is it used for , SE

A

Stomach ulcers
Bronchitis/viral infections

SE: Hypertension
Hypokalemia

How well did you know this?
1
Not at all
2
3
4
5
Perfectly
307
Q

Echinaceawhat is it, and what is it used for , SE

A

Treatment & prevention of cold & flu

sE:
Anaphylaxis (more likely in asthmatics)

How well did you know this?
1
Not at all
2
3
4
5
Perfectly
308
Q

Ephedra

A

Treatment of cold & flu
Weight loss & improved athletic performance

Hypertension
Arrhythmia/MI/sudden death
Stroke
Seizure

How well did you know this?
1
Not at all
2
3
4
5
Perfectly
309
Q

Cells for diagnosis of Spontanoeus bacterial peritonitis

A

250 cells/mm3

How well did you know this?
1
Not at all
2
3
4
5
Perfectly
310
Q

Hepatorenal syndrome

A

Patients with cirrhosis develop decreased peripheral vascular resistance secondary to splanchnic vasodilation, which can cause the decreased renal perfusion of hepatorenal syndrome.

How well did you know this?
1
Not at all
2
3
4
5
Perfectly
311
Q

Suspect hepatorenal syndrome, next step?

A

A volume challenge would be an appropriate next step to confirm a diagnosis of hepatorenal syndrome in this patient. A failure to respond would be consistent with hepatorenal syndrome

octreotide and midodrine or norepinephrine if no response
albumin for 1-3 days.

How well did you know this?
1
Not at all
2
3
4
5
Perfectly
312
Q

test of choice for evaluation of tension pneumothorax in the acute setting (eg, trauma bay, intensive care unit).

A

Bedside US- allows visualization of the parietal and visceral pleura; inability to detect lung sliding, the 2 pleural layers moving against one another during respiration, is consistent with pneumothorax.

At PE will present with hyperresonance and no fremitus

if highly suspected–tto is needle decompression or chest tube placement.

How well did you know this?
1
Not at all
2
3
4
5
Perfectly
313
Q

What is the best screen question for unhealthy consumption of alcholo? - single item

A

how many times in the past year have you had 5 (4 for women) or more drinks/day?

How well did you know this?
1
Not at all
2
3
4
5
Perfectly
314
Q

CAGE

A

Have you felt you should cut down on your drinking?
Have others annoyed you by criticizing your drinking?
Have you ever felt bad or guilty about your drinking?
Have you ever taken a drink first thing in the morning (eye-opener) to steady your nerves?

How well did you know this?
1
Not at all
2
3
4
5
Perfectly
315
Q

Audit C screen alcohol

A

How often do you drink alcohol?
How many drinks do you have on a typical day when you are drinking?
How often do you have 6 (4 for women) or more drinks on 1 occasion?

How well did you know this?
1
Not at all
2
3
4
5
Perfectly
316
Q

DX sicca syndrome

A
secretory deficiency (e.g. Schirmer test) 
Anti Ro-, la, RF, ANA
How well did you know this?
1
Not at all
2
3
4
5
Perfectly
317
Q

tto of Slipped capital femoral epiphysis

A

surgical pinning

Delay in treatment (>24 hours in unstable SCFE) can lead to avascular necrosis and femoroacetabular impingement, which increase the risk of future degenerative arthritis.

How well did you know this?
1
Not at all
2
3
4
5
Perfectly
318
Q

Dx scoliosis

A

Posteroanterior and lateral x-rays

How well did you know this?
1
Not at all
2
3
4
5
Perfectly
319
Q

TTO scoliosis

A

treatment is generally unnecessary, and no follow-up is required for mild/moderate scoliosis (Cobb angle <40 degrees, as in this patient).

severe scoliosis (Cobb angle ≥40 degrees) : orthopedic referral for surgical evaluation – high risk of chronic pain, neuro ss, pulmonary complications

How well did you know this?
1
Not at all
2
3
4
5
Perfectly
320
Q

Palivizumab ( Sinagis ) for prevention of bronchiolitis indications:

A

Preterm birth <29 weeks gestation
Chronic lung disease of prematurity
Hemodynamically significant congenital heart disease

How well did you know this?
1
Not at all
2
3
4
5
Perfectly
321
Q

policies for physicians sponsored by pharmacy, lecturers vs. those who just attend to conferences.

A

Physicians attending conferences cannot accept subsidies from the industry for travel costs, lodging, or other personal expenses.

However, it is permissible for faculty physician lecturers to accept reasonable honoraria and/or reimbursement for reasonable travel expenses.
- They should full disclose conflicts of interest

How well did you know this?
1
Not at all
2
3
4
5
Perfectly
322
Q

Causes of serotonin syndrome

A
  • Serotonin drugs, especially in combination ( SSRIs, SNRIs, TCA, Tramadol)
  • Serotonin drugs in combination with MAOis ( phenelzine) or LINEZOLID
  • Drug overdose with Serotonin drugs
  • MDMA
How well did you know this?
1
Not at all
2
3
4
5
Perfectly
323
Q

presentation of serotonin syndrome

A

Altered mental status
Autonomic dysregulation( mydriasis, agitation, tachycardia, HTN, vomiting, diarrhea) can have fever
tremor, hyperreflexia, myoclonus, rigidity

How well did you know this?
1
Not at all
2
3
4
5
Perfectly
324
Q

difference between serotonin syndrome and NMS

A

SS: serotonin drug ( SSRIs, SNRIs, TCA, tramadol) + MAOs( phenelzine) or linezolid, or intoxication with SSRIs.
altered mental status, autonomic dysregulation, hyperrefelxia, tremor, rigidity

NMS: with antipsychotics ( haloperidol, risperidone). - dopamamine antagonists.
bradykinesia and generalized “lead pipe” muscular rigidity,. NO INCREASED IN REFLEXES.

How well did you know this?
1
Not at all
2
3
4
5
Perfectly
325
Q

Tto of serotonin syndrome

A

Discontinuation of all serotonergic medications
Supportive care, sedation with benzodiazepines
Serotonin antagonist (cyproheptadine) if supportive measures fail

How well did you know this?
1
Not at all
2
3
4
5
Perfectly
326
Q

to avoid serotonin syndrome how is the washout period to introduce MAOis in a patient who was previously on SSRIs

A

5 weeks

How well did you know this?
1
Not at all
2
3
4
5
Perfectly
327
Q

Dx of osteomyelitis

A

MRI with contrast of the bone

How well did you know this?
1
Not at all
2
3
4
5
Perfectly
328
Q

Diabetic ulcer with polymicrobial swab detected and osteomyelitis, next step in treatment

A

bone biopsy with culture and sensitivity to guide treatment

antibiotics > 6 weeks

How well did you know this?
1
Not at all
2
3
4
5
Perfectly
329
Q

TTO of Diabetic ulcer with osteomyelitis

A

glucose control,
surgical debridement, weight off-loading,
revascularization (if needed),
antibiotic therapy for >6 weeks.
Serial inflammatory markers (eg, erythrocyte sedimentation rate) can help confirm treatment response.

How well did you know this?
1
Not at all
2
3
4
5
Perfectly
330
Q

Blood cultures are often negative (>60%) in patients with osteomyelitis;

A

true

How well did you know this?
1
Not at all
2
3
4
5
Perfectly
331
Q

TTO of sporotrichosis

A

cultures from aspirate fluid are typically obtained. Treatment with 3-6 months of itraconazole

How well did you know this?
1
Not at all
2
3
4
5
Perfectly
332
Q

Diagnosis of Hirshprung’s

A

Rectal suction biopsy-absence of ganglion cells

after a contrast enema that delineates theobstruction

How well did you know this?
1
Not at all
2
3
4
5
Perfectly
333
Q

What is associated with Hirshprungs

A

Up to 25% of patients with HD have another congenital anomaly (eg, renal) and approximately 10% of patients with HD have a chromosomal defect (eg, Down syndrome).

How well did you know this?
1
Not at all
2
3
4
5
Perfectly
334
Q

pyloric stenosis,

A

nonbilious emesis without abdominal distension in infants age 4–8 weeks.

Abdominal US

How well did you know this?
1
Not at all
2
3
4
5
Perfectly
335
Q

Dx malrotation with volvulus

A

Upper gastrointestinal series

How well did you know this?
1
Not at all
2
3
4
5
Perfectly
336
Q

Complication of compartment syndrome

A

AKI - rhabdomyolisis– myoglobin

How well did you know this?
1
Not at all
2
3
4
5
Perfectly
337
Q

How is myoglobin from rhabdomyolysis toxic for kidney

A

The released myoglobin is filtered and degraded in the kidney. Heme pigment from myoglobin degradation:

Is directly toxic to proximal tubular cells
Combines with Tamm-Horsfall protein to form tubular casts
Induces vasoconstriction, reducing medullary blood flow

How well did you know this?
1
Not at all
2
3
4
5
Perfectly
338
Q

Nex step if suspect compartment syndrome

A

lower extremity tissue pressure:

Pressure >30 mm Hg or delta pressure (diastolic blood pressure − compartment pressure) <20-30 mm Hg indicates significant CS.

How well did you know this?
1
Not at all
2
3
4
5
Perfectly
339
Q

tto compartment syndrome

A

fasciotomy

How well did you know this?
1
Not at all
2
3
4
5
Perfectly
340
Q

CK needed in compartment syndrome?

A
NOT PRIORITY 
Creatine kinase (CK) levels may be elevated in patients with crush injury with or without CS and may be helpful for predicting possible renal injury.  However, waiting for CK level results may unnecessarily delay the more important diagnosis of CS.
How well did you know this?
1
Not at all
2
3
4
5
Perfectly
341
Q

SE of amlodipine

A

edema

preferential dilation of precapillary vessels (arteriolar dilation), which leads to increased capillary hydrostatic pressure and fluid extravasation into the interstitium.

other side effects of DHP CCBs: flushing headache edema

How well did you know this?
1
Not at all
2
3
4
5
Perfectly
342
Q

Rinne and Weber tests interpretation

A

see phone

How well did you know this?
1
Not at all
2
3
4
5
Perfectly
343
Q

Sudden hearing loss, no trauma, normal neuro exam

A

urgently by otolaryngology with a formal audiogram, MRI, and corticosteroid therapy.

How well did you know this?
1
Not at all
2
3
4
5
Perfectly
344
Q

CI tPA (6)

A

Presence of active internal bleeding
Bleeding diathesis (eg, platelets <100,000/µL)
Hypodensity in >33% of an arterial territory on CT scan
Presence of intracranial hemorrhage on CT scan
Intracranial surgery in the last 3 months
Blood pressure >185/110 mm Hg

How well did you know this?
1
Not at all
2
3
4
5
Perfectly
345
Q

Window for tPA

A

3-4.5hrs

How well did you know this?
1
Not at all
2
3
4
5
Perfectly
346
Q

ancillary studies for patients with stroke

A

Magnetic resonance or CT angiography of head and neck , OR dupplex carotid
EKG ( arrhythmia)
TEE Echo(presence of an intracardiac thrombus.)

** Hypercoagulability studies are NOT indicated in all patients with acute stroke. They should be ordered in those with a personal or family history of hypercoagulable disease, in young patients with no apparent etiology for stroke despite evaluation for vascular and cardiac sources, and in patients with clinical findings that suggest systemic lupus erythematosus.

How well did you know this?
1
Not at all
2
3
4
5
Perfectly
347
Q

labs dermatitis atopic

A

High serum IgE

Eosinophilia

How well did you know this?
1
Not at all
2
3
4
5
Perfectly
348
Q

management for atopic dermatitis (AD)

A

oral antihistamines, emollients to maintain skin hydration, and avoidance of excessively hot or dry environments.

Low-potency agents (eg, hydrocortisone) may be adequate for mild symptoms. More significant symptoms may warrant moderate- to high-potency glucocorticoids (eg, triamcinolone, betamethasone).

How well did you know this?
1
Not at all
2
3
4
5
Perfectly
349
Q

low potency and high potency topical glucocorticosteroids. in atopic dermatitis

A

Low-potency agents (eg, hydrocortisone) may be adequate for mild symptoms. More significant symptoms may warrant moderate- to high-potency glucocorticoids (eg, triamcinolone, betamethasone).

severe AD may require UV light.

How well did you know this?
1
Not at all
2
3
4
5
Perfectly
350
Q

Topical glucocorticoids are relatively contraindicated for Atopic dermatitis on the face, eyelids, or flexural areas;

A

calcineurin inhibitors (eg, tacrolimus) may be considered for use in these areas

How well did you know this?
1
Not at all
2
3
4
5
Perfectly
351
Q

Indications for parathyroidectomy in tertiary hyperparathyroidism

A
  1. Persistently elevated calcium (eg, >10.5 mg/dL), phosphorus, or PTH (eg, >800 pg/mL) levels
  2. Soft tissue calcification or calciphylaxis (vascular calcification with skin necrosis)
  3. Intractable bone pain or pruritus
How well did you know this?
1
Not at all
2
3
4
5
Perfectly
352
Q

How CKD leads to secondary hyperparathyroidism

A

CKD leads to 1. Decreased conversion from 25 hydroxyvit D to 1,25 hydroxyvit D. 2. Decrease PO4 clearance.

If low vit D, decreased absorption of Ca in the gut. And decreased clearance then increase PO4 that binds to calcium in the serum

aLL THESE LEAD TO LOW Ca IN SERUM AND THEN COMPENSATORY pth ELEVATION

How well did you know this?
1
Not at all
2
3
4
5
Perfectly
353
Q

secondary hyperPTH

A

elevated PTH levels, low/low-normal calcium levels, and low 1,25-dihydroxyvitamin D levels (despite adequate 25-hydroxyvitamin D stores).

How well did you know this?
1
Not at all
2
3
4
5
Perfectly
354
Q

tertiary hyperparathyroidism

A

result of prolonged secondary hyperPTH

autonomous PTH secretion unresponsive to rising calcium levels, resulting in hypercalcemia with very high PTH levels.bone disease with bone pain and an elevated bone-specific alkaline phosphatase (due to high bone turnover).

How well did you know this?
1
Not at all
2
3
4
5
Perfectly
355
Q

Stroke patients, particularly those with symptoms of dysarthria, are often at risk for oropharyngeal dysphagia.

A

In the setting of acute stroke, such patients should be given nothing (eg, food, drink, medications) by mouth until a swallow evaluation can be performed.

How well did you know this?
1
Not at all
2
3
4
5
Perfectly
356
Q

Patient with stroke passed the tPA window. Management?

A

evaluate swallow study

aspirin
Low-dose heparin or low-molecular-weight heparin to prevent stroke/DVT - particularly in those who have dense hemiparesis.

How well did you know this?
1
Not at all
2
3
4
5
Perfectly
357
Q

Types of necrotizing fascitis

A

Type I : underlying diabetes and peripheral vascular disease. It is generally a polymicrobial infection; some commonly isolated organisms include Staphylococcus aureus, Bacteroides fragilis, Escherichia coli, group A Streptococcus, and Prevotella species. - here you see more crepitus ( anaerobic)

type II: MC! usually occurs in individuals with no concurrent medical illness. Many patients report a history of laceration, blunt trauma, or a surgical procedure as a predisposing factor. It is typically caused by group A Streptococcus (Streptococcus pyogenes).

How well did you know this?
1
Not at all
2
3
4
5
Perfectly
358
Q

Microorganisms causing necrotizing fascitis

A

usually due to minor trauma or underlying disease( DM, glucocorticoid use, alcohol or drug abuse)

  • S.pyogenes ( MC if no underlying disease, and just minor trauma)
  • S aureus
  • Polymicrobial
  • C. perfringens ( MC in diabetics or PAD)
How well did you know this?
1
Not at all
2
3
4
5
Perfectly
359
Q

Presentation of necrotizing fascitis

A

Systemic ss: hypotension, fever, tachyardia

severe pain out of proportion
erythema
can have crepitus ( if c. perfringens or B. fragilis)
BULLAE

How well did you know this?
1
Not at all
2
3
4
5
Perfectly
360
Q

Tto necrotizing fascitis

A

debridement and antibiotics.

How well did you know this?
1
Not at all
2
3
4
5
Perfectly
361
Q

Drunk person who threat with suicide and now says no.

A

When there is concern that the patient may be suicidal, the physician must observe the patient and perform a suicide assessment when the patient is completely sober. Speaking with the boyfriend and contacting family members for collateral information would also be advisable.

How well did you know this?
1
Not at all
2
3
4
5
Perfectly
362
Q

Lead poisoning ss

A

anorexia, decreased activity, irritability, vague abdominal pain and insomnia, PICA behavior

How well did you know this?
1
Not at all
2
3
4
5
Perfectly
363
Q

if suspicion of lead poisoning

A

CBC, serum iron and ferritin levels, and reticulocyte count.

How well did you know this?
1
Not at all
2
3
4
5
Perfectly
364
Q

actinic keratosis (AK) tto

A

due to sun exposure

Topical 5FU cream for a period of 3–6 weeks.
other include: cryotherapy or surgical excision

How well did you know this?
1
Not at all
2
3
4
5
Perfectly
365
Q

Untreated AK is associated with?

A

Untreated AK has up to a 20% risk of progression to squamous cell carcinoma. For this reason, any AK lesions that are detected should be removed or destroyed

How well did you know this?
1
Not at all
2
3
4
5
Perfectly
366
Q

selective deficiency of IgG3

A

more common in adult females and is associated with recurrent sinopulmonary as well as gastrointestinal infections.

How well did you know this?
1
Not at all
2
3
4
5
Perfectly
367
Q

Alpha-1-antitrypsin deficiency

A

emphysema and liver damage, but does not increase the risk of recurrent infections.

How well did you know this?
1
Not at all
2
3
4
5
Perfectly
368
Q

Cystic fibrosis manifests earlier in life and does not cause gastroenteritis.

A
defect in the CFTR gene leads to impaired chloride and water transport
this causes viscous secretions in the
respiratory tract
exocrine pancreas
sweat glands
intestines
genitourinary tract
Associated conditions
nasal polyposis
infertility in men and subfertility in women
pancreatic insufficiency
How well did you know this?
1
Not at all
2
3
4
5
Perfectly
369
Q

ss in cystic fibrosis

A

a productive cough
respiratory symptoms are more prominent in adulthood
recurrent pulmonary infections
S. aureus is more common in pediatric patients (treat with vancomycin)
Pseudomonal spp. are more common in adults (treat with amikacin, ceftazidime, and ciprofloxacin)

chronic sinusitis
chronic productive cough
dyspnea on exertion
bronchiectasis
gastrointestinal symptoms are more prominent in infancy
chronic and frequent diarrhea
greasy stool with flatulence from malabsorption
secondary to pancreatic insufficiency
can lead to rectal prolapse
meconium ileus in infants (15%)

How well did you know this?
1
Not at all
2
3
4
5
Perfectly
370
Q

How do you measure respiratory status in Guillain Barre

A

frequent measurement of vital capacity and negative inspiratory force. Tidal volume.

Approximately 30% of patients with GBS ultimately require mechanical ventilation.

How well did you know this?
1
Not at all
2
3
4
5
Perfectly
371
Q

Patients with GBS should receive plasma exchange or IVIG if:

A

Nonambulatory
Within 4 weeks of symptom onset

Those who are ambulatory and recovering generally do not require treatment.

How well did you know this?
1
Not at all
2
3
4
5
Perfectly
372
Q

tto botulism

A

equine serum antitoxin and antibiotics.

How well did you know this?
1
Not at all
2
3
4
5
Perfectly
373
Q

course of disease in Guillain Barre

A

The manifestations of Guillain-Barré syndrome (GBS) tend to evolve as follows:

2 weeks of progressive motor weakness that can lead to paralysis
2-4 weeks of plateaued symptoms
Slow, spontaneous recovery over months

How well did you know this?
1
Not at all
2
3
4
5
Perfectly
374
Q

prognosis Guillain Barre

A

slow recovery

weakness may resolve spontaneously but treatments shortness the time needed for recover.

How well did you know this?
1
Not at all
2
3
4
5
Perfectly
375
Q

Measures to decrease rate of transmission

A

Contact precautions
hand washing with soap and water is more effective in C difficile spore elimination than alcohol-based sanitizers.
cleaning the patient’s environment (eg, room, medical equipment) and limiting the use of certain antibiotics (eg, clindamycin, fluoroquinolones, cephalosporins).

How well did you know this?
1
Not at all
2
3
4
5
Perfectly
376
Q

tto giardia

A

Metronidazole

How well did you know this?
1
Not at all
2
3
4
5
Perfectly
377
Q

tto giardia outbreak

A

Symptomatic individuals require metronidazole therapy.

Symptomatic students with positive stool do not need to be restrained from schoolunless they are incontinent.

NO Recreational water venues. until symptoms have resolved for 2 weeks.

How well did you know this?
1
Not at all
2
3
4
5
Perfectly
378
Q

Immune thrombocytopenia pathogenesis

A

Anti GpIIb/IIIa antibodies. Spleen macrophages consume the platelet-antibody complex. Commonly in viral illness.

How well did you know this?
1
Not at all
2
3
4
5
Perfectly
379
Q

ITT pathogenesis

A

inhibition or deficiency of ADAMS13 ( vWF metalloproteinase)- so decreased degradation of vWF.

large vWF –> increased platelet adhesion–>increased platelet aggregation and thrombosis

Labs: schystocytes, LDH,

How well did you know this?
1
Not at all
2
3
4
5
Perfectly
380
Q

ITT presentation

A
neurologic and renal ss
fever
thrombocytopenia 
microangiopathic hemolytic anemia. 
Petechial rash, abdominal pain
How well did you know this?
1
Not at all
2
3
4
5
Perfectly
381
Q

ITT tto

A

plasmapheresis, steroids.

How well did you know this?
1
Not at all
2
3
4
5
Perfectly
382
Q

Pregnancy is associated with an ADAMTS13 deficiency that becomes more pronounced with increasing gestational age and persists into the postpartum period and can precipitate TTP

A

t

How well did you know this?
1
Not at all
2
3
4
5
Perfectly
383
Q

stress fx appear in X ray

A

Possible abnormalities on x-ray may take up to 4 weeks to become apparent and include bone sclerosis, cortical thickening, periosteal elevation, and visible fracture line

How well did you know this?
1
Not at all
2
3
4
5
Perfectly
384
Q

tto of stress fracture

A

pneumatic splinting, reduced weightbearing, and a graduated exercise program. Most patients may resume full-intensity exercise within 12 weeks,

How well did you know this?
1
Not at all
2
3
4
5
Perfectly
385
Q

Hungtington disease

A

CAG triplet repeat expansion in one allele of the HD (huntingtin) gene on chromosome 4

severity depends on the number of repeats > 39 repeats: full penetrance allele

anticipation: number of repeats may increase with generations, presenting earlier.

Involuntary movements ( chorea, ataxia, tics)
cognitive deterioration ( memory, language disability)
psychiatric ss)

atrophy of caudate and putamen, as well as general cerebral atrophy

fatal. death 20 years after ss onset.

How well did you know this?
1
Not at all
2
3
4
5
Perfectly
386
Q

Treatment lyme

A

Doxy: non pregnant and children >8 years
Amoxi or cefuroxime: pregnant, lactant or children < 8
no congenital defects from lyme to baby

How well did you know this?
1
Not at all
2
3
4
5
Perfectly
387
Q

Types of lyme

A

Early localized:days to 1 m ( erythema migrans, headache, myalgias)
Early disseminated days to months (carditis AV block, cardiomypathy, VII palsy, encephalitis, meningitis, migratory arthritis, conjunctivitis, )
Late or chronic m to years ( arthritis, polyneuropathy)

How well did you know this?
1
Not at all
2
3
4
5
Perfectly
388
Q

why serology is not recommended in Lyme

A

very insensitive. IgM usually develop 1-2weeks and IgG antibodies typically after within 2-6 weeks.

BUT SEROLOGY SHOULD BE PERFORMED IN PTS WITH SIGNS OF EARLY DISSEMINATED OR LATE DISEASE

How well did you know this?
1
Not at all
2
3
4
5
Perfectly
389
Q

Indications for Lyme tto

A

must meet all 5
1.attached tick is adult or nymphal Ixodes scapularis
2. tick attached for >=36 hrs or engorged
3.prphylaxis started within 72 hrs of tick removal
4. endemic area >20%
no contraindications to doxy

How well did you know this?
1
Not at all
2
3
4
5
Perfectly
390
Q

iron def anemia

A

hemoglobin <11 g/dL, often accompanied by low mean corpuscular volume (MCV), elevated red blood cell (RBC) distribution width, and a low RBC count.

How well did you know this?
1
Not at all
2
3
4
5
Perfectly
391
Q

Differentiate thalassemia from iron def anemia

A

Mentzer index (MCV/RBC) >13 is also suggestive of iron deficiency and can help differentiate it from thalassemia;

Alpha and beta thalassemias are also microcytic but characterized by a Mentzer index <13

How well did you know this?
1
Not at all
2
3
4
5
Perfectly
392
Q

Mentzer index

A

Differentiate thalassemia from iron def anemia

Mentzer index (MCV/RBC) >13 is also suggestive of iron deficiency and can help differentiate it from thalassemia;

Alpha and beta thalassemias are also microcytic but characterized by a Mentzer index <13

How well did you know this?
1
Not at all
2
3
4
5
Perfectly
393
Q

testicular torsion

A

elevated, transverse testi
is erythematous, edematous, firm, and tender
elevating the scrotum does not tend to relieve the pain
absent cremasteric reflex

How well did you know this?
1
Not at all
2
3
4
5
Perfectly
394
Q

Timefram for surgery in testicular torsion

A

ideally within 6 hours

How well did you know this?
1
Not at all
2
3
4
5
Perfectly
395
Q

Epididymitis,

A

commonly due to chlamydia in sexually active boys, causes scrotal pain and swelling, with increased blood flow to the inflamed epididymis evident on Doppler ultrasound.

experience pain relief with scrotal elevation (Prehn sign),

How well did you know this?
1
Not at all
2
3
4
5
Perfectly
396
Q

Indications for implantable cardioverter/defibrillator in hypertrophic cardiomyopathy

A
Family history of HCM
Syncope ( w or w/o exertion) 
Hypotensive episode with exertion
nonsustained VT on Holter monitoring
Extreme LVH > 3 cm
397
Q

Although quitting at younger ages is associated with a larger absolute decline in premature mortality, cessation at age 60 or older has also been shown to lower the risk of all-cause mortality and cardiovascular events. This benefit can be seen within 5 years of quitting.

A

elderly people also benefit from quitting smoking.

decrease mortality seen 5 years of quitting.

398
Q

5 A’sapproach of smoking cessation

A
Ask
Assess readiness to quit
Advice patient to quit
Assist- pharmacologically or referral 
Arrangea quit date and a follow-up appt.
399
Q

risk of osteoporosis may decline 10 years after quitting smoking

A

true

400
Q

Dx of TB

A

First X ray: signs of active disease such as upper lobe cavitation (70%-80%), hilar lymphadenopathy, or pleural effusion.

Then SPUTUM :Three single sputum samples (spontaneous or induced) are submitted in 8- to 24-hour intervals with at least 1 early-morning sample. Sputum should be sent for acid-fast bacillus smear, mycobacterial culture, and nucleic acid amplification testing.

401
Q

Tuberculin skin test VS. Interferon-gamma release assay

A

Tuberculin skin test

High specificity in non-BCG-vaccinated patients
Cost effective, requires training for interpretation
Can diagnose latent but not active TB
Return visit required in 48 hours for final result

Interferon-gamma release assay

Blood draw required, 1 patient visit
High specificity with results in 16-24 hrs
No interference with BCG vaccine
Cannot differentiate between latent & active TB

402
Q

who to treat in TB

A

wall

403
Q

Patient in whom you suspect TB but the three sputu samples came back negative, next?

A

Do not rule it out - low sensitivity (45%-80%)
so false negatives are common.

-Additional testing with mycobacterial culture (takes 2-6 weeks)or nucleic acid amplification(<48hrs) is required for confirmation.

404
Q

Developmental dysplasia of the hip

A

abnormal acetabular development resulting in a shallow hip socket and inadequate support of the femoral head.

405
Q

ppt of Developmental dysplasia of the hip

A

DDH may be missed in the newborn period if classic symptoms and signs (eg, hip clunk, asymmetric leg creases) are absent or undiagnosed.

adolescents and young adults may have a leg-length discrepancy and gait abnormalities such as toe-walking on the affected side or a Trendelenburg gait.
activity-related pain in the front hip and groin,
premature joint degeneration and osteoarthritis

406
Q

radiation proctitis.

A

A cute:diarrhea, mucus discharge, and tenesmus (ineffectual/painful straining on defecation) during or within 6 weeks of pelvic radiation

Chronic/; >9 weeks to years after radiation therapy and is more commonly associated with strictures, fistula formation, and rectal bleeding.

407
Q

Colonoscopy in radiation proctitis.

A

lesions with pallor, friability, telangiectasias, and mucosal hemorrhage,

408
Q

Incidentaloma in adrenal glands, labs

A

serum electrolytes, dexamethasone suppression testing, and 24-hour urine catecholamine, metanephrine, vanillylmandelic acid and 17-ketosteroid measurement.

409
Q

Criteria for surgical manangement of incidentaloma

A

all functional tumors, all malignant tumors (which demonstrate a characteristic heterogenous appearance on imaging), and all tumors greater than 4 cm.

  • Rest can be observed with serial imaging and consider removal
410
Q

Hep B labs

A

elevated aminotransferases, positive hepatitis B surface ANTIGEN, hepatitis B IgM core antibody, hepatitis B e antigen (indicator of high infectivity), and detectable hepatitis B DNA.

411
Q

Management of acute Hep B

A

symptomatic patients are at very low risk of acute liver failure or other significant complications.

most cases of acute HBV (even with marked elevations in aminotransferases) resolve spontaneously and can be managed with outpatient supportive care and close follow-up.

412
Q

Hospitalization criteria in Hep B ( not a common thing)

A

fever or hemodynamic instability,
impaired hepatic synthetic function (eg, abnormal coagulation markers),
or other signs of acute liver failure (eg, encephalopathy). age >50, have poor oral intake, or have minimal social support.

413
Q

Risk of progression from acute to chronic HBV according to age.

A

the risk of progression from acute to chronic HBV infection decreases with age.

Perinatally acquired: 90%
Acquired at 1-5 years :20-50%
Adults: 5%

*Hepatitis C has a much higher risk of progression to chronic infection (approximately 75%-85%)

414
Q

Urethral swab is performed, and Gram stain of the urethral fluid reveals many neutrophils but no organisms. NAAT for Chlamydia and gonorrhea. Empiric treatment with?

A

Azythromycin. (likely chlamydia)

otherwise : Intracellular gram-negative diplococci

415
Q

Gonococcal vs. Non-Gonoccocal urethritis

A

Gonococcal: N. gonorrhea, purulent, Intracellular gram-negative diplococci, CEFTRIAXONE PLUS AZYTHRO

Non-Gonococcal: Chlamydia trachomatis
Ureaplasma urealyticum
Mycoplasma genitalium
Trichomonas vaginalis

watery, scant
Aseptic with leukocytes
Azythromycin

416
Q

Patient with urethritis that continue to have ss after initial treatment and NAAT for gonorrhea/chlamydia negative.

A

The first diagnostic step is repeating the urethral fluid Gram stain to confirm ongoing urethritis (>2 leukocytes/hpf). If positive, patients should undergo nucleic acid amplification (NAAT) testing of the urine for Chlamydia trachomatis, Neisseria gonorrhoeae, Mycoplasma genitalium, and Trichomonas vaginalis (if available).

in this scenario –>azithromycin-insensitive organism.
So possibly not Chlamydia but M. genitalium. –>moxifloxacin.

417
Q

tto of urethritis by M. genitalium.

A

moxifloxacin.

418
Q

PCOS tto

A

Weight loss (first-line)
OCPs for menstrual regulation
Progestins for endometrial protection
Clomiphene citrate for ovulation induction

419
Q

Management of decompensated heart failure

A

LMNO
Initiate with diuretics and intravenous vasodilators to reduce cardiac preload and relief ss.

An urgent pulse oximetry, chest radiograph, and electrocardiogram (ECG)

420
Q

TTo of vericose veins.

A

leg elevation, compression stockings, sclerotherapy, and surgical ligation.

Compression stockings should not be used in patients with an underlying arterial insufficiency.

421
Q

TTo of vericose veins.

A

leg elevation, weight reduction, compression stockings, sclerotherapy, and surgical ligation.

Compression stockings should not be used in patients with an underlying arterial insufficiency.

422
Q

When is surgical ligation indicated in varicose veins

A

large symptomatic varicose veins with ulcers, bleeding, or recurrent thrombophlebitis of the veins.

423
Q

Risk factors for persistent breech presentation

A
uterine anomalies (eg, leiomyoma, bicornuate uterus), placental anomalies (eg, placenta previa), 
and fetal anomalies.
424
Q

risk of asphyxia and fetal injury. is associated with which complications?

A

risk of asphyxia and fetal injury.

425
Q

When is the best time to do an external cephalic version?

A

> =37 weeks

426
Q

Treatment of graves disease

A

ALWAYS B blocker

  • ATDs- Mild
  • Radioactive iodine
  • Thyroidectomy
427
Q

Treatment of graves disease

A

ALWAYS B blocker

  • ATDs- Mild
  • Radioactive iodine
  • Thyroidectomy
  • if patient has moderate to severe ophtalmologic ss prednisone.
428
Q

When is indicated antithyroid drugs in hypertyroidism

A
  • Mild hyperthyroidism
  • Elderly who have low life expectancy
  • preparation of iodine or thyroidectomy
  • Pregnancy ( PTU in 1st trimester)
429
Q

When is indicated radioactive iodine in hypertyroidism

A

Moderate to severe hyperthyroidism( with or without ophtalmo)
patient preference in mild hypothyroidism

430
Q

When is indicated thyroidectomy in hypertyroidism

A
  1. very large goiter
  2. suspicion of Ca
  3. Coexisting primary hyperPTH
  4. Pregnant patient
  5. severe ophtalm.
  6. retrosternal goiter with obstructive ss
431
Q

RAI may cause an exacerbation of symptoms of hyperthyroidism

A

true!

432
Q

RAI may cause an exacerbation of symptoms of ophtalmology in hyperthyroidism

A

true!

433
Q

Prednisone is recommended for patients with moderate to severe Graves ophthalmopathy (eg, exophthalmos, periorbital edema, vision changes) prior to definitive treatment with surgery or RAI.

A

tryue

434
Q

When do you evaluate thyroid function after initiated treatment?

A

4-6 weeks after initiation of antithyroid drug (ATD) treatment and then every 2-3 months.

YOU ORDER , total T3 and free T4 .

TSH is not often order as it may be suppressed for a long time even after initiation of therapy and would

435
Q

major SE of antithyroid drugs

A

agranulocytosis and hepatotoxicity(mainly with PTU).

agranulocytosis -occurs within 90 days of initiation and presents with fever, chills, or sore throat. If patients on ATDs develop these symptoms, a white blood cell count with differential should be obtained. if confirm stop antibiotic

436
Q

Physician asked to treat a family member.

A

Treating family members is generally considered to be ethically problematic and potentially unsafe.
Acute and limited care may be appropriate when no other physician is available.

437
Q

Immunosuppressed woman due to chemo with fever and a rash on her right thigh. Within a few hours, a pustule formed in the center of the lesion, rapidly evolved into a dark bulla, and then ruptured and formed an ulcer.

A

Ecthyma gangrenosum- pseudomona
PAINLESS
Skin examination shows a nontender, necrotic ulcer with an erythematous rim and yellow-green, purulent exudate on the right thigh. There is another lesion with a hemorrhagic dark-bluish bulla and surrounding erythematous, indurated skin.

438
Q

Ecthyma gangrenosum

A

pseudomona’s bacteremia
In immunosuppressed

PAINLESS
Skin examination shows a nontender, necrotic ulcer with an erythematous rim and yellow-green, purulent exudate on the right thigh. There is another lesion with a hemorrhagic dark-bluish bulla and surrounding erythematous, indurated skin.

anogenital area, axilla, and extremities are most commonly affected,

FEVER is often present

439
Q

Pyoderma gangrenosum

A

associated with inflammatory bowel disease or anarthritides.

inflammatory nodules, pustules, or vesicles and quickly evolve to ulcers; however, they are usually quite painful.

440
Q

Clostridial myonecrosis,

A

aka gas gangrene. fever, severe muscle pain, and painful purple-colored bullae

441
Q

Management of ecthyma gangrenosum

A

IV antibiotics ( pip tazo, aminoglycoside)

442
Q

Dumping syndrome

A

Complication of gastrectomy- occurs when high amount of carbs are consumed

Early:rapid onset, usually within 15 minutes. It is the result of rapid emptying of food into the small bowel. Due to the hyperosmolality of the food, rapid fluid shifts from the plasma into the bowel occur, resulting in hypotension and a sympathetic nervous system response. Patients often present with colicky abdominal pain, diarrhea, nausea, and tachycardia [65].
avoid foods that are high in simple sugar content and replace them with a diet consisting of high fiber, complex carbohydrate, and protein rich foods.

Late : aka postprandial hyperinsulinemic hypoglycemia (PHH).
dizziness, fatigue, diaphoresis, and weakness, usually occur one to three hours after ingestion of a carbohydrate-rich meal, typically months to years after surgery, and are associated with documented hypoglycemia

443
Q

Dumping syndrome tto

A

decreasing the speed of the passage of fluids and food into the small gut. A high-protein and low-carbohydrate diet is advised, as well as smaller but more frequent meals throughout the day.

444
Q

Amoxicillin administration in a patient with infectious mononucleosis classically causes a generalized maculopapular rash.

A

The rash will resolve spontaneously after withdrawal of the antibiotic and observation alone.

445
Q

Prognosis of infectious mononucleosis

A

supportive tto, avoid exercise for next 3 weeks

symptoms will resolve spontaneously within weeks of onset. BUT persistent fatigue is common and may exist for months (with some patients experiencing chronic fatigue for >6 months).

446
Q

pyloric stenosis dx

A

abdominal us

Nonbloody, nonbilious, projectile vomiting
weight loss, dehydration
olive shape
peristaltic waves can be seen prior to vomiting

447
Q

complications of pyloric stenosis

A

hypokalemic and hypochloremic metabolic alkalosis results from volume contraction and the loss of gastric hydrochloric acid. Blood urea nitrogen and creatinine may be elevated due to prerenal azotemia.

448
Q

tto and diagnosis intussusception

A

air contrast enema

6-36 months with colicky abdominal pain, “currant jelly” stools, and emesis.

449
Q

RF for pyloric stenosis

A

3-6 weeks; first-born, preterm, and bottle-fed infants, erythromycin/azythromycin exposure( prophylaxis for pertussis)

450
Q

pt has pyloric stenosis , tto?

A

FIRST CORRECTION OF ELECTROLYTE ABNORMALITY they have hypokalemic and hypochloremic metabolic alkalosis

After fluid resuscitation and stabilization, surgical correction with a pyloromyotomy is indicated

451
Q

Rotavirus and intussusseption.

A

Rotavirus increases peyer;s M patch — increase intussusseption

452
Q

Anion gap formula

A

Na -( CL+HCO3)

453
Q

In diabetic patient with metabolic acidosis, AKI think of

A

metformin related lactic acidosis

Although lactic acidosis is rare in patients with normal renal function, patients with impaired renal function are at increased risk, or in patients with hypovolemia.

454
Q

tto hypoparathyroidism

A

vit D and calcium

in patients with borderline low serum calcium and high urinary calcium, as seen in the patient described in the vignette, the addition of a thiazide diuretic will not only decrease the urinary calcium, but also increase serum calcium levels.

455
Q

Complications systemic sclerosis

A

Lung: Interstitial lung disease, pulmonary arterial HTN
Kidney: HTN, scleroderma renal crisis (oliguria, thrombocytopenia, MAHA)
Heart: Myocardial fibrosis, pericarditis, pericardial effusion

456
Q

Labs systemic sclerosis

A

Antinuclear antibody
Anti-topoisomerase I (anti-Scl-70) antibody
Anticentromere antibody

457
Q

CREST syndrome,

A

limited cutaneous systemic sclerosis

Calcinosis
Raynaud 
Esophageal dysmotility
Sclerodactily
Telangiectasis
458
Q

types os systemic sclerosis:

A
  1. limited cutaneous systemic sclerosis(CREST)
  2. diffuse cutaneous systemic scleroderma (which includes more diffuse skin involvement as well as involvement of the lungs, kidneys, and gastrointestinal tract)
459
Q

All patients with scleroderma should be screened with pulmonary function testing at diagnosis.

A

In patients with interstitial lung disease, there is a concordant decline in both total lung capacity (TLC) and diffusing capacity for carbon monoxide (DLCO),

whereas in patients with pulmonary hypertension, there is a greater drop in DLCO and a relatively preserved TLC.

460
Q

PATHOLOGY OF systemic sclerosis:

A

Progressive tissue fibrosis

Vascular dysfunction

461
Q

Indications of HPV

A

All girls & women* age 11-26 (but may be given up to age 45)
Boys & men age 11-21 (up to age 26 for men who have sex with men)
Immunocompromised individuals (including HIV patients) age 11-26
Not indicated in pregnant women

In children<15 only 2 doses
In children> 15 3 doses

a person should still get the vaccine even if they are already sexually active. This is because they are unlikely to have been exposed to all of the types of HPV contained in the vaccine

462
Q

Posion Ivy type of hypersensitivity reaction

A

type IV (cell-mediated) hypersensitivity reaction

463
Q

Tto poison Ivy

A

Removal of apparel and cleansing of areas to avoid spread
cool compress, topical corticosteroids
If severe we may consider oral corticosteroids

464
Q

scabiestto

A

ivermectin

465
Q

Concern for delayed speech, next step

A

Audiology eval

466
Q

threshold for blood transfusion in most stable patients with upper gastrointestinal bleeding is

A

hemoglobin level <7 g/dL as this is associated with fewer complications and reduced mortality

A hemoglobin level >8 g/dL is recommended in patients with stable coronary artery disease or in those planning to undergo major surgery (eg, certain orthopedic procedures). Patients with acute coronary syndrome, severe thrombocytopenia, or cancer with a high bleeding risk may require a higher transfusion goal.

467
Q

when do you do surgery in pancreatitis caused by gallstones

A

IF mild disease (lack of both organ failure and local or systemic complications (eg, acute necrotic collection, heart failure), –> cholecystectomy within 7 days of clinical improvement (usually during the same hospitalization) – if not they will have ecurrence rate of 30% in 6-8 weeks.

Delayed cholecystectomy should be considered for patients with severe GP, which is characterized by persistent failure of one or more organ systems (eg, hypotension not responsive to fluid resuscitation) .

468
Q

gallstone pancreatitis : In addition to having a cholecystectomy, patients should also undergo evaluation of the biliary system via preoperative endoscopic retrograde cholangiopancreatography or intraoperative cholangiogram to ensure no gallstones remain.

A

true

469
Q

Immunocompromised with Multiple small papules with central umbilication. Some of the lesions have central necrosis and are covered with a hemorrhagic crust.

A

cutaneous cryptococcosis.- mainly head and neck

470
Q

cutaneous cryptococcosis. in HIV. At what CD4 count

A

Cryptococcus neoformans is an encapsulated yeast

CD4<100

Although meningoencephalitis , pulmonary or disseminated are the most common

471
Q

Disseminated Mycobacterium avium

A

fever, night sweats, abdominal pain, diarrhea, and weight loss

472
Q

Tto of cutaneous cryptococcosis

A

> 2 weeks of intravenous amphotericin B and oral flucytosine

473
Q

Dx of cryptococcosis

A

BIOPSY OF THE LESION

*not fungal cultures

474
Q

In digital injuries, tendons are more likely to be injured than arteries, veins, or nerves due to their relative, vulnerable, anatomic location.

A

true

475
Q

Patient treated for pneumonia, who develops PE. Wife asks if that complication could have been prevented

A

Thrombosis can occur with or without prophylaxis, but your husband should have been placed on it to decrease risk.

timely disclosure of errors help mitigate any potential to affect the patient-physician relationship.

476
Q

Sjogren is associated to other diseases?

A

Primary form: none

Secondary: to rheumatologic conditions such as RA, SLE

477
Q

Complications of xerostomia

A

dental caries, Candida, chronic esophagitis

478
Q

Sjogren manifestations

A

Xerosis (ie, abnormal dryness) (see ‘Xerosis’ below)
●Purpura, associated with vascular or hematologic abnormalities
●Raynaud phenomenon
●Cutaneous vasculitis
●Annular erythema
●Other manifestations, including eyelid dermatitis and angular cheilitis

479
Q

What questions to ask if suspecting Sjogren

A

“Do you wake up at night feeling dry, and then drink some water?” or “Do you frequently drink water to help you swallow some dry foods?”

480
Q

Amlodipine SE

A

fluid retention and urticarial rash.

481
Q

ACEIs SE

A

angioedema and urticaria.

Occasionally, lisinopril can lead to aggravation of psoriatic rash.

482
Q

Which antihypertensive causes photosensitivity?

A

Thiazides

483
Q

Management of pelvic organ prolpase

A

If not surgical candidate (cardiovascular or pulmonary comorbidities) VAGINAL PESSARY

Weight loss
Pelvic floor exercises
VAGINAL PESSARY
Surgical repair(ysterectomy with prolapse repair): in those who do not improve despite conservative management.

484
Q

tto of stress urinary incontinence

A

mid-urethral sling procedure

485
Q

isolated elevation of anti-HBc, next step?

A

rare.
1st repeat testing with all the serologies.
if remains order:HBc IgM AND LIVER TEST FUNCTION TO SEE IF ACTIVE INFECTION

*If liver enzymes are abnormal with negative IgM anti-HBc, or if there is evidence of chronic liver disease, HBV DNA should be obtained to evaluate for chronic HBV infection

TOTAL anti-HBc includes IgM and IgG

486
Q

3 occasions in which isolated anti HBc is present

A
  1. Window period
  2. Years after recovery
  3. Chronic infection
487
Q

Normal CSF Parameters

A

Cells: 0-5
Glucose: 40-70
Protein < 40

488
Q

CSF parameters for bacterial

A

Cells :>1,000
Glucose < 40
Prot > 250

Normal:
Cells: 0-5
Glucose: 40-70
Protein < 40

489
Q

CSF for TB

A

Cells :100-500
Glucose < 45
Prot 100-200

Normal:
Cells: 0-5
Glucose: 40-70
Protein < 40

490
Q

CSF for Viral

A

Cells :10-500
Glucose 40-70
Prot <150

Normal:
Cells: 0-5
Glucose: 40-70
Protein < 40

491
Q

CSF for Guillain Barre

A

Cells :0-5
Glucose 40-70
Prot 45-1000

Normal:
Cells: 0-5
Glucose: 40-70
Protein < 40

492
Q

cryptococcal meningoencephalitis

A

Markedly elevated opening pressure, often >250-300 mm H2O
Low leukocyte count (<50/mm3) (compared to other meningitides) with a lymphocytic predominance
Elevated protein and low glucose
Positive India ink preparation or cryptococcal antigen test

493
Q

tto cryptococcal meningoencephalitis

A

3 STAGES:
Induction - amphotericin B and flucytosine for >2 weeks (until symptoms abate and CSF is sterilized)
If negative cultures in LP:
Consolidation - high-dose oral fluconazole for 8 weeks
Maintenance - lower-dose oral fluconazole for >1 year to prevent recurrence

Antiretroviral treatment should generally be initiated 2-10 weeks after beginning treatment for meningitis.

494
Q

tto neurosyphilis

A

aqueous crystalline penicillin G.

495
Q

In adults, the most common causes of bacterial meningitis

A

treptococcus pneumoniae and Neisseria meningitidis

496
Q

Empiric tto for bacterial meningitis

A

Ceftriaxone +Vancomycin

In adults>50 ampicillin
as they have an increased risk of Listeria monocytogenes meningitis

497
Q

CMV encephalitis in HIV tto

A

ganciclovir plus foscarnet.

498
Q

Why cryptococcal meningitis presents with increased ICP

A

The yeast and capsular polysaccharides can clog the arachnoid villi, which prevents cerebrospinal fluid outflow and increases intracranial pressure (ICP).

499
Q

Patient dx with cryptococcal meningitis and started treatment has worsening of headache, nausea, vomiting, next step?

A

Serial lumbar punctures– signs of increased ICP
**Mannitol is NOT EFFECTIVE

Some patients may need more invasive tto such as VP shunt

500
Q

When is dexamethasone recommended in bacterial meningitis?

A

meningococcal meningitis) to reduce inflammation, morbidity, and risk of death.

501
Q

Organophosphates poisoning presentation

A
Inhibition of AChE
DUMBBELSS
Diarrhea
Urination
Myosis
Bradycardia
Bronchospasm
Excitation of skeletal muscle and CNS
Lacrimtion
Salivation 
Swating
502
Q

Treatment of organophosphate poisoning

A

Atropine( competitive inhibitor) + Pralidoxime ( regenerates ACHe if given early)

Emergent resuscitation (eg, oxygen, fluids, intubation)
Atropine &amp; pralidoxime
Activated charcoal (if within 1 hour of exposure)
503
Q

Antimuscarinic tto

A

Physostigmine!

NOT Pyridostigmine

504
Q

Pyridostigmine use

A

Myasthenia Gravis

505
Q

urinary schistosomiasis

A

urinary symptoms, terminal hematuria, and peripheral eosinophilia

506
Q

urinary symptoms, terminal hematuria, and peripheral eosinophilia in young patient from Ghana

A

urinary schistosomiasis

507
Q

Dx urinary schistosomiasis

A

identification of eggs using URINE SEDIMENT MICROSCOPY

508
Q

TTO urinary schistosomiasis

A

praziquantel

509
Q

Chronic schistosomiasis can be associated with bladder cancer, which usually presents with intermittent, gross, painless hematuria that is present throughout micturition.

A

true

510
Q

Tetanus prophylaxis in wounds

A

If Clean wound:
>=3 tetanus toxoid doses: Vaccine only if >=10 years, NO TIG
unimmunized, unknown or < 3 tetanus toxoid: Vaccine , no TIG

If Dirty:
>=3 tetanus toxoid doses: Vaccine only if >=5 years, NO TIG
unimmunized, unknown or < 3 tetanus toxoid: Vaccine plus TIG

511
Q

vaginal bleeding and a friable, exophytic cervical lesion., next step?

A

cervical biopsy

512
Q

Non stop epistaxis

A

Oxymetazoline . a squirt bottle or cotton pledget.

topical vasoconstrictors should not be used for >3 days due to the risk of rebound congestion.

Advice to humidify with saline or humidified air

513
Q

Location of Kiesselbach plexus

A

anteriorly,

514
Q

Athletes participating in intense training can develop nonpathologic cardiovascular changes, including resting sinus bradycardia with or without first-degree atrioventricular block and left ventricular hypertrophy detected on ECG.

A

In the absence of other findings suggestive of underlying cardiac disease (eg, unexplained symptoms, cardiac murmur), these patients should be reassured without further cardiac testing.

515
Q

Patient asking about reduction of melanoma risk with sunscreen.

A

Sunscreen use reduces the incidence of all 3 types of cutaneous carcinomas (basal and squamous cell, and malignant melanoma).

RECOMMEND:
15-30 minutes prior to sun exposure; sunscreen should be reapplied every 2 hours or after water exposure. Sun protection factor (SPF) 15 (Choice D) is recommended for regular daily use and is often contained in facial lotions and cosmetics. SPF >30 is recommended for outdoor work or recreation, as those participating in outdoor activities tend to have less consistent sunscreen application.

516
Q

TANNING PRODUCTS

A

dihydroxyacetone which temporarily darkens or “bronzes” the skin. These are generally safe but the color does not protect from UV rays. Patients should also be discouraged from tanning bed and suntan lotion/oil use as these products increase exposure to UV light.

517
Q

Sunscreen use reduces the incidence of all 3 types of cutaneous carcinomas (basal and squamous cell, and malignant melanoma).

A

TRUE

518
Q

patient fell onto his outstretched hand and has acute pain at the wrist and tenderness

A

SCAPHOID FX , x-rays of the wrist in full pronation and ulnar deviation

519
Q

Management of scaphoid FX

A

If nothing is seen in imaging: wrist splint and imaging in a. week.

Nondisplaced fractures should be treated with a short arm thumb spica cast, but other fractures should be referred to an orthopedic surgeon for evaluation.

520
Q

Colles fracture

A

distal radius fracture
- deformity - dinner fork deformity
order lateral radiography

521
Q

X-rays can initially be negative for nondisplaced scaphoid fractures.

A

Patients with suspected scaphoid fracture should be evaluated further with either an immediate MRI or CT scan, repeat x-ray in 7-14 days, or bone scan in 3-5 days.

522
Q

Inadequate management of scaphoid fracture lead to ?

A

nonunion and avascular necrosis.

arterial blood supply to the scaphoid enters in the distal pole and travels to the proximal pole

this is why these fractures require prolonged immobilizations: >12 weeks

523
Q

Dupuytren contracture, what is it? what is associated with?

A

progressive fibrosis of the palmar fascia of unknown etiology.

diabetes mellitus
 repetitive vibration to the hands, 
CRPS, 
alcohol
malignancy.
524
Q

peak expiratory flow in asthma exacerbation;

A

a drop in peak expiratory flow >20% is consistent with the diagnosis.

525
Q

outpt tto asthma exacerbation;

A
b agonists
oral steroids (eg, prednisone 40 mg daily for 5-10 days)
526
Q

chest x ray in asthma

A

Chest x-ray is not routinely obtained during an asthma exacerbation unless there is evidence of concurrent pneumonia

527
Q

RF urinary tract malignancy

A

gross hematuria, age >40, male gender, and smoking history. Other potential risk factors include history of pelvic radiation or exposure to aniline dyes

528
Q

suspect bladder cancer, next step?

A

cystoscopy

529
Q

Gross hematuria warrants evaluation of both the upper and lower urinary tracts.

A

upper with CT contrast- CI in renal disease, so US

lower with cystoscopy

530
Q

Dx of DM

A

HbA1C ≥6.5% = diabetes mellitus
FASTING GLUCOSE: ≥126 mg/dL = diabetes mellitus
Random glucose levels:≥200 mg/dL with symptoms of hyperglycemia = diabetes mellitus
oral glucose tolerance test ≥200 mg/dL

Asymptomatic patients with an abnormal screening test for diabetes require a repeat measurement with the same test to confirm the diagnosis. Patients with symptomatic hyperglycemia and abnormal screening tests can be diagnosed with diabetes without repeating confirmatory testing.

531
Q

Bicuspid aortic valve is associated with..

A

aortic dilation, aortic aneurysm, and aortic dissection, and should be screened with imaging of the aortic root and proximal aorta.

532
Q

Biscupid aortic valve manangement

A

Echo every 1-2 years
Ballon valvoplasty or surgery

They are at high risk of infective endocarditis, aortic dilation, stenosis or dilation.

533
Q

vitamin B12 deficiency,

A

increase in indirect bilirubin due to ineffective erythropoiesis, as well as pancytopenia or bicytopenia, as in this case. The neurologic presentation includes ataxia, dementia, and occasionally, delirium.

534
Q

Lab testing for dementia

A

Routine: CBC, vitamin B12, TSH, CMP
Selective (specific risk factors): folate, syphilis, vitamin D level
Atypical (early onset): CSF

535
Q

Patient who is having dementia, and alcohol abuse. wHAT LAB TO ORDER

A

Folate

Although thiamine deficiency and subsequent Wernicke-Korsakoff syndrome may also lead to cognitive decline in patients with alcoholism, suspected cases are generally treated empirically as the utility of laboratory tests for thiamine is low.

536
Q

most frequent cause of asthma

A

house dust mite allergens

other are
cat allergens, dog allergens

** air pollution is not considered

537
Q

Wallenberg sx presentation

A

.

538
Q

Women with hypothyroidism + increased liver enzymes, next step?

A

autoimmune hepatitis - so order ANA AND Anti Smith antibodies

slight elevation of ALT/AST
normal bilirubin levels

539
Q

Antimitochondrial antibodies

A

primary biliary cirrhosis (PBC),

lymphocytic infiltrate +granuloma destruction of intralobar ducts.

Ursodeoxycholic acid

elevated alkaline phosphatase level , high ALP, Bilirrubin.

540
Q

high gamma-glutamyl transferase (GGT)

A

cholestasis.

541
Q

TCA intoxication

A

Drowsy, delirium, coma
seizures,respiratory distress

tachycardia, HYPOTENSION,
PROLONGED QT,PR,RR
CAN PROGRESS TO VT OR VFIB

Dry mouth, blurred vision
urinary retention

542
Q

Why do you use sodium bicarb in TCA toxicity

A

prevent arrhythmia- can evolve to Vfib and Vtach

increases serum pH and extracellular sodium. The increased pH (goal 7.50-7.55) modifies TCAs to their neutral (non-ionized) form, making them less available to bind to the rapid sodium channels.

543
Q

Tto of TCA toxicity

A

Oxygen, intubation
IV fluids
Activated charcoal if within 2 hours ( unless ileus is present)
Intravenous sodium bicarbonate for QRS widening and/or ventricular arrhythmia

544
Q

How TCAs cause arrhythmia

A

inhibit fast sodium channels in the His-Purkinje tissue and the myocardium to decrease conduction speed, increase phase 0 depolarization, and prolong the refractory period.
This can prolong the QRS interval (>100 msec) and cause arrhythmias (eg, ventricular tachycardia, ventricular fibrillation).

545
Q

Patient with TCA refractory to sodium bicarb, what are some options

A

magnesium or lidocaine.

546
Q

N-acetylcysteine is the antidote for

A

acetaminophen

547
Q

Salicylate toxicity tto

A

IV sodium bicarbonate to alkalinize the urine and promote salicylate excretion by trapping the salicylate in its ionized form.

548
Q

severe lactic acidosis with severe acidemia (serum pH <7.1) and serum bicarbonate <6 mEq/L., tto?

A

sodium bicarbonate

549
Q

Uses of sodium bicarb

A
  • TCA poisoning:prevent arrhythmias
  • Salicylate tox: alcalinize urine and favor excretion
  • Severe lactic acidosis ph<7.1, serum bicarb < 6
  • significant hyperkalemia
550
Q

What does cross sectional studies is able to measure

A

prevalence

551
Q

Measure of case control studies

A

OR

552
Q

Measure of cohort studies

A

RR

553
Q

Phase I study

A

healthy volunteers- safety, toxicity, pharmacodynamics/pharmacokinetics

554
Q

Phase II study

A

small number of patients with disease

Does it work? treatment efficacy, dosing, adverse events

555
Q

Phase III study

A

large population of patients with disease

Compares the new treatment to current standard of care-is it good or better

556
Q

measure to ruling out disease

A

Sensitivity

557
Q

The higher the sensitivity

A

ruling out disease and indicate a low false negative rate.
High sensitivity test used for screening in disease with LOW prevalence

SN N OUT: When negative rules out

558
Q

Specificicy

A

When high specificity and positive test rules in.

SP P IN

559
Q

PPV and NPV are affected by the prevalence of the disease

A

yes

560
Q

Incidence

A

of cases during a period of time / # of people AT RISK

561
Q

Crude mortality rate

A

deaths/total size of population

562
Q

Cause specific mortality rate

A

of deaths from a specific disease/total population

563
Q

Case fatality rate

A

of deaths from a specific disease/total population affected by the disease

564
Q

SMR

A

observed deaths /expected

565
Q

Attack rate

A

of patients with disease/population at risk

566
Q

Maternity Mortality rate

A

maternal deaths/number of live births

567
Q

Treatment of scabies

A

Topical permethrin or 2 doses of ivermectin. Permethrin should be applied from neck to feet soles, and washed out after 8-12 hours.

568
Q

Movements triggering BBPV

A

rolling head in the bed or looking up while standing

569
Q

Dx and treatment of BBPV

A

Dx: Dix Hall pike
TTo: epley manueve

570
Q

Patient who had prolonged hospitalization, history of stroke and left hemiparesis, and that has troubles walking now. At exam has limited extension of the leg. Cause

A

Deconditioning, also some other things is that when he tries to walk has tachycardia. Its not contracture, as it may be related with prior stroke

571
Q

Critical illness neuropathy

A

limb and respiratory muscle weakness. Patients are very difficult to wean from ventilatory support, and have encephalopathy or coma

572
Q

Patient with insomnia and sadness following death of husband. Next step

A

Provide education of grief reactions and counsel sleep hygiene. BZDs are NOT HELPFUL

573
Q

BZDs in insomnia/sadness in the setting of grief

A

BZDs are NOT HELPFUL

574
Q

Bilious emesis, abdominal distension, hypoactive bowel sounds, failure to pass meconium in 48 hours

A

Hirshprung

575
Q

Dx of Hirshprungs

A

Suction biopasy- absence of ganglion cells and nerve fiber hypertrophy

576
Q

Patient with lispro and glargine that has high Bedtime glucose. how to modify the insulin dose

A

Increase the pre-dinner insulin

577
Q

Differential of hepatic abscess

A

Protozoa( Entamoeba- developing countries, poor sanitation, colitis(stool ova and parasites),liver abscess(serology), tto with MTZ and paramomycin, NO ORGANISMS IN GRAM STAIN )

Bacterial,Pyogenic ( pt with underlying disease DM, hepatobiliary disease, generally after surgery, GI infection, appendicitis)-GRAM STAIN OF ASPIRATE

Hydatid(Ecchinococcus, contact with dogs, sheeps. Egg shells calcification of livery cyst, EOSINOPHILIA, Surgery under coverage with albendazole

578
Q

41 yo male with UTI ss in addition to fever, malaise, myalgias, PERINEAL DISCOMFORT

A

Has a UTI + either cystitis or acute bacterial prostatitis:

UTI+ cystitis: older males, urinary tract abnormality or recent intrumentation

Acute bacterial prostatitis: young male with normal anatomy. prostate warmth, tender. PERINEAL DISCOMFORT

RECTAL EXAM IS NEEDED TO DIFFERENTIATE THE TWO

579
Q

Tto of uncomplicated UTI in men

A

Ciprofloxacin or TMP/SMX

580
Q

Tto of acute bacterial prostatitis

A

Ciprofloxacin or TMP/SMX for 6 weeks

581
Q

Management of coin ingestion > 24 hours or sympatomatic

A

FLEXIBLE ENDOSCOPY

582
Q

Management of coin ingestion < 24 hours AND Asymptomatic

A

Observation and Xray in 24 to 48 hours

583
Q

Imaging in coin ingestion

A

2 view chest x ray anteroposterior and lateral

584
Q

When to use emergent bronchoscopy

A

foreign body causing airway compromise

585
Q

Indications for AAA repair

A
  1. Sympatomatic( abdominal or back pain)
  2. > 5.5 cm
  3. Growth of 0.5 or more in 6 months

If not then monitor with US or CT scan every 6 months during 2 years

586
Q

Patient with abdominal cramping, emesis, hyperactive sounds and mass in the abdomen, next steps

A

small bowel obstruction-LIKELY BY INCARCERATED INGUINAL HERNIA , SURGERY ! given risk of strangulation

587
Q

MC of small bowel obstruction

A

Hernias, postoperative adhesions, tumors

588
Q

Mesenteric angiography

A

when suspecting mesenteric ischemia. some risk factors include atrial fibrillation, and presents with bloody stool and abdominal pain. no masses.

589
Q

Effect of PTH in Ca, Phosph, Cl

A

Hypercalciuria, Hypercalcemia
HypoPhos
HyperCl

590
Q

Indications of Parathyroidectomy

A

Age <50
Symptomatic ( muscle/bone pain, constipation, osteoporosis, kidney stones)
Osteoporosis ( T score1 mg/dl above normal
Urine Ca excretion >400

591
Q

Pt with hyperPTH who does not meet criteria for surgery, next step

A

Follow up with regular serum ca, creatinine, and DXA

592
Q

Spiral fracture in femur baby

A

Child abuse- other typical fractures posterior rib fractures, methaphysial corners, fractures at varios stages of healing

593
Q

How many cafe au lait spots need to be for NF dx

A

> =6

594
Q

Defect in osteogenesis imperfecta

A

Collagen, multiple fractures in NEONATAL period and blue sclera

595
Q

Two main factors contributing to foot ulcer development

A
  1. Diabetic Neuropathy 2. Vascular disease ( decreased dorsalis pulse)
596
Q

Prevent diabetic foot ulcer

A

Tight glycemic control

597
Q

After MI what are the recommendations to return to work/sex, and who needs a stress test prior to resuming activities

A

Uncomplicated MI with no additional symptoms are ok to resume activities at 2 weeks

Patients with intermediate risk ( stable angina, incomplete revascularization, LV EF<40) need a stress test prior

Patients who have unstable angina, congestive HF need evaluation and stabilization prior to that - could be 3-4 months.

598
Q

Indications for statin therapy

A

Any stroke/MI/stable angina ( If=< 75 high intensity, >75 low intensity)

LDL>=190 high intensity

> 40 +DM ( If risk >=20 high intensity, if risk < 20 low intensity)

Risk >=75 High t moderate

599
Q

What statin is recommended after MI

A

HIGH INTENSITY - atorvastatin 40-80 or rosuvastatin 20-40

600
Q

Causes of Erythema multiforme

A

HSV, Mycoplasma
antibiotics, allopurinol
autoimmune diseases

601
Q

Prognosis erythema mutliforme

A

most resolve 1-2 weeks without intervention

602
Q

other symptoms that can occur with erythema multiforme

A

oral lesions(erythema, erosions, bullae) and flu likeprodrome

603
Q

Management of breast mass in =<30

A

FIRST US, +/- mammogram
If simple cyst- FNA
If complex/solid mass- Core biopsy

604
Q

Management of breast mass in>30

A

FIRST Mammogram! if benign or indeterminate do US
to characterize lesions

If mammogram shows complex or suspicious lesion core biopsy.

605
Q

When do you do excisional biopsy vs core biopsy in breast mass

A

when core biopsy cant be performed
when cyst does not resolve with FNA
When core biopsy shows indeterminate or atypical results ( carcinoma insitu, hyperplasia)

606
Q

Patient with lumbar pain + degenerative changes in X ray ( disc space narrowing, traction ostephytes, endplate sclerosis), next step

A

IF NO RED FLAGS( Hx of cancer, pain at night, fever, weight loss), then NSAIDs and follow-up in 6 weeks

607
Q

When do you order MRI in lumbar pain

A

progressive neuro symptoms, suspicion of cancer, pain persisting > 12 weeks

608
Q

Differential of postpartum hemorrhage

A

Uterine atony ( RF:prolonged labior, chorioamnionitis, enlarged uterus(macrosomia, twins,,polyhydramnios) boggy enlarged uterus —- bimanual pelvic massagge and oxytocin

Retained products of contraception ( succenturiate placenta, manual extraction, enlarged boggy uterus, placenta missing cotiledones, retained fragment—-manual extraction

Laceration

Coagulopathy

Uterine rupture: severe abdominal pain , hypotension, fetal parts sin abdomen

609
Q

Newly diagnosed HTN with no additional risk factors, next step?

A

FIRST non-pharmacologic measures: WEight loss(< 25 BMI), DASH diet (high fruits and vegetables, low fat), Exercise 30 min 5 times a day, Dietary Na <3 g/day , < 2 drinks a day in men and < 1 drink in women.

pt may be observed up to a year, if BP further increases or does not normalize then initiate antihypertensive med

610
Q

Shortness of breath, pleuritic pain, sinus tachycardia, hypoxemia

A

Pulmonary embolism

611
Q

Potential complication of acute PE

A

Pulmonary infarct - in periphery of the lung ( irritate pleura and causes pleural pain)

612
Q

what does bleomycin causes in. the lung?

A

Fibrosis -diffuse pattern of reticular septal thickening and honeycombing.

613
Q

Patient with hepatitis B progressive fatigue, increased abdominal girth, decreased appetite and an US showing a 3 cm lesion in one of the hepatic lobes

A

Fibrosis -diffuse pattern of reticular septal thickening and honeycombing.

614
Q

Etiology of stress urinary incontinence

A

urehtral hypermobility , decreased tone of urethral sphincter

615
Q

etiology ofurge urinary incontinence

A

Hyperactivity of detrusor muscle

616
Q

etiology ofoverflow urinary incontinence

A

impaired detrusor activity, bladder outlet obstruction

617
Q

Relationship urinary incontinence overflow and diabetes

A

Patients with diabetic neuropathy may have urinary incontinence . Also look for other signs such as postural hypotension is an evidence of impairment of autonomic system in diabetics

618
Q

Dx overflow incontinence

A

> 200 mL

619
Q

Meds that can cause urinary retention

A

anticholinergics, opioids, alpha 1 agonists

620
Q

Common causes of overflow urinary retention

A

Diabetic neuropathy and BPH

621
Q

Tto of hot flashes

A

If no hysterectomy combined progesterone/estrogen. If hysterectomy estrogen. CI to these therapy include Breast Ca, Stroke, DVT, CHD, liver disease. In women < 60 is considered less risk and if no RF short period 3-5 years.

622
Q

Statin induced myositis medications

A

Macrolides, Gemfibrozile, HIV protease inhibitors Cyclosporine

623
Q

When do you give Hib vaccine?

A

Usually in < 5years, starting at 2 months.

624
Q

When do you give Hep vaccine

A

0,1,6 ( unless 2kg). If exposed vaccinated: no tto. If exposed unvaccinated: Ig and vaccine. Other : chronic liver disease and high risk ( men sex men, IV drugs, healthcare workers)

625
Q

When do you give meningococcal vaccine

A

11-12 years and booster 16-21.

626
Q

When do you give pneumococcus

A
19-65
  PPSV23: chronic heart, lung, liver disease, DM, Alcoholic, liver disease
PCV13 PLUS PPSV23 ( high risk patients)
   CSF leak, cochlear implant 
   Sickle cell , asplenia
   immunocompromised 
CKD

> 65 PCV13 plus PPSV23 WITH A 8 WEEK DIFFERENCE

627
Q

When to give herpes zooster vaccine

A

> =60 years

CI:Immunocsuppressed, Ca, pregnant

628
Q

Patient that says to doctor No resuscitative measures , but doesnt sign anything.When it happens what to do?

A

Follow next kin desire.

Patient had needed to sign it.
If oral advaced directive is done needs the physician plus two witnesses.

629
Q

Tto of kawasaki

A

Aspirin + IgG

630
Q

Kawasaki ppt

A

Rule of 5: 90% < 5 years. Fever >=5 + 4 of the following: 1. Bilateral conjunctitivs, nonexudative 2. Cervical LAD >=1 of 1.5cm, 3. mucositis 4. Rash, 5. erythema and edema of the hands. Patients < 6 months have often atypical kawasaki =<3 criteria but have inflammatory markers CRP, ESR

631
Q

intrinsic pathway disorders

A

Hemophilia A and B( def of factor presents early) , Von Willebrand disorder, acquired coagulation factor inhibition( presents late)

632
Q

tto Aflutter

A

Same as Afib. If <48 hrs cardioversion. If > 48 hours: warfarin, rivaroxaban or dabigatran

> =3 weeks of anticoagulants is needed prior to think on cardioversion.

633
Q

anti mitochondrial antibodies

A

primary biliary cirrhosis

634
Q

Pulmonary and cardiac manifestations of scleroderma

A

Pulmonary fibrosis presenting as angina, shortness of breath

HF: prominent heave in the lower sternum

635
Q

Peu d’orange,erythema, lymph node involvement dx and tto

A

“inflammatory breast cancer

Do a core needle biopsy / Is often confused with mastitis– MASTITIS THAT DOESN’T IMPROVE AFTER ABCS. “

636
Q

recurrent infections of the lower extremity, and evidence of no DVT,

A

Cellulitis is often the first, but if recurrent think of: fungal infection (Tinea pedis)- ESPECIALLY IN THOSE WHO HAD SAPHENECTOMY , lymphedema or chronic venous insufficiency.

637
Q

Types of acne and tto

A

CIN

Comedonal -Topical retinoids, salicilic, glycolid acid

Inflammatory-
Mild: topical retinoids + benzoyl peroxidase
Moderate: Topical antibiotics ( erythromycin, clindamycin)
Severe: oral antibiotics

Nodulo cystic-
Moderate: topical retinoids + benzoyl peroxidase+ topical antibiotics
Severe:add oral antibiotics
Refractory: ISOTRETINOIN

638
Q

Frequent UTIs in the setting of chronic diarrhea, lower abdominal pain, chronic back pain, weight loss

A

THINK OF INFLAMMATORY BOWEL DISEASE –enterovesical fistula. - think of E.coli, bacteroides fragilis.
Pneumaturia can also be seen

639
Q

46 yo with intermenstrual bleeding and new onset heavy bleeding

A

Think about fibroids and endometrial cancer and hyperplasia.– endometrial biopsy is needed

640
Q

First line treatment for hypertension in patients with gout

A

ARBs– losartan, uricosuric event
and you can add Ca channel blockers ( amlodipine)

thiazide should be avoided

641
Q

Positive psoas sign

A

psoas abscess, retroperitoneal hemorrhage

642
Q

Patient with sudden onset abdominal pain in right lower quadrant , hypotense, psoas sign positive. Afib noted., next steps

A

Retroperitoneal hematoma, CT scan

643
Q

Patient with retroperitoneal hemorrhage in warfarin, tto

A

intravenous K and fresh frozen plasma

644
Q

WaRrfarin anticoagulation

A

vit K and FFP

645
Q

Pt with Minimental score 25/30 ( normal>=26) that is able to understand consequences but still refuses to tto

A

RESPECT WISHES- COMPREHENSION, CONSEQUENCS, CHOICE

646
Q

Prior to prescribing antidepressve in depression screen for:

A

Maniac episodes

647
Q

Mom with ADHD ASKS IF THEY CAN HAVE SPECIAL ACCOMODATIONS

A

YES

648
Q

Treatment scabies

A

Topical permethrin or 2 doses of ivermectin. Permethrin should be applied from neck to feet soles, and washed out after 8-12 hours.

649
Q

Dx of scabies

A

often clinically, but skin scraping

650
Q

Movements triggering BBPV

A

rolling head in the bed or looking up while standing

651
Q

Dx and tto of BBPV

A

Dx: Dix Hall pike
TTo: epley manueve

652
Q

Patient who had prolonged hospitalization, history of stroke and left hemiparesis, and that has troubles walking now. At exam has limited extension of the leg. Cause

A

Deconditioning, also some other things is that when he tries to walk has tachycardia. Its not contracture, as it may be related with prior stroke

653
Q

Critical illness neuropathy

A

limb and respiratory muscle weakness. Patients are very difficult to wean from ventilatory support, and have encephalopathy or coma

654
Q

Patient with insomnia and sadness following death of husband. Next step

A

Provide education of grief reactions and counsel sleep hygiene. BZDs are NOT HELPFUL

655
Q

Bilious emesis, abdominal distension, hypoactive bowel sounds, failure to pass meconium in 48 hours

A

Hirshprung

656
Q

Dx of Hirshprungs

A

Suction biopsy - absence of ganglion cells and nerve fiber hypertrophy

657
Q

Patient with lispro and glargine that has high Bedtime glucose. how to modify the insulin dose

A

Increase the pre-dinner insulin

658
Q

Differential of hepatic abscess

A

Protozoa( Entamoeba- developing countries, poor sanitation, colitis(stool ova and parasites),liver abscess(serology), tto with MTZ and paramomycin, NO ORGANISMS IN GRAM STAIN )

Bacterial,Pyogenic ( pt with underlying disease DM, hepatobiliary disease, generally after surgery, GI infection, appendicitis)-GRAM STAIN OF ASPIRATE

Hydatid(Ecchinococcus, contact with dogs, sheeps. Egg shells calcification of livery cyst, EOSINOPHILIA, Surgery under coverage with albendazole

659
Q

41 yo male with UTI ss in addition to fever, malaise, myalgias, PERINEAL DISCOMFORT

A

Has a UTI + either cystitis or acute bacterial prostatitis:
UTI+ cystitis: older males, urinary tract abnormality or recent intrumentation
Acute bacterial prostatitis: young male with normal anatomy. prostate warmth, tender. PERINEAL DISCOMFORT
RECTAL EXAM IS NEEDED TO DIFFERENTIATE THE TWO

660
Q

Treatment of uncomplicated UTI in men

A

Ciprofloxacin or TMP/SMX

661
Q

Treatment of acute bacterial prostatitis

A

Ciprofloxacin or TMP/SMX for 6 weeks

662
Q

Management of coin ingestion > 24 hours or symptomatic

A

FLEXIBLE ENDOSCOPY

663
Q

Management of coin ingestion < 24 hours AND Asymptomatic

A

Observation and Xray in 24 to 48 hours

664
Q

Imaging in coin ingestion

A

2 view chest x ray anteroposterior and lateral

665
Q

When to use emergent bronchoscopy

A

foreign body causing airway compromise

666
Q

Indications AAA repair

A
  1. Sympatomatic( abdominal or back pain)
  2. > 5.5 cm
  3. Growth of 0.5 or more in 6 months

If not then monitor with US or CT scan every 6 months during 2 years

667
Q

Patient with abdominal cramping, emesis, hyperactive sounds and mass in the abdomen, next steps

A

small bowel obstruction- LIKELY BY INCARCERATED INGUINAL HERNIA, SURGERY ! Given risk of strangulation

668
Q

MC of small bowel obstruction

A

Hernias, postoperative adhesions, tumors

669
Q

Mesenteric angiography

A

when suspecting mesenteric ischemia. some risk factors include atrial fibrillation, and presents with bloody stool and abdominal pain. no masses.

670
Q

Effect of PTH in Ca, Phosph, Cl

A

Hypercalciuria, Hypercalcemia
HypoPhos
HyperCl

671
Q

Indications parathyroidectomy

A

Age <50
Symptomatic ( muscle/bone pain, constipation, osteoporosis, kidney stones)
Osteoporosis ( T score1 mg/dl above normal
Urine Ca excretion >400

672
Q

Pt with hyperPTH who does not meet criteria for surgery, next step

A

Follow up with regular serum ca, creatinine, and DXA

673
Q

Spiral fracture in femur baby

A

Child abuse- other typical fractures posterior rib fractures, methaphysial corners, fractures at varios stages of healing

674
Q

How many cafe au lait spots need to be for NF dx

A

> =6

675
Q

Defect in osteogenesis imperfecta

A

Collagen, multiple fractures in NEONATAL period and blue sclera

676
Q

Two main factors contributing to foot ulcer development

A
  1. Diabetic Neuropathy 2. Vascular disease ( decreased dorsalis pulse)
677
Q

Prevent diabetic foot ulcer

A

Tight glycemic control

678
Q

After MI what are the recommendations to return to work/sex, and who needs a stress test prior to resuming activities

A

Uncomplicated MI with no additional symptoms are ok to resume activities at 2 weeks
Patients with intermediate risk ( stable angina, incomplete revascularization, LV EF<40) need a stress test prior
Patients who have unstable angina, congestive HF need evaluation and stabilization prior to that – couldbe 3-4 months

679
Q

Indications for statin therapy

A

Any stroke/MI/stable angina ( If=< 75 high intensity, >75 low intensity)
LDL>=190 high intensity
>40 +DM ( If risk >=20 high intensity, if risk < 20 low intensity)
Risk >=75 High t moderate

680
Q

What statin is recommended after MI

A

HIGH INTENSITY - atorvastatin 40-80 or rosuvastatin 20-40

681
Q

Causes of Erythema multiforme

A

HSV, Mycoplasma
antibiotics, allopurinol
autoimmune diseases

682
Q

Prognosis erythema mutliforme

A

most resolve 1-2 weeks without intervention

683
Q

other symptoms that can occur with erythema multiforme

A

oral lesions(erythema, erosions, bullae) and flu likeprodrome

684
Q

Management of breast mass in =<30

A

FIRST US, +/- mammogram
If simple cyst- FNA
If complex/solid mass- Core biopsy

685
Q

Management of breast mass in>30

A

FIRST Mammogram! if benign or indeterminate do US
to characterize lesions

If mammogram shows complex or suspicious lesion core biopsy.

686
Q

When do you do excisional biopsy vs core biopsy in breast mass

A

when core biopsy cant be performed
when cyst does not resolve with FNA
When core biopsy shows indeterminate or atypical results ( carcinoma insitu, hyperplasia)

687
Q

Patient with lumbar pain + degenerative changes in X ray ( disc space narrowing, traction ostephytes, endplate sclerosis), next step

A

IF NO RED FLAGS( Hx of cancer, pain at night, fever, weight loss), then NSAIDs and follow-up in 6 weeks

688
Q

When do you order MRI in lumbar pain

A

progressive neuro symptoms, suspicion of cancer, pain persisting > 12 weeks

689
Q

Differential of postpartum hemorrhage

A

Uterine atony ( RF:prolonged labior, chorioamnionitis, enlarged uterus(macrosomia, twins,,polyhydramnios) boggy enlarged uterus —- bimanual pelvic massagge and oxytocin

Retained products of contraception ( succenturiate placenta, manual extraction, enlarged boggy uterus, placenta missing cotiledones, retained fragment—-manual extraction

Laceration

Coagulopathy

Uterine rupture: severe abdominal pain , hypotension, fetal parts sin abdomen

690
Q

Anal cancer is associated with HPV

A

Is higher in patients with HIV

691
Q

Newly diagnosed HTN with no additional risk factors, next step?

A

FIRST non-pharmacologic measures: WEight loss(< 25 BMI), DASH diet (high fruits and vegetables, low fat), Exercise 30 min 5 times a day, Dietary Na <3 g/day , < 2 drinks a day in men and < 1 drink in women.

pt may be observed up to a year, if BP further increases or does not normalize then initiate antihypertensive med

692
Q

Dx of DM

A

fasting glucose > 126 or Hb >=6.5

693
Q

Shortness of breath, pleuritic pain, sinus tachycardia, hypoxemia

A

Pulmonary embolism

694
Q

Potential complication of acute PE

A

Pulmonary infarct - in periphery of the lung ( irritate pleura and causes pleural pain)

695
Q

what does bleomycin causes in. the lung?

A

Fibrosis -diffuse pattern of reticular septal thickening and honeycombing.

696
Q

Pulmonary infarct in CT

A

peripherally located, Hemispherical consolidation that abutts the pleura.

697
Q

Patient with hepatitis B progressive fatigue, increased abdominal girth, decreased appetite and an US showing a 3 cm lesion in one of the hepatic lobes

A

Hepatocellular Carcinoma!!- liver parenchyma. It arrives from the hepatocytes not the intersticial liver cells.

698
Q

AFP relation with liver

A

AFT is elevated in 60% of hepatocellular carcinoma

699
Q

Presentation of cholangiocarcinoma

A

jaundice, bile duct dilation, abdominal pain, weight loss.

700
Q

Role of physician in organ donation

A

Physician may initiate the process of organ donation with family if he is trained for doing that

or the organ procuremetnt organization staff member

701
Q

Gleason score

A

The Gleason score is calculated by adding together the two grades of cancer cells that make up the largest areas of the biopsied tissue sample. The Gleason score usually ranges from 6 to 10. The lower the Gleason score, the more the cancer cells look like normal cells and are likely to grow and spread slowly.

702
Q

Prognosis of prostate Ca

A

Patients with very low risk ( PSA <10, Normal rectal exam, gleason score <6) have excellent 10 year survival and active surveillance should be made. –PSA every 3-6 months, yearly rectal exam, and repeat a biopsy by the end of year 1. in high risk patients: radiotherapy or radical prostectomy– GI irritation and erectyle dysfunction.

703
Q

Inhalants presentation

A

Euphoria,PUPILARY DILATION, lethargy, uncoordination, loss of consciousness . Effects last15-45 min, dermatitis in nose, carrhtyhtmias. INHALANTS ARE NOT IN THE MEDICAL DRUG SCREEN

704
Q

Drugs causing hyperK

A
  1. Non selective b blockers( labetalol, carvedilol, propanolol, nadolol)
  2. ACEis
  3. ARBS
  4. K sparing diuretics- spironolactone
  5. Digoxin
  6. NSAIDS
  7. Heparin
  8. Succinylcholine
705
Q

Treatment for pertussis

A

Azythromycin, and treat all close contacts regardless of the immunization status. Incompletely immunized contacts should receive also the vaccine.

706
Q

Phases of pertussis

A

Catarrhal(1-2 weeks) mild cough, rhinitis, Paroxysmal ( 2-6 weeks) inspiratory whoop, cough with postusive emesis, APNEA IN INFANTS, Convalescent: chronic ocugh.

707
Q

bilateral reticular opacities, enlarged hilium, irregular thickening of the bronchovascular bundles predominantly affecting UPPER LUNGS.

A

Sarcoidosis

708
Q

Besides hyperCa, bilateral hilar lymph nodes how does sarcoidosis presents

A

shortness of. breath, malaise, cough , weight loss, african american

709
Q

Silicosis presentation

A

sandblasting, mines, eggshell calcification of the hilar lymph nodes . CXray nodular opaicities. risk of TB, Bronchogenic cancer.

710
Q

Dx of sarcoidosis

A

non caseating granulomas in transbronchial biopsy

711
Q

Management of inguinal hernia

A

If asymptomatic: surgical repair 1-2 weeks. If asymptomatic: surgery.

712
Q

Direct vs Indirect hernia

A

Direct: medial to epigastric vessesls, wekaness of abdominal wall, behind the inguinal ring.
Indirect: lateral to epigastric vessels, same course as the spermatic cord, can pass to the scrotum or labia mayora. PATENT PROCESSUS VAGINALIS

713
Q

prognosis of hydroceles

A

resolve by age 1

714
Q

When do you repear cryptorchisdism?

A

wait until 6 months

715
Q

urine leak/overflow

A

incomplete bladder emptying

716
Q

Etiology of stress urinary incontinence

A

urehtral hypermobility , decreased tone of urethral sphincter

717
Q

etiology ofurge urinary incontinence

A

Hyperactivity of detrusor muscle

718
Q

etiology ofoverflow urinary incontinence

A

impaired detrusor activity, bladder outlet obstruction

719
Q

Relationship urinary incontinence overflow and diabetes

A

Patients with diabetic neuropathy may have urinary incontinence . Also look for other signs such as postural hypotension is an evidence of impairment of autonomic system in diabetics

720
Q

Dx overflow incontinence

A

> 200 mL

721
Q

Meds that can cause urinary retention

A

anticholinergics, opioids, alpha 1 agonists

722
Q

Common causes of overflow urinary retention

A

Diabetic neuropathy and BPH

723
Q

Tto of hot flashes

A

If no hysterectomy combined progesterone/estrogen. If hysterectomy estrogen. CI to these therapy include Breast Ca, Stroke, DVT, CHD, liver disease. In women < 60 is considered less risk and if no RF short period 3-5 years.

724
Q

Statin induced myositis medications

A

Macrolides, Gemfibrozile, HIV protease inhibitors Cyclosporine - CYP 3A4 system

725
Q

When do you give Hib vaccine?

A

Usually in < 5years, starting at 2 months.

726
Q

When do you give Hep vaccine

A

0,1,6 ( unless 2kg). If exposed vaccinated: no tto. If exposed unvaccinated: Ig and vaccine. Other : chronic liver disease and high risk ( men sex men, IV drugs, healthcare workers)

727
Q

When pneumococcus

A
19-65
  PPSV23: chronic heart, lung, liver disease, DM, Alcoholic, liver disease
PCV13 PLUS PPSV23 ( high risk patients)
   CSF leak, cochlear implant 
   Sickle cell , asplenia
   immunocompromised 
CKD

> 65 PCV13 plus PPSV23 WITH A 8 WEEK DIFFERENCE

728
Q

When to give herpes zooster vaccine

A

> =60 years

729
Q

Patient that says to doctor No resuscitative measures , but doesnt sign anything.When it happens what to do?

A

Follow next kin desire.

Patient had needed to sign it.
If oral advaced directive is done needs the physician plus two witnesses.

730
Q

tto of Kawasaki

A

Apirin +Ig

731
Q

Kawasaki ppt

A

Rule of 5: 90% < 5 years. Fever >=5 + 4 of the following: 1. Bilateral conjunctitivs, nonexudative 2. Cervical LAD >=1 of 1.5cm, 3. mucositis 4. Rash, 5. erythema and edema of the hands. Patients < 6 months have often atypical kawasaki =<3 criteria but have inflammatory markers CRP, ESR

732
Q

Complications kawasaki

A

Coronary aneurysms, infarct.

733
Q

Intrinsic and extrinsic pathways

A

intrinsic TENET, PTT, HEPARIN…………… Extrinsic tissue factor, VII, PT, Warfarin

734
Q

intrinsic pathway disorders

A

Hemophilia A and B( def of factor presents early) , Von Willebrand disorder, acquired coagulation factor inhibition( presents late)

735
Q

Management atrial flutter

A

Same as Afib. If <48 hrs cardioversion. If > 48 hours: warfarin, rivaroxaban or dabigatran

736
Q

Pulmonary and cardiac manifestations of scleroderma

A

Pulmonary fibrosis presenting as angina, shortness of breath

HF: prominent heave in the lower sternum

737
Q

Peu d’orange,erythema, lymph node involvement dx and tto

A

inflammatory breast cancer

Do a core needle biopsy / Is often confused with mastitis– MASTITIS THAT DOESN’T IMPROVE AFTER ABCS.

738
Q

Types of acne and tto

A

CIN

Comedonal -Topical retinoids, salicilic, glycolid acid

Inflammatory-
Mild: topical retinoids + benzoyl peroxidase
Moderate: Topical antibiotics ( erythromycin, clindamycin)
Severe: oral antibiotics

Nodulo cystic-
Moderate: topical retinoids + benzoyl peroxidase+ topical antibiotics
Severe:add oral antibiotics
Refractory: ISOTRETINOIN

739
Q

Frequent UTIs in the setting of chronic diarrhea, lower abdominal pain, chronic back pain, weight loss

A

THINK OF INFLAMMATORY BOWEL DISEASE –enterovesical fistula. - think of E.coli, bacteroides fragilis.
Pneumaturia can also be seen

740
Q

46 yo with intermenstrual bleeding and new onset heavy bleeding

A

Think about fibroids and endometrial cancer and hyperplasia.– endometrial biopsy is needed

741
Q

First line treatment for hypertension in patients with gout

A

ARBs– losartan, uricosuric event

and you can add Ca channel blockers ( amlodipine) thiazide should be avoided

742
Q

Positive psoas sign

A

psoas abscess, retroperitoneal hemorrhage

743
Q

Patient with sudden onset abdominal pain in right lower quadrant , hypotense, psoas sign positive. Afib noted., next steps

A

Retroperitoneal hematoma, CT scan

744
Q

WaRrfarin anticoagulation

A

Tvit K and FFP

745
Q

Pt with Minimental score 25/30 ( normal>=26) that is able to understand consequences but still refuses to tto

A

RESPECT WISHES- COMPREHENSION, CONSEQUENCS, CHOICE

746
Q

Prior to prescribing antidepressve in depression screen for:

A

Maniac episodes

747
Q

Hypothyroidism + mildly elevated transaminases, next step?

A

ANA ( homogeneous staining) and anti-smooth abs( against actin)- Autoimmune hepatitis - normal ALP, normal bilirubin

748
Q

antimitochondrial antibodies seen in

A

Primary biliary cirrhosis - high ALP

749
Q

Pt with anemia of chronic disease and signs suggesting RA, management of anemia?

A

Underlying cause- anti TNF factors. Caused by Suppression of hematopoiesis by cytokines

750
Q

Iron and ferritin in anemia of chronic disease

A

low iron, high ferritin, normal transferrin

751
Q

Characteristics catatonia

A

psychomotor disturbance seen in very ill psychiatric patients. Posturing against gravity, mutism, negatitivity, immobility– examiner puts arm up and he leaves it up.

752
Q

Tto catatonia

A

BZDs- lorazepam or Electroconvulsive therapy. Lorazepam challenge test confirms diagnosis of catatonia- temporary relief 5-10 minutes after

753
Q

Acute dystonia tto

A

Benztropine or diphenhydramine

754
Q

Akathisia tto

A

betablocker ( propanolol) or BZD

755
Q

Parkinsonism

A

Benztropine or amantadine

756
Q

Tardive dyskinesia

A

no definitive tto but clozapine may help

757
Q

Dantrolene

A

Neuroleptic malignant syndrome - fever, altered mental status, muscle rigidity, autonomic instability

758
Q

difference serotonin syndrome and NMS

A

serotonin: vomiting diarrhea, autonomic instability, can have fever, NO RIGIDITY but YES TREMORS, HYPERREFLEXIA, CLONUS. IMAO ( phenelzine) AND SSRI

759
Q

Strict glycemic control in DM effects on micro, macro, mortality

A

Macro ( MI):No change short term. Micro ( nephropathy, retinopathy): iMPROVE. Mortality : no change or increased.

760
Q

Etiology of serum sickness disease

A

immune complex mediation. B lactams/sulfas. Acute Hep B infection

761
Q

Hep B infection causing serum sickness disease

A

1-2 weeks after exposure. pain in ankle, feet, hands. Then rash can also have purpura areas, fever, polyarthralgia. – polyartheritis nodosa and glomerulonephtritis

762
Q

tto serum sickness reaction

A

retire offending agent, supportive, steroids or plasmpaheresis if needed.

763
Q

What is the best initial approach to stablish trust with an schizophrenic patient

A

maintain interpersonal distance and avoid challenging patients belief. DO NOT NEED TO BE FRIENDLY ALL THE TIME THAT IS SUSPICIOUS.

764
Q

Patient in whom you suspect HIV, and pt refuses to testing. Next step?

A

First step is always to explore the reasoning behind it.

765
Q

Patient positive for HIV, hesistant to tell wife. Next step

A

encourage and support to tell the wife. Physicians are not required to disclose results with third parties but new cases should be reported to the Department of public health.

766
Q

management PCP

A

Lorazepam- restless patient, nystagmus, aggressive behavior, altered mental status. PCP is an NMDA receptor antagonist.

767
Q

Patient with pancreatitis, which type of drug is related?

A

Thiazides ( hydrochlorothiazide, chlorthalidone) and LOOP diuretics ( Furosemide) - ischemia due to increased intravascular volume, and increased viscosity of pancreatic secretions.

768
Q

Medications that can cause pancreatitis

A

Heart failure or hypertension (ACE inhibitors, angiotension II receptor blockers, diuretics)
Autoimmune disease (azathioprine, mesalamine, corticosteroids)
Chronic pain (acetaminophen, opiates, nonsteroidal anti-inflammatory drugs)
Seizure disorder (valproic acid, carbamazepine)
HIV (lamivudine, didanosine, trimethoprim-sulfamethoxazole)

769
Q

VSD

A

Holosystolic murmur. Not continuous ( THAT ONE IS PDA)

770
Q

Acute limb ischemia

A

6Ps:pain, pallor, paresthesia, pulselessness, paralysis, poikylothermia

771
Q

Management limb ischemia

A

Depends: 1. if capillary refill intact-catheter based or surgical revascularization. 2. If no pulses, cold- emergent revascularization.-given risk of myonecrosis in 4-6 hours 3. Non viable - amputation. ALL OF THESE WITH HEPARIN BOLUS FOLLOWED BY INFUSION

772
Q

pain in leg can be

A

DVT OR ACUTE ISCHEMIA!!!

773
Q

Definition of priapism

A

> 4 hours persistent painful erection. Common risk factors for ischemic (low-flow) priapism include phosphodiesterase-5 inhibitors (eg, sildenafil), intracavernosal injections (eg, alprostadil), certain medications (eg, trazodone), and sickle cell disease.

774
Q

Dxand management of priaprism

A

blood gas analysis of a corporeal aspirate. If mild ( urination and col compressess help) if > 4 hours aspiration of corpa cavernosa ( with irrigation saline), or PHENYLEPHRINE

775
Q

When is angiographic embolization indicated in priaprism

A

treatment of choice for nonischemic (high-flow) priapism, which is significantly less common than ischemic priapism and is often due to a traumatic fistula from the cavernosal artery.

776
Q

Patient on SSRI that complains of sexual dysfunction, next step

A

Can switch to bupropion or mirtazapine. Patients who are SSRI responders may benefit from addition of sildenafil, or buproprion.instead of switching. SSRIs:decreased libido in women and men, anorgasmia in women, and delayed ejaculation in men

777
Q

Physician that notices that multiple pts have dyspareunia after procedure of same doctor. Next step

A

Report state medical board, report obstetritian and adverse outcomes

778
Q

When to test for H.pylori

A

diagnosis of peptic ulcer disease, which typically presents with postprandial epigastric pain and early satiety.

779
Q

Girl with abdominal pain > 4 months, inespecific. No other ss, next step

A

Symptoms diary. Likely functional abdominal pain > 2 months, normal examination

780
Q

Management spinal epidural abscess

A

MRI with gadolinium- patient with fever, urinary incontinence, neuro deficit, tachycardia,. IMMEDIATE SURGICAL DECOMPRESSION. STEROIDS ARE NOT GIVEN IN THIS SETTING

781
Q

Organism in spinal epidural abscess

A

Staph aurerus

782
Q

Patient with stroke who received tPA, with elevated BP NEXT steo

A

iV labetalol.Strict control with intravenous medications such as labetalol, nitroprusside, or nicardipine is recommended to keep blood pressure <185/105 mm Hg to avoid the risk of hemorrhagic transformation.

783
Q

patient that will undergo bariatric surgery, considerations pregnancy

A

bariatric surgery is recommenddedbut pregnancy would need to be delayed for at least a year

784
Q

Bariatric ssurgery indications

A

obesity class II (BMI ≥35 kg/m²) with at least 1 comorbidity (eg, diabetes mellitus, hypertension, sleep apnea) or for those in class III (BMI ≥40 kg/m²). Bariatric surgery isrecommended in reproductive-age womenas it reduces the risk of long-term adverse health consequences andreducesseveral obstetriccomplications(eg, preeclampsia, gestational diabetes mellitus)- delay pregnancy for at least a year after bariatric surgery to achieve weight loss goals and stabilize nutritional status – babies can have neural tube defects or preterm delivery.

785
Q

Bariatric surgery does NOT increase risk of miscarriages and is NOT an indication for cesarea

A

True

786
Q

preservation of finger

A

container should have ice mixed with saline or sterile water 50/50. goal temperature of 1-10 C (33.8-50 F). However, the body part should not be cooled below this range to avoid inducing frostbite.

787
Q

tto rosacea

A

Patients with only erythema and telangiectasias: brimodinine. If pt have papular or pustular lesions : topical metronidazole or azelaic acid. Oral antibiotics (eg, metronidazole, tetracyclines) are sometimes required for more severe cases.

788
Q

most common complications rosacea

A

ocular manifestations:burning or foreign body sensations, blepharitis, keratitis, conjunctivitis, corneal ulcers, and recurrent chalazion. Patient with ocular chalazion should have ophtalmology consult

789
Q

Actinic keratoses is precursor of

A

squamous cell carcinoma

790
Q

Fistulous tracts are a complication of

A

Actinomycosis

791
Q

Cerebral venous thrombosis can be a complication of mucormycosis.

A

true

792
Q

Gingo biloba

A

Memory enhancement- increased bleeding risk

793
Q

Gingseng

A

Memory enhancement- increased bleeding risk

794
Q

Saw palmetto

A

BPH- GI issues and increased bleeding

795
Q

Black cohosh

A

postmenopausal symptoms- hepatic injury

796
Q

kava kava

A

anxiety- hepatic injury

797
Q

St.Johns wort

A

Depression,insomnia- hypertension, serotonin sydrome, increased bleeding risk, digoxin

798
Q

Licorice

A

stomach ulcers,bronchitis- hypertension, hypokalemia

799
Q

echinacea

A

flu symptoms- analhylaxis particularly in asthma

800
Q

Ephedra

A

flu, weight loss, athletic performance - HTN, arrhythmia/MI/Sudden death syndrome/Stroke.

801
Q

Which is the most effective intervention that decreases likelihood expansion of aortic aneurysm

A

SMOKE CESSATION!!!! MORE THAN BP control

802
Q

Which is the most common part of aorta that is affected in aortic aneurysm

A

infrarenal aorta

803
Q

whom do you screen for abdominal aortic aneurysm

A

Abdominal US in men 65-75 who have ever smoked

804
Q

3 Indications for surgical repair of aneurysm

A

> 5.5 cm , >0.5 cm expansion in 6 months, AAA associated with PAD or aneurysm-ss.

805
Q

tto of adbominal aortic aneurysm

A

smoking essation, aspirin, statin. Possibly surgery If large, rapidly expanding or sympomatic.

806
Q

18 year old with recurrent UTIs, one sexual partner. Currently with UTI, next step in manangement.

A

ANTIBIOTIC PROPHYLAXIS ( TMP/SMX,Fluoroquinolones, nitrofurantoin) in addition to postcoital void and cranberry juice. Abcs could be given continuously for several years or only post coital. NOT ABDOMINAL US- ANATOMIC ABNORMALITIES WOULD NOT BE EXPECTED AT THIS AGE.

807
Q

When to give antibiotic prophylaxis for UTI

A

more than two UTI in six months, more than 3 in one year.

808
Q

Pt recently treated with amoxicillin for acute otitis media, presents to clinic with no ss and serous otitis media (retracted tympanic membrane with fluid), next step

A

If patient is asymptomatic do not treat. Serous otitis media can persist up to 3 months after treatment - so watchful waiting. If patient is symptomatic, effusion is bilateral, or effusion persists in > 3 months then treat with amoxicillin clavulonate.

809
Q

Tourette syndrome diagnosis

A

Multiple motor tics and >=1 vocal tic ( not necessarily concurrent, > 1 year) , and age of onset < 18.

810
Q

Tourette syndrome management

A

behavioral training ( habit reversal training), RISPERIDONE, ARIPIPRAZOLE ( ANTIPSYCHOTICS), TETRABENAZINE ( Dopamine blocker), ALPHA 2 ADRENERGIC RECEPTOR AGONISTS( Clonidine, guanfacine)

811
Q

Organism in severe intertrigo

A

Candida- miconazole, terbinafine, nystatin

812
Q

Erythrasma

A

in skin fold regions, leaves a dark mark - Corynebacterium minuttism.

813
Q

neurocardiogenic (vasovagal) syncope

A

education and reassurance, physical counterpressure manuevers like leg crossing, tensing arms with clench fist.

814
Q

Management of PAD

A

ESCALATED APPROACH: 1st step: smoking cessation, DM and BP control , aspirin and statin treatment. 1b. Supervised exercise program. 2

815
Q

Patient with signs of meningitis that gram stain is negative, and CSF fluid consistent with meningitis

A

they still can have bacterial meningitis even though the gram stain and culture are negative particularly if they had recent antibiotic therapy.

816
Q

Pregnant women after vehicle collision presents with uterine tenderness, baby with accelerations but no decelerations, and increased contractions. 27 weeks.AND MOM RH -Next step?

A

Kleihauer Betke test to detect and quantify the amount of fetomaternal hemorrhage to determine how much IGD should be given - and continue fetal monitoring.

817
Q

Patient pregnant, uterine tenderness, anemic,

A

always think of contained abruptio of placenta.

818
Q

Patient with COPD who has signficant weight loss and some depressive ss. What is the most common cause of weight loss?

A

Pulmonary cachexia syndrome ( BMI <20%, or weight loss> 5 %). Caused by 1. increased work of breathing ( increased calorie use), systemic inflammation( decreased appetite and skeletal muscle wasting), skeletal muscle hypoxia.

819
Q

Primary ovarian insufficiency causes

A

Turner syndrome, fragile X syndrome, chemotherapy, radiation, galactosemia

820
Q

tto of primary ovarian insufficiency

A

estrogen plus progesterone( if intact uterus)- ideally until 50s or when menopause was supposed to be . Helps with hot flashes and to decrease bone loss. SO NO NEED OF BIPHOSPHONATES

821
Q

Use and SE of Raloxifen

A

for postmenopausal osteoporosis BUT WORSEN HOT FLASHES

822
Q

DKA Management.

A

draw

823
Q

Tight glycemic control benefits

A

nePHROPATHY ( proteinuria, CKD), and retinopathy. NOT PAD, MI, Stroke. Target < 7

824
Q

Thyroid nodule management

A

draw

825
Q

Management of medullary carcinoma

A

Before thinking about resection think of other places for metastasis- check calcitonin, CEA Ag, neck US, abdominal US, plasma free metanephrines, 24 hr metanephrines, catecholamines , RET mutation

826
Q

Patient with cushings characteristics. How to differentiate primary and secondary- management for each

A

Primary ( Adrenal) - high cortisol levels, no suppression with dexamethasone test, suppressed ACTH. - DO CT OF ADRENAL GLANDS- CT and MRI have same sens/spes. Seconday: high cortisol, ACTH should be high or normal.

827
Q

Management of primary enuresis

A

urinalysis, lifestyle modification, enuresis alarm, desmopressin

828
Q

Blood glucose in DKA VS. HHS

A

> 250 vs. 600

829
Q

Alcoholic ketoacidosis

A

anion gap acidosis, increased osmolal gap, ketonemia or ketonuria and variable blood glucose levels. Blood glucose levels are generally higher than 250 mg/dL in patients with diabetic ketoacidosis.

830
Q

Management of alcoholic ketoacidosis

A

IV fluids and THIAMINE. THERE IS NO NEED TO GIVE INSULIN IN THESE CASES

831
Q

Signs of hypercalcemia

A

If mild ( < 12) no symptoms. If more than this: stones (nephrolithiasis), bones (bone pain, arthralgias), abdominal groans (abdominal pain, nausea, vomiting), and psychiatric overtones (anxiety, depression, fatigue)

832
Q

Signs of hypocalcemia

A

Chvostek, and hyperreflexia. numbness and/or tingling of the hands, feet, or lips,
muscle cramps,muscle spasms,seizures,facial twitching,muscle weakness, ightheadedness, and.slow heartbeat.

833
Q

21 alpha hydroxylase

A

ambiguous genitalia, hypotension, cerebro perdedor de sal, hypoNA, hyperK, hypoglycemia, 46 XX

834
Q

11 b hydroxylase

A

ambigous genitalia, hypertension

835
Q

17 alpha hydroxylase

A

all are phenotypically female, hypertension

836
Q

Baclofen and anticholinergic can cause urinary retention

A

TRUE Immediate bladder decompression using urethral or suprapubic catheter is required to prevent progression and acute renal failure. For most patients, urethral catheterization is attempted prior to consideration of suprapubic catheterization.

837
Q

Mc Cune albright syndrome

A

irregular café au lait spots, precocious puberty peripheral, fibrous dysplasia of the bone.

838
Q

Management of renal cell carcinoma

A

If contained within capsule- partial nephrectomy. If extends through the capsule but not the Gerota’s fascia is stage II, and RADICAL NEPHRECTOMY. Radical nephrectomy also for Stage III where there is invasion of major veins, abdominal lymph nodes and adrenal glands. CHEMO/IMMUNOTHERAPY just for metastasis

839
Q

Hyponatremia in CHF

A

CHF-low cardiac output and poor perfusion-ADH secretion– reabsorption of water. MANAGEMENT IS TO WATER RESTRICT!! TOLVAPTAN (VASOPRESSIN ANTAGONIST ) CAN BE USED IN SYMPTOMATIC CHF, OR IN PATIENTS WITH SEVERE HYPONATREMIA < 120.

840
Q

Patient with DM , coming for hypoglycemia, hypotension, weakness, fatigue, weight loss. Labs consistent with hypoglycemia, hypoNatremia, hyperK, EOSINOPHILIA

A

Think adrenal insufficiency- to cosyntropin test( ACTH) and measure cortisol before and after. It should increase if normal. iF INSUFF no response.

841
Q

Patient with hematuria due to exercise, next step

A

repeat urinalysis in 1 week.

842
Q

Hmaturia due to rhabdomyolysis, next step

A

CPK and serum myoglobin

843
Q

2-step approach is involved in screening for GDM.

A

glucose challenge test (GCT) 50-g glucose load. A blood glucose level >140 mg/dL is an indication for the second step, a 3-hour glucose tolerance test (GTT). The GTT is a diagnostic test that consists of a fasting blood glucose and blood glucose levels measured 1, 2, and 3 hours after a 100-g glucose load. GDM is diagnosed when >2 of the GTT values are elevated

844
Q

Targets of glucose in GDM, and management

A

Fasting <95, 1 hour < 140, 2 hour <120. Insulin should be started to reduce risk of shoulder dystocia if these are not met. Metformin and glyburide are other options. BUT first line insulin. After failure of lifestyle modifications

845
Q

Hyperthyrodism in elderly - apathic hyperthyroidism

A

lethargy, apathy, decreased appetite, and weight loss (often termed “apathetic hyperthyroidism”) or have muscle weakness (myopathy). atrial fibrillation and heart failure can be readily triggered by thyrotoxicosis, but tachycardia may be absent due to concurrent medications (eg, beta blockers) or underlying cardiac conduction system disease.

846
Q

Type I diabetic that is on glargine and aspart. He is admitted to the hospital for vomiting and dehydration. How to adjust the regimen of insulin

A

Decrease the basal insulin ( long acting) as they often eat less during hospitalization. And give short term insulin based on the status of glycemia. The goal is to maintain 140 -180 levels.

847
Q

Polyruia, polydipsia, hypertension and hypoK

A

Primary hyperaldosteronism ( Conn syndrome) – measure plasma aldosterone to renin ratio–a ratio of 30 or more is suggestive of excessive aldosterone secretion from the adrenal gland

848
Q

Assess Conn’s syndrome

A

plasma aldosterone to renin ratio

849
Q

Recent bariatric surgery, HyperPTH, back pain, normal calcium, hypophosphatemia,

A

Vit D deficiency. paTIENTS WITH BARIATRIC SURGERY NEED at least 2000-3000 units of cholecalciferol (vitamin D3) per day to maintain vitamin D levels of 30-50 ng/mL,

850
Q

Diabetic neuropathy presentatio and DX

A

bilateral sensory changes, feel burning pain , changes in vibration and proprioception. TUNING FORK TEST IS A GOOD FIRST STEP. – NERVE STUDIES DIAGNOSE IT BUT ARE EXPENSIVE

851
Q

fiRST LINE TREATMENT FOR DIABETIC NEUROPATHY

A

Duloxetine ( SNRIs), gabapentin pregabalin, TCAs. Topical capsaicin. While tricyclic antidepressants are effective pain control agents for diabetic neuropathy, SSRIs are not

852
Q

Patient with normal TSH, normal T4, LOW T3

A

“low T3 syndrome” and is thought to be primarily the result of decreased conversion of T4 to T3. Causes: acute illness,inflammatory cytokines (eg, tumor necrosis factor), starvation, and certain medications (eg, glucocorticoids, amiodarone). REPEAT IN 8 WEEKS.

853
Q

hypocalcemia after transfusion-ACUTE HYPOCALCEMIA TTO

A

An ionized calcium level is required for diagnosis, as serum calcium is often normal. Give CA gluconate or chloride

854
Q

fASTING GLUCOSE IN DM DX

A

Normal fasting glucose <100. If 100-126 impaired fasting glucose and puts you at risk for CAD( even with normal lipid profile). Fasting glucose of > 126 DM

855
Q

Management of asymptomatic bacteriuria in pregnancy

A

cephalexin for 3-7 days, amoxicillin-clavulanate for 3-7 days, or fosfomycin as a single dose.

856
Q

maNAGEMENT OF Pyelonephritis in pregnancy

A

Patients with acute pyelonephritis during pregnancy receive a treatment course of antibiotics. Once treatment is completed, they receive daily antibiotic suppression until 6 weeks postpartum to prevent recurrence. During the first trimester, nitrofurantoin is a second-line antibiotic due to potential associations with congenital defects. It is contraindicated at term due to an increased risk of hemolytic anemia in the newborn.

857
Q

First line tto of cystitis in non pregnant

A

TMP-SMX

858
Q

Treatment of asymptomatic bacteriuria during pregnancy decreases the risk of maternal (eg, pyelonephritis) and fetal complications (eg, preterm birth, low birth weight, perinatal mortality).

A

TRUE

859
Q

Newborn to DM mother with respiratory distress and murmur.

A

Hypertrophic interventricular septum- insulin triggers glycogen synthesis, glycogen gets stored in the myocardium particularly in the septum - they will present with signs of congestive HF - pulmonary edema- tachypnea and respiratory distress

860
Q

Prognosis of hypertrophic cardiomyopathy in newborns of diabetic mom

A

Most cases resolve without surgery

861
Q

RF for hypoplastic left ventricle

A

Mothers with pregestational DM- babies present with cyanosis, and is often recognized in the second trimester.

862
Q

Congenital pulmonary valve stenosis

A

Noonan synrome

863
Q

Delayed puberty in boys

A

lack of testicular enlargement >=4mL by 14 yo. Always test for FSH, LH, Testosterone, prolactin,.Prolactin and TSH tests should also be ordered, as increased levels interfere with GnRH secretion.

864
Q

Brain death algorithm

A

draw. Patients with brain death can have spontaneous movements originating from peripheral nerves or the spinal cord.

865
Q

suicidal patient

A

Patients who are actively suicidal and refusing treatment should be placed on 1:1 observation and hospitalized under involuntary status.

866
Q

TTO IMMUNE THROMBOCYTOPENIA

A

Mild - asymptomatic or cutaneous bleeding (eg, petechiae, purpura) - watch
Moderate/severe - mucosal bleeding (eg, gingival bleeding, epistaxis), internal hemorrhage (eg, hematochezia, intracranial hemorrhage)- < 30,000 -IVIG

867
Q

Dr. that doesn’t want to take care of a patient, what is the next step

A

Ask him to care for him until he has a new doctor. Ensuring continuity of care is fundamental to patient safety.

868
Q

Immunizations in HIV

A

Tdap followed by Td every 10 years, Annual influenza( not the intranasal one as it is alive), PCV13 followed by PPSV3 8 weeks later and then every 5 years after, Hep A and B. Live vaccines are contraindicated except for MMR, Varicella, and Zoster only I if their CD4+ cell counts are >200/mm3 and they have no history of an AIDS-defining illness.

869
Q

septic aborption management

A

Blood & endometrial cultures
Broad-spectrum antibiotics
Suction curettage
Hysterectomy

870
Q

septic aborption complications

A
Myometrial infection/necrosis
Sepsis
Acute respiratory distress syndrome
Disseminated intravascular coagulation
Death
871
Q

antibiotics for septic abortion

A

gentamicin plus clindamycin

872
Q

when do you consider hysterectomy in septic abortion

A

no response to antibiotic or curretage, pelvic abscess and clostridial myonecrosis (eg, pelvic tissue crepitus, radiographic evidence of air within the uterine wall).

873
Q

Patient calling for dysuria. Has a history of UTI, next step?

A

Prescribe TMP/SMX x 3 days if acute uncomplciated- history alone is enough. No need to do urinalysis /urine culture . Alternative is 5 days of nitrofurantoin . IF PATIENT WERE TO BE PREGNANT, WITH S OF VAGINAL INFECTION OR PYELO NEED TO BE SEEN.

874
Q

WHAT IS partial small bowel obstruction.

A

Pt with vomiting, X ray cosnsitent with small bowel obstruction but there is air in colon

875
Q

Management of partial small bowel obstruction

A

observation and supportive care, if patient is not able to to improve in 12-24 hours then surgery.

876
Q

Electro convulsive therapy indications

A

Conditions treated

Unipolar & bipolar depression
Catatonia
Bipolar mania
Specific indications

Treatment resistance
Psychotic features present
Emergency conditions
Refusal to eat or drink
Imminent risk for suicide
Pharmacotherapy contraindicated due to comorbid medical illness or poor tolerability
Pregnancy when pharmacotherapy is undesirable or ineffective
History of ECT response No absolute contraindications
Increased risk
Severe cardiovascular disease, recent myocardial infarction
Space-occupying brain lesion
Recent stroke, unstable aneurysm

877
Q

pregnant women with hx of poorly controlled bipolar, suicidal, risk of hurting child

A

ECT, Lithium is associated with a slightly increased risk of cardiac malformations (Ebstein anomaly) but can be used during pregnancy in patients with severe bipolar illness.

878
Q

hidradenitis suppurativa (HS) (also known as acne inversa), TTO

A

General (all patients)

Weight loss, smoking cessation
Daily skin cleansing of affected area
Hurley stage I (mild disease)
Topical clindamycin
Intralesional steroids or oral antibiotics for flare-ups
Hurley stage II (moderate disease with nodules, sinus tracts & scarring)
Oral tetracyclines (preferred)- doxycycline
Oral clindamycin + rifampin in refractory cases
Hurley stage III (severe disease with diffuse involvement & extensive sinus tracts)
Biologic TNF-alpha inhibitors (eg, infliximab)
Oral retinoids (eg, acitretin)
Surgical excision

879
Q

myocardial ischemia due to acute cocaine intoxication

A

benzodiazepines and nitroglycerin to improve the mismatch of myocardial oxygen supply and demand. Patients with persistent ST elevation despite medical therapy should undergo coronary revascularization without delay.

880
Q

Funduscopic examination shows a small, densely pigmented lesion with irregular borders in the peripheral right choroid. The lesion is about 8 mm in diameter and is minimally raised (1 mm).

A

ocular melanoma from choroid pigmented nevus- Asymptomatic patients with small pigmented lesions (diameter <10 mm, thickness <3 mm) follow-up in 3-6 months. If > 10mm, thickness> 3 mm – RADIOTHERAPY

881
Q

When to do enucleation in ocular melanolma

A

tumors are very large, have extrascleral extension, or severe associated pain. - not the best as it has multiple morbidities

882
Q

Sickle cell trait

A

Hbs A 40 and Hbs S 60. Trait,asymptomatic and does not cause anemia.

883
Q

Alpha thalassemia

A

Hb Barts (4 gamma chains) on electrophoresis,, or HBH ( 3 DELETIONS)

884
Q

B thalassemia

A

mutations in the beta globin genes- Point mutation. HbA>3.5. mild or severe. Target cells

885
Q

Postexposure HIV prophylaxis low and high risk

A

high risk: exposure to blood, semen, mucuous membranes , vaginal secretions , breast milk. LOW RISK ( NO NEED OF PROPHYLAXIS): urine, nasal secretions, saliva, sweat, tears

886
Q

postexposure HIV prophylaxis

A

within 72 hours, for 28days. Triple drug therapy ( tenofovir, emcitirabine, raltegravir). TEST IN 4-6 WEEKS

887
Q

Tuberous sclerosis genetics and ppt

A

AD, TSC 1 ( hamartin) or TSC 2 ( tuberin). Hamartonas in skin and CNS, Angiofibromas in skin, Mitral regurgitation, Ash leaf spots, Cardiac Rhabdomyoma, tUB/AD,Mental retardation, Shagreen spots, renal angyiolipoma.

888
Q

Initial management of childrens who ha seizures and you suspect Tuberous sclerosis

A

utaneous examination, funduscopy, and a brain MRI to evaluate for hamartomas. An electroencephalogram

889
Q

comorbidities is the predominant cause of death in patients with tuberous sclerosis?

A

epilepsy - brain MRI and electroencephalogram always whensuspecting it . Second most common cause is renal falure.

890
Q

pt requesting staying more days at the hospital

A

I understand your concerns, unfortunately nsurance only cover xtra days that are medically necessary

891
Q

lichen sclerosus can evolve to vulvar cancer

A

do a biopsy if lesion seems vulvar cancer single, raised plaque or ulcer covering the labia majora. Treatment of vulvar cancer is with surgical excision and possible chemoradiation.

892
Q

tto lichen sclerosus

A

topical corticosteroids

893
Q

atrophic vaginitis

A

thinned vulvar skin and fusion of the labia minora - estrogen

894
Q

patient with hx of raynaud, GERD, with hypertensive crisis. Most common cause?

A

scleroderma renal crisis . ALWAYS CHECK BP IN SCLERODERMA PATIENTS.

895
Q

pHARMACOTHERAPY FOR HTN CRISIS IN SCLERODERMA

A

ACEIs ( Captopril) and intrvenous nitroprusside if CNS manifestations or papilledema present.

896
Q

Best prognostic factor in primary cns lymphoma

A

the degree of immunosupression- so increase in CD4 count better outcome

897
Q

primary CNS lymphoma tto in HIV

A

radiation, corticosteroids, and HAART

898
Q

Pregnant woman 19 weeks, with vaginal pressure and spotting. Bulging of amniotic fluid through cervix 3 cm dilated

A

Cervical incompetence

899
Q

Prognosis of cervical incompetence with cerclage

A

If cerclage is done prophylactically at 12-14 weeks the prognosis is good. However, if cerclage is done later, and the patient presents with bulgin og amniotic bag through cervix, POOR prognosis and there is no point in doing cerclage.

900
Q

ould provide the greatest benefit for decreasing this patient’s risk of an ischemic cerebrovascular accident

A

control of bp

901
Q

Suspicion of epididymitis, next step

A

order urine analysis and culture, NAAT for gonorrhea and chlamydia. In < 35 is likely STIs, in > 35 due to E.coli

902
Q

Tto of epididymitis

A

Antibiotics- if gonorrhea suspicion ( Ceftriaxone + doxy), if not levofloxacine

903
Q

Presentation of viral orchitis, or when to suspect

A

children/ adolescents - viral meningitis, mumps, bilateral orchitis

904
Q

Management of HIT 2

A

Stop heparin, give argatroban, fondaparinux, . And given that these are not oral patient will need to transition to warfarin once platelets are > 1500

905
Q

acute heart failure due to acute mitral valve regurgitation

A

Pt with sudden onset of short breath, diaphoresis, pulmonary edema, hypotension, late systolic murmur

906
Q

acute heart failure due to acute mitral valve regurgitation tto

A

bedside echo, and emergent surgical intervention

907
Q

When do you see acute heart failure due to acute mitral vavlve regurgitation.

A

Ehler Danlos, Marfan have both mitral valve prolapse but velvety skin with atrophic scarring is consistent with EDS. EDS Abdominal & inguinal hernias
Uterine prolapse

908
Q

dX AND MANAGEMENT OF MUCORMYCOSIS

A

Sinus endoscopy with biopsy and culture. Management is surgical debridement and amphotericin B

909
Q

Pulmonary embolism

A

Sudden shortness of breath, pleuritic pain, pleural friction rub, tachycardia, pleural effusion. IF V/Q mismatch is low probability and you still suspect it don’t let if fool you!

910
Q

Wells criteria

A

3 DVT, alternate diagnosis. +1.5 Previous PE or DVT, tachycardia > 100, surgery/immbolization. Wells> 4 probability high A12

911
Q

Early signs of compartment syndrome

A

AT EXAM TIGHTNESS AREA, PAIN WITH PASSIVE STRETCHING OF THE MUSCLE, MUSCLE WEAKNESS

912
Q

Later stages of compartment syndrome

A

pulse.

913
Q

management of compartment syndrome

A

Compartment pressures should be measured, and pressures >20-30 mm Hg typically require fasciotomy as definitive therapy to relieve high compartment pressure. T

914
Q

complications of transurethral resection of the prostate.

A

retrograde ejaculation

915
Q

Strongest RF for PID

A

FIRST MULTIPLE SEXUAL PARTNERS, THEN PRIOR PID

916
Q

PID tto

A

CEFTRIAXONE PLUS DOXY . If inpatient: Cefoxitin + doxy

917
Q

Infantile botulism

A

inhibition of ACH release - honey, living in farms/rural areas

918
Q

Degeneration of anterior horn cells

A

spinal muscular atrophy,

919
Q

management of infantile botulism

A

otulism immune globulin

920
Q

Pooled human immune globulin

A

tto for Guillain Barre

921
Q

Vertebrobasilar insuffiency

A

vertigo, dizziness, dysarthria, diplopia, and numbness.

922
Q

prognosis of febrile seizures

A

o long-term sequelae
↑ risk of subsequent febrile seizure
Slight ↑ risk of epilepsy (~1%)

923
Q

increased risk preoperatively for anesthetic and operative drug reactions.

A

multiple drugs

924
Q

Immunocompromised patient with pneumonia, organism? Tto?

A

PCP, tmp/smx + corticosteroids. Hypoxia, ground grass opaicites on chest x ray

925
Q

Vaginal spotting in elderly-

A

SEVERE ORGAN PROLAPSE can cause it!! . If cervical cancer, you would be able to see something in the speculum . severe prolapse cause vaginal erosions that lead to abnormal vaginal bleeding (eg, postmenopausal bleeding, postcoital spotting). Treatment of prolapse is via pessary or surgical correction; erosions can be treated with vaginal estrogen.

926
Q

Intussusspetion

A

AIR OR SALINE ENEMA . NOT BARIUM– RISK OF PERITONITIS

927
Q

intussuseption is a medical emergency!!

A

if not available someone to consent– do it without consent/

928
Q

Pt that underwent air enema for intussusspetion, and one hour later has severe abdominal pain

A

order Abdominal X rays- to evaluate for perforation .

929
Q

pregnant woman with HPV lesiosn, in labor. Next step

A

expectant manangement and vaginal delivery . Unless the lesions are too big that obstruct birth canal

930
Q

Treatment of hemochromatosis

A

therapeutic phlebotomy

931
Q

Wilsons disease tto

A

Penicillamine

932
Q

Hemochromatosis can present with

A

infertility, diabetes, bronze skin

933
Q

Patient with symptoms of renal stone, CT negative, and history of analgesic use for a lot of time, and has BUN AND CREAT HIGH

A

Anaglesic nephropathy– causes papillary necrosis and can present similarly

934
Q

Turner syndrome has normal intelligence?

A

YES , low set ears, micrognathia, streak ovaries, they have normal IQ but are at risk of learning disabilitis

935
Q

Turner syndrome genetic defect

A

NONDISJUNCTION 45 X, sporadic event. - probability to have another pregnancy with that is similar to rest of population

936
Q

BACTERIAL ENTERITIS- BLOODY IN KIDS-NEXT STEP AFTER OBTAINING CULTURE

A

FLUID EVEN IF NOT DEHYDRATED. ALWAYS THINK OF E.COLI , SALMINELLA, SHIGELLA. , E.COLI, Plain water may exacerbate electrolyte disturbances and lead to dangerous hyponatremia or hypoglycemia. Juice is also not recommended as its high sorbitol content can exacerbate diarrhea.

937
Q

Causes of transient PSA elevation

A

Urinary retention, mild acute prostate inflammation or infection, urologic procedure ( cystoscopy) ,digital recta exam ( minimal elevation of PSA), recent ejaculation — recheck in 6-weeks

938
Q

Persistent elvations of PSA

A

bph, prostate cancer, or severe prostatitis

939
Q

tto of keloids

A

intralesional glucocorticoids, but risk of recurrence so sometimes may require surgical excision

940
Q

pt with ALS complaining of difficulty sleeping, orthopnea, dysphagia, dyspnea, and has fasciculations,

A

diaphragmatic paralysis

941
Q

tto of salmonella

A

NO ANTIBIOTIC USE!!!! SUPPORTIVE TREATMENT IN IMMUNOCOMPETENT AND OLDER OF 12 MONTHS.

942
Q

treatment of subclinical hypothyroidism, and when do you treat

A

treat if:

  1. . TSH >10
  2. Antithyroid peroxidase abs positive
  3. Pregnancy +, infertility or ovulatory dysfunction, goiter, symptoms, abnormal lipid profile ( hypercholesterolemia)

tto : levothyroxine

943
Q

When do you screen for thyroid disease

A

is controversial:( 2 pts of view)
1. all patients > 40
2 >50 women with ss

944
Q

in patients who present with HTN < 30 years of age, or aa worsening of HTN after being controlled for a long time, think of

A

secondary HTN

In the second case, renovascular HTN

945
Q

Patient with controlled HTN with 2 meds and everything has been the same in his life, that now has increased BP., decreased peripheral pulses, carotid bruitNEXT STEP

A

Think of renovascular HTN ( IN THE SETTINGOF ATHEROSCLEROSIS)
NEXT STEO– MR angiography of renal vasculature

946
Q

do inhakled corticosteroids for asthma cause hyperglycemia

A

NO. the dosing , and appropirate use with spacers should not cause hyperglycemia.

It is most commonly seen with oral corticosteroids

947
Q

patient with fever> 39, nausea and diarrhea. Dehydrated somnolent. Labs with hyperglycemia 200, ketones in urine, hb A1C 5.6. What is the cause of hyperglycemia?

A
stress hyperglycemia ( caused by high circulating stress hormones)
seen with fever > 39
severe illness
sepsis
meningitis
ICU 
  • not DM because no typical ss of polyuria, polydipsia, weight loss. Hb A1C not show long time DM
948
Q

16 yo with tiredness, weigh loss, amenorrhea, decreased appetite, multiple freckles, patchy brown spots on mucosa, and lips, decreased amount of axillary and decreased axillary and pubic hair. labs with hyponatremia and hyperK, next step?

A

suspect Addison’s disease or adrenal insuff

order cortisol with ACTH
with cosyntropin

949
Q

Hypoaldosteronism does not present with hyperpigmentation

A

hyperpigmentation is not seen as there is no increased production of ACTH/melanocyte-stimulating hormone.

950
Q

Patient IV user who has malaise, nausea, mjaundice, RUQ pain, elevated LFTs, hep B labs negative, Hep C antibodies negative, hep A antibody negative. Cause?

A

it is highly suspicious for HEPATITIS. Hep C antibodies are not useful to detect infection in earlier stages as they appear only 2-6 months after exposure. SO ORDER HCV RNA BY PCR BECAUSE THIS CAN BE DETECTED WITHIN DAYS.

951
Q

67 yo smoker, COPD, who had pneumonia 3 weeks ago, now presenting with pneumonia. next step?

A

CT chest! in the setting of non-resolving pneumnia particularly on the same place and in a smoke think of endobronchial neoplasm. Other things that the CT will be helpful to rule out are: abscess and empyema.

952
Q

factors associated with euthanasia request

A

MC:

  • loss of autonomy and control of dying process
  • fears of future suffering

others include:

  • loss of dignity
  • inability to engage in pleasurable activities

” WHEN THEY REQUESTED: ASK PLEASE TELL ME MORE ABOUT WHY YOU FEEL THAT WAY”

953
Q

ASCITIS FLUID ANALYSIS- SAAG

A

SAAG: Serum albumin- Ascitic fluid albumin

SAAG>=1.1 : Indicates portal hypertension : CHF, hepatic cirrhosis, alcoholic hepatitis, Budd Chiari

SAAG< 1.1 : Peritoneal carcinomatosis, peritoneal TB, nephrotic Sx, pancreatitis, serositis.

954
Q

patient with ADHD who will need medication. Has a family hx of substance abuse and he has tried a couple of times alcohol. any risks with ADHD medication and substance abuse?

A

NO. THERE IS NO DATA SHOWING THAT ADHD MEDS INCREASE RISK FOR SUBSTANCE ABUSE

955
Q

Elderly alcoholic with abdominal pain after feeds that improve with sitting, postprandial bloating and discomfort, and large stools that require multiple flushes. next step?

A

MR cholangiopancreatography or an abdominal CT scan

Chronic pancreatitis- so this image will show calcifications which is patognomonic . other signs: ductal dilation, eudocysts.

they also may develop diabetes.

**amylase and lipase may be a little bit elevated but are freq normal

956
Q

tto of chronic pancreatitis

A
alcohol and smoking cessation
pain management
frequent small meals
pancreatic enzymess
f/u in 1-2 months.
957
Q

Fat embolism

A

after fx
shortness of breath hypoxemia
petechial rash
confusion

dx:presence of fat droplets in urine or intra-arterial fat globules on fundoscopy

Serial X-rays: increasing diffuse b/l pulm. Infiltrates within 24-48 hours of onset of clinical findings

958
Q

What can be done to prevent fat embolism

A

early immobilization and correction of fracture.

959
Q

tto fat emboslism

A

supportive.

960
Q

wat to ask about suicide

A

ideation, intent and plan

961
Q

brief psychotic disorder>

A

> 1 day and < 1 month. sudden onset and full return to function.

962
Q

schizophrenifrom disorder

A

> 1 m and < 6 months. Same as schizophrenia , functional decline not required

963
Q

schizophrenia

A

> 6 months ( includes at least 1 month of active ss, and can include prodromal and residual periods), requires functional decline

964
Q

schizoaffective disorder

A

concurrent mood episode + ss of schizophrenia

and at least 2 week lifetime hx of delusions/hallucinations in the abscence of mood ss

965
Q

Cannabis can lead to psychosis?

A

High-potency cannabis has been associated with paranoia, depersonalization, and hallucinations.

-Some experts suggest that early cannabis use may be a causal factor in developing schizophrenia in those with susceptibility

966
Q

cocaine can lead to psychosis

A

ocaine intoxication and withdrawal can also present with acute psychotic symptoms.

967
Q

kid with psychosis hx of substance abuse and family hx of of schizophrenia

A

always rule out substance induced psychosis and consider timing to dx schizphrenia

968
Q

tto PTSD

A

SSRIS
trauma- focused psychotherapy
fOR nightmares: prazosin

969
Q

always screen for PTSD in veterans

A

individuals experiencing new-onset anxiety, insomnia, interpersonal conflicts, or escalating substance use.

970
Q

Elderly with torsaides de pointes. Causes?

A

bradyarrhythmias.

Also think of ABCDE
Electrolyte disturbancesHypoMg, HypoK, HypoCa
hypothermia
Sinus dynfucntion , AV blocks, 
hypothyroidism
starvation
971
Q

Electrolyte disturbances causing Long QT syndrome

A

hypoK, HypoMg, HypoCa

972
Q

tto torsaides de points independent of the cause

A

First give IV Mg

then if not helpful temporary transvenous pacing

973
Q

Can ABO incompatibility - acute hemolytic reaction occur in a patient who has been transfused multiple times?

A

Yes-Patients who require chronic red cell transfusions (eg, African American patients with sickle cell anemia) may form multiple antibodies to common Rh, Kell, or other blood group antigens.

974
Q

tto of acute hemolytic reaction

A

stop transfusion and IV NS ( do not give LR) to treat hypotension and prevent renal failure

975
Q

Febrile nonhemolytic transfusion reactions can be prevented?

A

no, Premedication with antihistamines and acetaminophen does not prevent blood transfusion reactions

976
Q

Patient who developed peripartum cardiomyopathy, and now is pregnant with the second baby and wants to know the risk ?

A

Some patients can have spotaneous resolution of ss,however, there is a risk of recurrence, with patients with LVEF <20% at diagnosis at highest risk.

Patients with persistent PPCM are at risk for further LV function decline and death in subsequent pregnancies.

REGARDLESS OF RESOLUTION OF SYMPTOMS THESE MOMS ARE FOLLOWED WITH SERIAL ECGS and if there is further LV dysfunction they are recommended to avoid pregnancy

977
Q

presentation of peripartum cardiomyopathy

A

36 weeks gestation and 5 months postpartum.
Risk factors include maternal age >30, multiple gestation, and preeclampsia.
Patients present with left heart failure symptoms (eg, dyspnea), left ventricular ejection fraction (LVEF) <45%, and no other cause of heart failure (ie, diagnosis of exclusion).

978
Q

Management. ofperipartum cardiomyopathy: Hydralazine, nitrates and delivery based on mom stability .

Adter delivery how should it be managed?

A

serial TTE

979
Q

PPT AND PATHOPHYSIOLOGY OF SEBORREIC KERATOSIS

A

proliferation of immature keratinocytes.
after 50s
well-demarcated, pigmented, round or oval, and have a dull or verrucous surface with a “stuck-on” appearance. Typical locations include the face, upper extremities, and trunk.

CAN ITCH OR NOT

Explosive onset of multiple pruritic SKs (Leser-Trélat sign) has been associated with malignancies (especially lung and gastrointestinal tumors).

980
Q

Tto of seborreic dermatitis

A

reassurance, is a benign tumor

If symptomatic or cause cosmetic problems cryotherapy or removed by curettage/shave excision or electrodessication

981
Q

tto of expanding neck hematoma- tracheal deviation, hoarseness

A

intubation

982
Q

malignant otiis media

A

purulent drainage, ear pain in diabetes/HIV

pseudomona

983
Q

Ramsay Hunt syndrome

A

(also known as herpes zoster oticus)

ear pain, external auditory vesicles, and ipsilateral facial paralysis.

984
Q

TTO OF MALIGNANT OTITIS EXTERNA

A

CIPROFLOXACIN IV and once ESR/CRP normalize can transition to oral. to complete 6-8 weeks of therapy is recommended.

Other options: anti-pseudomonals: piperacilin, ticarcilin
third generation cephalosporins: ceftazidime

985
Q

pt with prostate cancer and mestastasis to lumbar spine, in managementt with narcotics but still ver painful. no bladder/rectal incont or neuro abnorm. next step

A

External beam radiation therapy should be used for pain alleviation in patients with single or few focal bone metastatic lesions due to hormone-refractory prostate cancer.

*surgery only indicated if neuro abn

986
Q

Woman 55 yo concerned about Colon Ca. Her sister was recently diagnosed. She is asymptomatic and had a colonoscopy 5 years ago which was normal.

A

a patient at average risk should have a repeat colonoscopy every 10 years.

987
Q

Patients at average risk colon cancer -screening

A

Start screening at age 50. Options include:

Colonoscopy every 10 years
gFOBT or FIT every year
FIT-DNA every 1-3 years
CT colonography every 5 years
Flexible sigmoidoscopy every 5 years (or every 10 years combined with annual FIT)
988
Q

Patients at risk colon cancer -screening

A

Colonoscopy at age 40 or 10 years before the age of cancer diagnosis in a relative (whichever comes first)
Repeat every 3-5 years